ngoại tiếp tam giác ABC. Do đó trục đẳng phương của đường.. Đường thẳng qua B, C lần lượt vuông góc với AB, AC theo thứ tự cắt AH tại X, Y. a) Theo bài toán 2 thì BC là trục đẳng phương [r]
(1)“Mỗi bảng hình chữ nhật 4×4 được điền số −1 Biết rằng tổng số hàng cột Hỏi có cách điền khác nhau?”
“Các toán tổ hợp liên quan số “Bài tốn “chốt hình chữ nhật”
nhìn giống loại câu hỏi mà học sinh trung học giải được với compa thước kẻ,
nhưng cản đường nỗ lực cao nhà toán học trong nhiều thập kỷ.”
“Các quan điểm hình học việc giải tốn cũ hình chữ nhật” - KEVIN HARTNETT
THUẬT TOÁN GIÚP TĂNG TỐC ĐỘ XÉT NGHIỆM CORONA
Smriti Mallapaty
ĐUỔI BẮT VÀ CHẠY THOÁT Trịnh Đào Chiến
VÀ CÁC CHUYÊN MỤC KHÁC
sta
y home sta
y saf
(2)(3)"Nắng tháng vàng rực đường, mùa hè đến độ chín mùi Những vui, những chuyến xa dịp tưng bừng Nhưng đâu đó, vào độ tháng tám lưng chừng, vài mưa đến, vài gió sớm mát lành đưa đến mùi vị mùa thu tựu trường chẳng cịn bao xa."
Đó lời tựa mà viết cho Epsilon số 10 Bốn năm sau, sống trong năm kỳ lạ bậc lịch sử nhân loại, năm 2020 hay gọi năm đại dịch Covid-19 Mùa hè khơng cịn cũ, niềm vui quây quần bên bạn bè gia đình, chuyến du lịch khám phá khác xa
Nhưng, không chùn bước, chiến đấu, lại lần vượt qua đại dịch nhân loại vượt qua vơ vàn khó khăn lịch sử Bởi lẽ, nhiều người, ta xa
"Đi nhiều người, ta xa " Đó tinh thần Epsilon, dẫn mà theo đuổi từ ngày
Bởi lẽ,Epsilon, tức nhỏ, khơng Và nhiều epsilon cộng lại trở thành những đáng kể Có thể 1, 2, vơ cùng.
Bởi lẽ,Epsilon tờ báo cộng đồng, dành cho cộng đồng.
(4)Lý Ngọc Tuệ
Tính bậc hai số nào? 6
Phan Thành Nam
Cuộc hôn nhân hai tốn hình học ngun tử? 11
Nguyễn Lê Anh
Huyền sử đời Hùng có phải dân tộc ta? 29
Olivier Bleu, Dmitry Solnyshkov, Guillaume Malpuech
Chế ngự Photon Topo 35
Kevin Hartnett
Góc nhìn cho toán cũ? 42
Hideki Yukawa
Thơ Khoa học - Viết cho trẻ em 54
Smriti Mallapaty
Thuật toán giúp tăng tốc độ xét nghiệm Corona 58
Trịnh Đào Chiến
Đuổi bắt chạy thoát 63
Nguyễn Thái Vũ
Thặng dư toàn phương, bổ đề Gauss luật thuận nghịch bậc hai 72
Trần Nam Dũng
Câu chuyện bất đẳng thức 86
Lê Phúc Lữ
Các toán tổ hợp liên quan số học 90
Lê Phúc Lữ, Trần Bá Đạt
Giới thiệu số đề thi Olympic Online 99
Võ Quốc Bá Cẩn
Một số toán phương trình hàm đề thi Olympic Tốn 112
Nguyễn Duy Phước
Khám phá từ toán hay 130
Lê Viết Ân
(5)Trần Quang Hùng
(6)TÍNH CĂN BẬC HAI CỦA MỘT SỐ NHƯ THẾ NÀO?
Lý Ngọc Tuệ
TÓM TẮT
Ai học luyện tập cách thành thục cách chia số theo chữ số trường học phổ thông Tuy nhiên với phép tính khác, chẳng hạn bậc 2√X, phải tính tốn tay giá trị cụ thể đó, chẳng hạn nhưp2,357 Và cần phẩi tính tốn giá trị cụ thể vậy, máy
tính cầm tay, hay phần mềm tính tốn máy vi tính điện thoại cho đáp số cách nhanh chóng Làm hộp đen tính tốn phép tính có lẽ điều bí ẩn với nhiều người, kết nhận từ chúng lại khơng xác Một ví dụ tiếng phần mềm Calculator Microsoft Windows tính ra√4 = 1,9999999 thời gian dài
Lỗi Microsoft sửa khoảng năm trở lại
Trong này, tìm hiểu mở rộng phương pháp tính chia theo chữ số để tính xác phép tốn khác tìm bậc hai √X, nghịch đảo bậc hai√1
X
1 Thuật tốn tính chia theo chữ số.
Cho hai số dươngX, Y >0vàb ≥2là số cố định GọiZ = X
Y thương số phép
chiaX choY, biểu diễn dạng chuẩn hóa với sốb ≥2:
Z =be·z0, z1z2 ., zi ∈ {0,1, , b−1}, z0 6= (1.1) ĐặtZnlà(n+ 1)chữ số củaZ:
Zn=z0, z1z2 zn,
chúng ta có mối liên hệ hiển nhiên giữaZnvàZn−1như sau:
Zn=Zn−1+znb−n (1.2)
Bên cạnh đó,Z vàZnbị chặn hiển nhiên bất đẳng thức:
(7)Phần dư thứncủa phép chia X
Y định nghĩa sau:
Rn:=bn(X0 −Y Zn) với X0 =b−eX (1.4)
Khi đấy, bất đẳng thức (1.3) tương đương với:
0≤Rn < Y (1.5)
Từ mối quan hệ giữaZn vàZn−1 (1.2), có công thức truy hồi choRnnhư
sau:
Rn =bn(X0−Y Zn)
=bn X0 −Y Zn−1 +znb−n
=bn X0 −Y Zn−1−znY b−n
=b·bn−1(X0−Y Zn−1)−znY
=bRn−1−znY
(1.6)
Thay (1.6) vào điều kiện (1.5):
0≤bRn−1−znY < Y ⇐⇒ 0≤
bRn−1
Y −zn <1
Hay nói cách khác,znchính phần nguyên
bRn−1
Y , cơng thức quen thuộc để
tìm chữ số thứncủa thương sốZ = X
Y : zn=
bRn−1
Y
(1.7)
Lưu ý sốb= 2, công thức (1.7) đơn giản hóa thành:
zn =
(
1 ,2Rn−1 ≥Y
0 ,2Rn−1 < Y
(1.8)
Chúng ta tổng kết lại thuật tốn tìm thương số dương sau:
Thuật toán 1. X
Y =b
e
·z0, z1z2 .là thương số phép chiaXchoY biểu diễn với sốb≥2có thể tìm thấy sau:
1 elà số nguyên lớn chobeY ≤X,
2
zn =
bRn−1
Y
Rn =bRn−1−znY
(8)2 Thuật tốn tính bậc hai theo chữ số.
Áp dụng phân tích phần trước, với số dương cho trướcX > 0, đặtZ = √X
bậc hai củaX, biểu diễn dạng chuẩn hóa với sốb≥2như (1.1)
Chúng ta định nghĩa phần dư thứncủa phép tính bậc 2√Xbởi: Rn :=bn X0−Zn2
với X0 =b−2eX (2.1)
Từ (1.3), ta suy chặn phần dưRnnhư sau:
0≤Rn=bn X0−Zn2
=bn√X0 −Zn√X0+Zn
=bn b−eZ−Zn
b−eZ+Zn
< bnb−n 2Zn+b−n
= 2Zn+b−n
(2.2)
Tương tự phần trước, công thức truy hồi choRncũng tính dựa vào (1.2) sau:
Rn =bn X0−Zn2
=bnX0− Z
n−1+znb−n
2
=bn X0− Zn2−1 + 2znZn−1b−n+zn2b−2n
=bn X0−Zn2−1− 2znZn−1+z2nb−n
=bRn−1− 2znZn−1+zn2b−n
(2.3)
Dựa vào công thức truy hồi này, có cơng thức tìm chữ số thứnnhư sau: zn= max
d∈ {0,1, , b−1}: 2dZn−1+d2b−n≤bRn−1 (2.4) Để thấy tính đắn cơng thức (2.4), giả sử chọn chữ sốznsao cho:
2(zn+ 1)Zn−1+ (zn+ 1)2b−n≤bRn−1 (2.5) Lưu ý điều kiện (2.5) đủ tổng quát vì2dZn−1+d2b−nlà hàm đơn điệu tăng vớid∈[0, b] Áp dụng (2.5) vào (2.3), ta có được:
Rn =bRn−1− 2znZn−1+zn2b−n
≥ 2(zn+ 1)Zn−1+ (zn+ 1)2b−n
− 2znZn−1+z2nb−n
= 2Zn−1+ 2znb−n+b−n
= 2(Zn−1+znb−n) +b−n
= 2Zn+b−n
(9)trái với (2.2)
Khi sốb= 2, công thức (2.4) đơn giản hóa thành:
zn=
(
1 ,2Rn−1 ≥2Zn−1+b−n
0 ,2Rn−1 <2Zn−1+b−n
(2.6)
Để ý với công thức (1.7) cho phép chia không phụ thuộc vào chữ số trước thương số, mà phụ thuộc vào phần dưRn−1 số bị chiaY Tuy nhiên, công thức (2.4) tìm chữ số thừn, cho√X phụ thuộc vào phần dưRn−1và kết quảZn−1 bước trước
Chúng ta tổng kết lại thuật tốn tìm bậc số dươngX:
Thuật toán 2. √X = be·z0, z1z2 .là bậc hai củaX biểu diễn với số b ≥ 2có thể tìm thấy sau:
1 elà số nguyên lớn chob2e ≤X,
2 (
zn = max
d∈ {0,1, , b−1}: 2dZn−1 +d2b−n≤bRn−1
Rn =bRn−1 − 2znZn−1+z2nb−n
với điều kiện ban đầu
(
Z−1 =
R−1 =b−2eX
3 Thuật tốn tính nghịch đảo bậc hai theo từng chữ số.
Với số dương cho trướcX > 0, đặtZ = √1
X nghịch đảo bậc hai củaX,
biểu diễn dạng chuẩn hóa với sốb≥2như (1.1)
Chúng ta định nghĩa phần dư thứncủa phép tính nghịch đảo bậc √1
X bởi: Rn:=bn 1−X0Zn2
với X0 =b−2eX. (3.1)
Từ (1.3), ta suy chặn phần dưRnnhư sau:
0≤Rn=bn 1−X0Zn2
=bn1−√X0Zn1 +√X0Zn
≤bn√X0 b−eZ−Z n
·2
<2bnb−n√X0
= 2√X0.
(10)Cơng thức truy hồi choRnđược tính dựa vào (1.2) sau:
Rn=bn 1−X0Zn2
=bn1−X0 Zn−1+znb−n
2
=bn 1−X0 Zn2−1+ 2znZn−1b−n+zn2b−2n
=bn 1−X0Zn2−1−X0 2znZn−1+zn2b−n
=bRn−1 −X0 2znZn−1+zn2b−n
(3.3)
Bài tập 3. Chứng minh tính đắn cơng thức tìm chữ số thứ ncho nghịch đảo
bậc củaX:
zn= max
d∈ {0,1, , b−1}:X0 2dZn−1+d2b−n
≤bRn−1 (3.4)
Thuật toán 4. √1
X = b
e
·z0, z1z2 .là nghịch đảo bậc hai củaXđược biểu diễn với sốb ≥2có thể tìm thấy sau:
1 elà số nguyên lớn chob2eX ≤1,
2 (
zn = max
d∈ {0,1, , b−1}:b−2eX· 2dZn−1+d2b−n
≤bRn−1
Rn =bRn−1 −b−2eX· 2znZn−1+zn2b−n
với điều kiện ban đầu
(
Z−1 =
R−1 = 1−b−2eX
Bài tập 5. Tim thuật tốn để tính bậc 3√3
X theo chữ số
Bài tập 6. Tim thuật tốn tổng qt để tính bậc n √n
X theo chữ số
Tài liệu
[1] M D Ercegovac, T Lang,Digital Arithmetic, Morgan Kaufmann (2004).
[2] J M Muller, Elementary Functions - Algorithms and Implementation, ed 3, Birkhăauser (2016)
(11)CUỘC HƠN NHÂN CỦA HAI BÀI TỐN HÌNH HỌC
TRONG MỘT NGUYÊN TỬ
Phan Thành Nam
Đại học Munich, Đức
GIỚI THIỆU
Năm1928;nhà vật lý George Gramov người Nga đưa mô hình dự đốn tồn
và phân rã hạt nhân nguyên tử Lý thuyết Gramov kết hợp hai tốn hình học cổ điển:Bài tốn đẳng chuđặt từ trước cơng ngun vàbài toán lượng hấp dẫnđặt Isaac Newton kỷ XVII Tuy phát biểu đơn giản, ẩn giấu nhiều mối quan hệ sâu sắc hình học, giải tích vật lý, nhiều kết thú vị khám phá gần Trong viết này, tác giả giới thiệu toán Gramov với số vấn đề liên quan
1 Bất đẳng thức hoán vị
Chúng ta bắt đầu câu chuyện với toán quen thuộc phổ thông
Định lý (Bất đẳng thức hoán vị) Cho hai dãy số thực giảma1 > a2 > > an và
b1>b2 > >bn Khi với hoán vị.b 1/; : : : ; b n//của.b1; : : : ; bn/ta có
a1b1C Canbn>a1b 1/C Canb n/ >a1bnC Canb1: (1)
Bất đẳng thức (1) chứng minh quy nạp theon:Từ bất hốn vị ta suy nhiều kết quan trọng, ví dụ bất đẳng thức Chebyshev cho hai dãy số thực tăng giảm
a1b1C Canbn
n >
a1C Can
n
b1C Cbn
n ;
bất đẳng thức trung bình cộng - trung bình nhân (AM-GM) bất đẳng thức Cauchy-Schwarz (trong sách giáo khoa Việt Nam gọi bất đẳng thức Cauchy bất đẳng thức Bunyakovsky) Nói nơm na, giả sử rằngai tài sản chàng trai thứi,bi tài sản cô gái thứi, vàaibi
(12)nên kết hôn kiểu “môn đăng hộ đối” Tất nhiên, thực tế khác xa với mơ hình này, “tiền bạc khơng phải tất cả”.
Tổng quát hoá lên nhiều chiều:Bất đẳng thức hốn vị tổng qt hố cho hàm số thực
Rd
! R, với tổng thay tích phân Để phát biểu kết này, cần định
nghĩa xác hoán vị giảm nhiều chiều Trước hết, xét hàm đặc trưngcủa tập hợptrongRd, tức
.x/D (
1 nếux 2 nếux …
Ta định nghĩa hoán vị đối xứng giảm (symmetric-decreasing rearrangement) củalà
./ D;
trong đó cầu trongRd có tâm tại0và có độ đojj D jj(độ đo độ dài trong1 chiều, diện tích trong2chiều, thể tích trong3chiều, )
Để mở rộng định nghĩa cho hàm sốf WRd !Œ0;1/, ta sử dụng công thức
f x/D Z t
0
ff >tg.x/dt;
trong đóff > tg D fx Rd W f x/ > tggọi “level set” Với cách nhìn này, ta định nghĩa
hốn vị đối xứng giảmfWRd !Œ0;1/bởi f.x/D
Z
0
ff >tg.x/dt:
Nói cách khác, “level set” củaflàfx Wf.x/ > tg D fxWf x/ > tg:Rõ ràngfđối xứng giảm, tức làf.x/6f.y/khijxj6jyj Hơn nữa, ta có
Z
Rd
f.x/dxD Z
Rd
f x/dx; nhờ “layer cake representation”
Z
Rd
f x/dx D Z
0
jfx Wf x/ > tgjdt:
Trong cơng thức trên, định nghĩa tích phân Lebesgue trongRd thơng qua tích phân
Riemann trongR Nói nơm na, muốn tính tổng thu nhập tháng, với cách làm Riemann tính thu nhập ngày sau cộng dồn, với cách làm Lebesgue đếm có tờ mệnh giá 500ngàn, 200ngàn,100 ngàn, cộng lại Ở chương trình phổ thơng, học cách tính tích phân theo kiểu Riemann Tuy nhiên, cách tính kiểu Lebesgue áp dụng cho lớp hàm số rộng (trong viết này, hàm số đo theo nghĩa Lebesgue)
(13)Định lý (Bất đẳng thức Hardy-Littlewood) Với hai hàm sốf; g WRd !Œ0;1/, ta có Z
Rd
f x/g.x/dx
Z
Rd
f.x/g.x/dx:
Đặc biệt, khif vàglà hàm đặc trưng hai tập hợp1; 2trongRd, ta có j1\2j6j1\2j Dmin.j1j;j2j/Dmin.j1j;j2j/:
Điều hiển nhiên, mặt trực giác hai tập hợp có tính đối xứng chúng “giống nhau” có nhiều “điểm chung” Bất đẳng thức Hardy-Littlewood chứng minh dựa trường hợp đặc biệt nói trên, với “layer cake representation” định lý Fubini (bạn đọc xem [1], chương3)
Z
Z
Z
Rd
ff >tg.x/fg>sg.x/dx ds dt
Z
Z
Z
Rd
ff >tg.x/fg>sg.x/ dx ds dt:
Một dạng tổng quát bất đẳng thức Hardy-Littlewood
Định lý (Bất đẳng thức Riesz) Cho ba hàm sốf; g; hWRd !Œ0;1/tuỳ ý Ta có Z
Rd
Z
Rd
f x/g.x y/h.y/dxdy
Z
Rd
Z
Rd
f.x/g.x y/h.x/dxdy:
Cấu trúc tích chập gh/.x/ D R
Rdg.x y/h.y/dy cho phép ta hiểug.x y/như nhân (kernel) toán tử Do đó, nhờ tính đối ngẫu, bất đẳng thức Riesz dẫn tới nhiều ứng dụng sâu sắc Chẳng hạn chọnglà “heat kernel”, ta có
Z
Rdjr
f x/j2dx>
Z
Rdjr
f.x/j2dx:
Ở đâyrf gradient (vector đạo hàm riêng phần) củaf Tổng quát hơn, ta có
Định lý (Bất đẳng thc PúlyaSzegăo) Vif WRd !Rvp>1;ta cú Z
Rdjr
f x/jpdx>
Z
Rd jr
f.x/jpdx:
(14)2 Bài toán đẳng chu
Trong trường hợp đặc biệtp D 1, f D , đại lượng R
Rdjrj trở thành Per./ D chu vi (perimeter) của(trong3chiều diện tích bề mặt của) Khi bt ng thc PúlyaSzegăo tr thnh bt ng thc ng chu (isoperimetric inequality)
Per./>Per./:
Điều nói số tập hợp Rd có độ đo, cầu có chu vi nhỏ nhất.
Một cách tương đương, ta có
Định lý (Bài tốn đẳng chu) Trong số tập hợp trongRd có chu vi, cầu có
độ đo lớn nhất.
Trong2chiều, kết nói số hình có diện tích hình trịn có chu vi nhỏ Những người Hi Lạp cổ đại biết điều hàng trăm năm trước công nguyên Một so sánh nhỏ trình độ tốn học đáng kinh ngạc họ: Trong sách “Đại Thành Toán Pháp” cụ Lương Thế Vinh 1441 1496/ta biết 3, trước khoảng 1700năm sách “On the Measurement of the Circle” cụ Archimedes (285 212trước cơng ngun) tính được3:1408 < < 3:14285:
Hình 1: Dido cho người cắt miếng da bò để bao quanh vùng đất Carthage [3]
(15)bờ biển Yêu cầu chấp thuận, với điều kiện “vùng đất phải bao phủ miếng da bò” Với tài toán học thiên bẩm, Dido cắt miếng da bị thành mảnh dài dùng để bọc quanh vùng đất hình trịn rộng lớn, sau xứ Carthage
Hình 2: Vùng Carthage đồ năm1535; thấy phần tường thành bên (vùng đất gốc nữ hoàng Dido) gần trịn hồn hảo
Tuy vậy, chứng minh chặt chẽ cho tốn đẳng chu có từ kỷ XIX, phép tính vi tích phân phát triển đầy đủ Trong thời gian này, Steiner đưa kỹ thuật tuyệt vời đối xứng hoá, ngày gọi Steiner symmetrization Ý tưởng ông cho trước hình chưa phải hình trịn, ơng ln tìm hình “đối xứng hơn” với diện tích có chu vi nhỏ Tuy nhiên, Weierstrass nhận vấn đề tính tế, để hồn tất chứng minh cần phải tồn hình với chu vi nhỏ Điều thúc đẩy phát triển mạnh mẽ phương pháp biến phân (calculus of variations) Cho tới năm1884;H A Schwarz đưa chứng minh hoàn chỉnh đầu tiên, cách bổ sung kỹ thuật đối xứng hoá mới, ngày gọi Schwarz symmetrization
(16)bất đẳng thức hoán vị liên quan tới đại lượng vật lý Đây cơng trình mở đường cho nhiều kết thú vị vật lý toán sau
Gần đây, người tiếp tục nghiên cứu dạng mở rộng cho toán đẳng chu Năm 2014;Fusco Julin chứng minh kết sau [4]
Định lý (Bất đẳng thức đẳng chu dạng mạnh) Tồn sốCd > 0sao cho với
mọi tậpRd;ta có
Per./ Per./>Cd inf y2Rd
Z
Rd
.x/ .xCy/
jxj dx:
Vì hàmjxj 1đối xứng giảm, từ bất đẳng thức Hardy-Littlewood ta biết Z
Rd
.x/ .xCy/
jxj dx >0:
Hơn nữa, dấu “=” xảy khilà cầu
Các nghiên cứu toán đẳng chu dẫn tới nhiều bước tiến quan trọng tốn học đại Bạn đọc xem thêm báo tổng quan D Bandle [5] M Ashbaugh R Benguria [6]
Thực bất đẳng thức đẳng chu có mặt nhiều nơi sống hàng ngày Các mèo khơng quan tâm tới tốn học, chúng biết xác làm để giảm thiểu diện tích tiếp xúc với mơi trường ngày lạnh giá
(17)3 Bài toán lượng hấp dẫn nội tại
Theo định luật hấp dẫn tiếng Newton, chất điểm R3 với trọng lượng tạo
ra jxj vị trí x R3 (đạo hàm cho ta lực hấp dẫn jxj 2,
ta cho số hấp dẫn 1) Tổng quát hơn, ta có vật thể R3 với mật độ
f WR3 !Œ0;1/, tạo vị tríx 2R3là
Vf.x/D f jxj 1D Z
R3
f y/
jx yjdy:
Hình 4: Một lát cắt2chiều Newton potentialV vớilà cầu trongR
3[8].
Trong trường hợp vật thể có mật độ đồng nhất, tứcf D hàm đặc trưng tập hợp
R3, tích phân củaV
trênlà lượng hấp dẫn nội của EN./D
Z
Z
1
jx yjdxdy:
Khoảng năm1687;Newton đặt câu hỏi tìm giá trị cực tiểu củaEN./trên tậpcó thể
tích cho trước Từ bất đẳng thức Riesz, ta thu
Định lý (Bài toán lượng hấp dẫn nội tại) Trong số tập hợp R3 có cùng
thể tích, cầu có lượng hấp dẫnEN./nhỏ nhất.
Ngoài ra, ta biết với thể tíchjjcho trước, lượng hấp dẫnEN./ln âm
(18)Định lý (Newton) Nếuf WR3 ! Œ0;1/là hàm đối xứng tâm, tức làf x/D f y/khi
jxj D jyj, thì
Z
R3
f y/
jx yjdy D Z
R3
f y/
max.jxj;jyj/dy; 8x:
Định lý tảng toán học cho lý thuyết lực hấp dẫn Newton Giả sử ta xem trái đất cầu tâm0, bán kínhR, với mật độ đồng > Khi đó, đứng vị trí x 2R3bên ngồi trái đất, bị hút lực hấp dẫn từ trái đất với
Z
B.0;R/
jx yjdy D Z
B.0;R/
max.jxj;jyj/dy D M
jxj;
trong đóM D jB.0; R/jlà trọng lượng trái đất Như vậy, lực hấp dẫn từ trái đất phụ
thuộc vào trọng lượng trái đất khoảng cách từ người tới tâm trái đất
Tương tự, ta giả sử mặt trăng cầu tâmB.x0; r/ở xa trái đất (tức làjx0j > RCr),
với mật độ0 > Vì lực hấp dẫn từ trái đất điểmx 2R3 mặt trăng
Mjxj 1, tổng lượng hấp dẫn tương tác mặt trăng trái đất là Z
B.0;R/ Z
B.x0;r/
jx yjdxdy D Z
B.x0;r/ M0
jxj dx D Z
B.0;r/
M0
jx x0jdx D
Z
B.0;r/
M0
max.jxj;jx0j/dx D
Z
B.0;r/
M0
jx0jdx D
M m
jx0j:
trong đómD0jB.0; r/j DmD0jB.x0; r/jlà trọng lượng mặt trăng Như tổng
lượng hấp dẫn phụ thuộc vào trọng lượng trái đất, trọng lượng mặt trăng, khoảng cách từ tâm trái đất (0) tới tâm mặt trăng (x0)
Quay lại với lượng hấp dẫn nội tại, với hình vành khănD fx 2R3Wr < jxj< Rg, theo
định lý Newton ta có EN./D
Z
Z
1
jx yjdxdy D Z
Z
1
max.jxj;jyj/dxdy
> Z
Z
1
rdxdy D
jj2
r :
Do đó, với thể tíchjjcho trước, lượngEN./có thể gần0tuỳ ý
Định lý Newton đỉnh cao Toán học kỷ XVII, điểm khởi đầu lý thuyết (Potential theory) Ngày nay, chứng minh định lý Newton từ định lý giá trị trung bình (mean-value theorem) hàm điều hồ (harmonic function)
Định lý (Định lý giá trị trung bình) Giả sử hàmglà điều hồ (sub-harmonic), tức làg.x/>0với mọix 2B.x0; r/Rd Khi đó
1
jB.x0; r/j Z
B.x0;r/
(19)Hệ khiglà hàm điều hoà, tức làg.x/D0với mọix 2B.x0; r/Rd, ta có
1
jB.x0; r/j Z
B.x0;r/
g.y/dy Dg.x0/:
Hàm sốjxj là hàm điều hoà trongR3nf0g Vớif D
B.0;R/ vàjxj > R, B.x; R/
R3
nf0gnên theo định lý giá trị trung bình ta có Z
R3
f y/
jx yjdy D Z
B.x;R/
1
jyjdy D
jB.x; R/j jxj D
Z
R3
f y/
max.jxj;jyj/dy:
Đây trường hợp riêng định lý Newton Các bạn tham khảo chứng minh đầy đủ định lý Newton [1], chương9:
Hàm sốjxj 1khơng phải hàm điều hồ tồnR3vì điểm kỳ dị tại0 Thực ta có
jxj D4ı0;
với ı0 hàm Dirac-delta (đây hàm số thông thường mà nên hiểu theo lý
thuyết phân bố) Một cách tương đương, Newton potentialVf thoả mãn phương trình Poison
Vf D4f:
Chú ý lực hút điện tích Coulomb có cơng thức giống lực hấp dẫn Newton Do đó, mặt tốn học, kết potentialjxj 1ở phía sử dụng để nghiên cứu tương
tác điện tích
4 Bài tốn Gramov
Trong lĩnh vực vật lý hạt nhân, mơ hình Gramov lý thuyết xấp xỉ hạt nhân nguyên tử mô tả giọt chất lỏng với mật độ đồng Lý thuyết Gramov đưa năm1928;khi ông mới24tuổi nghiên cứu sau tiến sĩ Copenhagen với Niels Bohr Về mặt tốn học, lý thuyết Gramov nói xem hạt nhân tập hợp trongR3, thìlà lời giải cho toán cực tiểu
EG.m/D inf
jjDm
Per./C
2 Z Z
jx yjdxdy
:
Ở mlà tổng số nucleon (gồm có proton neutron) Đại lượng Per./mô tả sức căng bề mặt giọt chất lỏng, lượng Coulomb mô tả lượng điện tích nội proton
Điều thú vị số tập hợpvới thể tích cho trước jj D m, cầu cực tiểu
(20)Giả thuyết Bài tốnEG.m/có lời giải cực tiểu cầu khi
m6mD5 2 2=3
22=3 1 3:518:
Hơn nữa, khim > mthìEG.m/khơng có cực tiểu.
Cụ thể hơn, giả thuyết khối lượng hạt nhân đủ lớn, hạt nhân bị tách làm hai hạt nhân nhỏ với khối lượng (đây cách để tính khối lượng tới hạnm)
Giả thuyết tương thích với lý thuyết phân rã hạt nhân (phân hạch) tìm năm1938bởi hai nhà hoá học Otto Hahn Fritz Strassmann hai nhà vật lý học Lise Meitner Otto Robert Frisch (Hahn giải Nobel hoá học năm1944cho phát minh này) Trong thí nghiệm mình, Hahn Strassmann bắn hạt neutron vào hạt nhân uranium thu loại hạt nhân (barium) Họ viết thư tới Meitner kể phát Sau Meitner Frisch chứng minh hạt nhân uranium bị phân rã, tạo lượng lớn Hiện tượng phân hạch sở khoa học cho dự án Manhattan bom nguyên tử chiến tranh giới thứ hai
Hình 5: Hiện tượng phân hạch tem Đức năm1979[9]
(21)Định lý 10 Tồn hai số0 < m1< m2 sao cho
Nếum6m1, thìEG.m/có cực tiểu cầu. Nếum > m2, thìEG.m/khơng có cực tiểu.
Cùng thời gian đó, Julin [11] đưa chứng minh khác cho tồn cực tiểu vớim nhỏ, dựa bất đẳng thức đẳng chu dạng mạnh Fusco-Julin, đồng thời Lu Otto đưa chứng minh khác cho không tồn cực tiểu m lớn, dựa đánh giá lý thuyết độ đo hình học (geometric measure theory) Năm2016; với Frank Killip [12], đưa chứng minh ngắn gọn cho không tồn tại, đồng thời đánh giá định lượng m2 Chứng minh này, với chứng minh Julin cho tồn tại, trình
bày lại Bạn đọc hứng thú tham khảo báo tổng quan Choksi, Muratov Topaloglu Notices of the AMS2017[13]
Lời giải Tồn cực tiểu nếumnhỏ:Xét D./WDPer./C
2 Z Z
jx yjdxdy Per
/ Z Z
jx yjdxdy:
Bằng cách tịnh tiếnnếu cần thiết, từ bất đẳng thức đẳng chu dạng mạnh Fusco Julin, ta có
Per./ Per./>Cd Z
R3 f x/
jxj dx;
vớif D :Mặt khác, ta viết
1 Z R3 Z R3
.x/.y/ jx yj dxdy
1 Z R3 Z R3
.x/.y/
jx yj dxdy D Z R3 Z R3
f x/f y/
jx yj dxdy Z
R3
Z
R3
.x/f y/
jx yj dxdy:
Ta biết Newton potential
Vf.x/D Z
R3
f y/
jx yjdy;
thoả mãn phương trình PoisonVf.x/D4f x/vớix2 R3 Do Z
R3
Z
R3
f x/f y/
jx yj dxdy D
1
Z
R3
Vf.x//Vf.x/dxD
1
Z
R3jr
Vf.x/j2dx>0:
Hơn nữa,Vf hàm điều hồ trênvì
Vf.x/D4f x/D4..x/ .x//>0 8x 2:
Do đó, theo định lý giá trị trung bình, ta thu
Z
R3
Z
R3
.x/f y/
jx yj dxdy D Z
Vf.x/dx >jjVf.0/D jj Z
R3 f x/
(22)Vậy tóm lại
D./>Cd jj Z
R3 f x/
jxj dx:
Theo bất đẳng thức Hardy-Littlewood, ta biết rằngR
R3
f x/
jxj dx >0, dấu “=” xảy khi
là cầu Do đó, jj D m < Cd, thìD./ >0và dấu “=” xảy khilà
quả cầu Vậy nếum < Cd thìEG.m/có cực tiểu cầu
Không tồn cực tiểu nếu m > WGiả sử EG.m/có cực tiểu Ta cắt thành hai
phần mặt phẳngH trongR3, tức
DC[ với ˙D\H˙;
trong đóHC,H phần khơng gian nằm bên phải bên trái mặt phẳngH Sau đó, ta di chuyển xaCbằng phép tịnh tiếnR Vìlà tập cực tiểu, lượng khơng vượt q lượng củaC[R Khi lấyR ! 1, lượng củaC[R trở thành tổng lượng củaCvà Do đó, ta có
Per./C Z Z
jx yjdxdy 6Per.
C/CZ
C
Z
C
jx yjdxdy CPer. /C
Z
Z
1
jx yjdxdy:
Bất đẳng thức tương đương với
2H2.\H / >
Z
C
Z
1
jx yjdxdy;
trong đóH2.\H /là độ đo chiều (Hausdorff measure) phần giao\H.
Tiếp theo, ta tham số hoá mặt phẳngH: Lấylà vector đơn vị trongR3, lấy`2Rvà chọn
H;` D fx 2R3 WxD`g;
khi
H;`C D fx2 R3Wx > `g; H;` D fx 2R
Wx < `g: Từ bất đẳng thức
2H2.\H;`/> Z
C
Z
1
jx yjdxdy D Z
Z
.x > ` > y/
jx yj dxdy;
ta lấy tích phân theo` R(tức ta tịnh tiến mặt phẳngH để quét toàn bộR3) Theo định lý Fubini, ta có
Z
RH
2
.\H;`/d `D jj:
Hơn nữa, với hàm đặc trưng.x > ` > y/ta có
Z
(23)trong đóŒt C Dmax.t; 0/ Tóm lại, thu
2jj> Z
Z
Œ.x y/C
jx yj dxdy:
Cuối cùng, lấy trung bình với vector đơn vị 2S2 R3và sử dụng Z
ŒzC
d D
jzj
2
Z =2
cossind D jzj
4 ; vớiz D.x y/, kết luận
2jj> Z
Z
jx yj
4jx yjdxdy D
1 4jj
2
;
tức làjj68 Nói cách khác, nếum > 8, tốnEG.m/khơng có cực tiểu 5 Bài tốn ion hố
Tương tự với tốn Gramov, ta có câu hỏi: “Một hạt nhân mang electron?” Trong chương trình hố học lớp 10; biết ngun tử trung hồ có số electron điện tích hạt nhân Nếu số electron khác với điện tích hạt nhân, thu ion dương ion âm Về mặt nguyên tắc, tất ion dương tồn Tuy nhiên, thực nghiệm có ion âm với điện tích 1hoặc tối đa 2mà Việc chứng minh vấn đề toán thách thức lớn vật lý toán đại, gọi toán ion hoá Để phát biểu xác tốn, cần sử dụng tiên đề vật lý lượng tử Xét nguyên tử bao gồm hạt nhân điện tíchZ > 0cố định gốc toạ độ02 R3vàN electron với điện tích 1chuyển động xung quanh Vì electron hạt lượng tử, chúng mô tả hàm sóng ‰.x1; : : : ; xN/trong xn R3 vị trí electron thứ n Đại
lượngj‰j2cho ta mật độ xác suất electron, Z
R3N j
‰.x1; : : : ; xN/j2dx1dx2 dxN D1;
Hơn nữa, hàm sóng phải thoả mãn tính phản đối xứng
‰.x1; : : : ; xi; : : : ; xj; : : : ; xN/D ‰.x1; : : : ; xj; : : : ; xi; : : : ; xN/; i Ôj:
iu ny m bo nguyờn lý loại trừ Pauli hai electron khơng thể có chung mt v trớ (nu xi Dxj vii Ôj thỡ D0)1 Ở trạng thái ổn định nhất, hàm sóng‰cực tiểu hố hàm
lượng
E.N /Dinf ‰ Z R3N N X
iD1
jrxi‰j
2 N X
iD1
Z
jxijj
‰j2C X 16i <j6N
1
jxi xjjj
‰j2dx1dx2 dxN: 1Chính xác hơn, “vị trí lượng tử” electron bao gồm tọa độ không gian quay nội gọi là
spin Trong thực tế, electron có2trạng thái spin, nên chương trình hóa học lớp10ta biết “orbital”
(24)Trong hàm lượng ta có động electron, tương tác Coulomb electron hạt nhân (lực hút), tương tác Coulomb electron với (lực đẩy)
Trong năm1960;G M Zhislin chứng minh nếuN 6Z, tốnE.N /ln có hàm sóng cực tiểu Hàm sóng gọi trạng thái (ground state) v tho phng trỡnh Schrăodinger
0
@ N X
iD1
xi
N X
iD1
Z
jxij C X
16i <j6N
1
jxi xjj
A‰ DE.N /‰:
Nói cách khác, hạt nhân điện tích Z mang 1; 2; : : : ; Z electron (tức nguyên tử trung hoà ion dương tồn tại) Mặt khác, ta có
Giả thuyết Nếu E.N /có cực tiểu, thì N ZCC với sốC độc lập với Z: (thực nghiệm cho thấy ta chọn sốC D1hoặc2)
Đây toán thứ9trong danh sách15bài toán m v toỏn t Schrăodinger ca Barry Simon [14] Trong danh sách này, toán số4(Ten Martini problem) toán số5(độ đo phổ Almost-Mathieu operator) giải Avila-Jitomirskaya Avila-Krikorian (Avila nhận giải Field năm2014cho đóng góp này)
Giả thuyết ion hố hiển nhiên mặt trực giác, nguyên tử có số electron vượt q nhiều điện tích hạt nhân, electron ngồi (có điện tích âm) bị đẩy phần cịn lại (có tổng điện tích âm) Tuy nhiên, chứng minh điều toán học hồn tồn khơng tầm thường
Năm 1982; Sigal Ruskai chứng minh cách độc lập E.N / có cực tiểu, N 6Nc với số hữu hạnNc DNc.Z/ <1 Năm1984;Lieb đưa chứng minh mới,
ngắn gọn đưa đánh giá định lượngNc 62Z Ý tưởng Lieb nhân hai v phng
trỡnh Schrăodinger vi jxNjN v nghiờn cu tương tác electron thứ N với phần lại
của nguyên tử (ý tưởng đề xuất trước Benguria mơ hình đơn giản hơn) Năm2012;tác giả chứng minh rằngNc 61:22ZC3Z1=3;cải thiện kết Lieb cho
Z >6 Về kết liên quan, bạn đọc tham khảo thêm [15]
Trong khuôn khổ viết này, tác giả trình bày lời giải cho tốn ion hố mơ hình đơn giản hơn, gọi lý thuyết Thomas-Fermi Lý thuyết Llewellyn Thomas Enrico Fermi đưa cách độc lập năm1927;trong thay xét hàm sóng củaN electron, họ tập trung vào hàm mật độ f W R3 ! Œ0;1/vớiR
Rdf x/dx D N Khi lượng electron tính
ETF.N /Dinf f
Z
R3
f5=3.x/dx
Z
R3 Z
jxjf x/dxC
1 Z R3 Z R3
f x/f y/
jx yj dxdy
:
Lý thuyết Thomas-Fermi nghiên cứu Lieb Simon năm1977:Họ quan sát hàm lượng hàm lồi theof, cực tiểu tồn thoả mãn tính đối xứng tâm (f x/Df y/nếujxj D jyj) Hơn nữa, thoả mãn phương trình Thomas-Fermi
5 3f
2=3
.x/D Z
jxj f jxj C D Z
jxj Z
R3
f y/
max.jxj;jyj/dy
C
(25)với số2 1; 0(đây nhân tử Lagrange liên quan tới ràng buộcRR3f x/dx D N) Ở ta sử dụng định lý Newton để viết lại vế phải phương trình Thomas-Fermi Sử dụng tính chất này, Lieb Simon chứng minh
Định lý 11 Bài tốn Thomas-FermiETF.N /có cực tiểu khiN 6Z
Sự tồn cực tiểu khiN 6Z đơn giản, tương tự định lý Zhislin cho phng trỡnh Schrăodinger Chng minh khụng tn cực tiểu khiN > Zthì phức tạp Lieb Simon sử dụng nguyên lý cực đại cách khéo léo vào phương trình Thomas-Fermi Dưới đây, tác giả trình bày chứng minh khác, sơ cấp hơn, sử dụng bất đẳng thức AM-GM !
Lời giải Giả sử tốn Thomas-FermiETF.N /có cực tiểuf Nhân hai vế phương trình
Thomas-Fermi vớijxjkf x/với số mũk>1, ta có
5 jxj
k
f5=3.x/ D Z
jxj Z
R3
f y/
max.jxj;jyj/dy
jxjkf x/; 8x2 R3:
Ở ta bỏ dấu phần dương biểu thức ngoặc Œ:;bởi biểu thức âm tạix, thìf x/D 0do phương trình Thomas-Fermi, đẳng thức phía hai vế bằng0 Vì60, ta có
Z
jxj Z
R3
f y/ max.jxj;jyj/dy
jxjkf x/>0 8x 2R3: Lấy tích phân vớix 2R3ta thu được
Z
Z
R3j
xjk 1f x/dx >
Z
R3
jxjkf x/f y/
max.jxj;jyj/ dxdy: Sử dụng tính đối xứng củaxvày biểu thức bên phải, ta có
Z
Z
R3j
xjk 1f x/dx >
Z
R3
.jxjk C jyjk/f x/f y/ 2max.jxj;jyj/ dxdy: Cách viết đối xứng cho phép sử dụng bất đẳng thức
jxjk C jyjk >1 k
; (có thể dùng bất đẳng thức AM-GM) Điều dẫn tới
Z
Z
R3j
xjk 1f x/dx>1 k
Z
R3
.jxjk 1C jyjk 1/f x/f y/dxdy
D1 k
Z
R3
f y/dy
Z
R3j
xjk 1f x/dx: Điều tương đương với
Z
jxj Z
R3
f y/
max.jxj;jyj/dy: (2)
Vì (2) với mọik >1, ta lấyk! 1và suy raZ >R
(26)Định lý giải toán ion hoá cho lý thuyết Thomas-Fermi, chứng tỏ lý thuyết Thomas-Fermi khơng q xác, khơng dự đốn tồn ion âm thực tế Trong hoá học lượng tử (chẳng hạn tính tốn xây dựng nên bảng tuần hồn hố học ngày nay), người sử dụng mơ hình tương tự lý thuyết Thomas-Fermi có hiệu chỉnh thích hợp Một mụ hỡnh nh vy gi l lý thuyt Thomas-Fermi-Dirac-von Weizsăacker, lượng củaN electron tính
ETFDW.N /Dinf f
nZ
R3
f5=3.x/dxCcW Z
R3jr
p
f x/j2dx
Z
R3 Z
jxjf x/dx C
2
Z
R3
Z
R3
f x/f y/
jx yj dxdy cD Z
R3
f4=3.x/dxo: Các số hạng với hệ sốcD > 0; cW > 0c xut bi Dirac.1928/v von Weizsăacker.1935/:
Năm 1987;P.-L Lions chứng minh tồn ion dương ngun tử trung hồ mơ hình TFDW, sử dụng phương pháp “concentration-compactness” (ông nhận giải Field năm 1994 cho đóng góp phương trình đạo hàm riêng phi tuyến) Năm1993; Le Bris chứng minh số ion âm tồn mơ hình TFDW Mặt khác, vấn đề khơng tồn cực tiểu khiN lớn toán mở nhiều năm, lý kỹ thuật nhân phương trình vớijxjkhơng sử dụng số hạng Dirac âm Năm2017;cùng với Frank Van den Bosch, chứng minh giả thuyết ion hố cho mơ hình TFDW kỹ thuật hồn tồn khác
Định lý 12 Nếu tốn ETFDW.N / có cực tiểu, thì N Z CC với số
C > 0độc lập vớiZ
Chứng minh kết sử dụng ý tưởng từ toán Gramov Cụ thể hơn, lời giải cực tiểu cắt mặt phẳng biến đổi tịnh tiến cách thích hợp Điều cho phép kiểm soát số electron xa hạt nhân Mặt khác, electron gần hạt nhân tuân theo lý thuyết Thomas-Fermi Với electron trung gian, sử dụng phép lặp để cải tiến đánh giá “xa” “gần” cách liên tục (ý tưởng Solovej sử dụng trước cho lý thuyết Hartree-Fock [16]) Bạn đọc quan tâm xem thêm [17]
Để kết thúc, tơi xin giới thiệu tốn liên quan danh sách Simon Ngược lại với ion âm, nguyên tử trung hoà bị số electron, thu ion dương Năng lượng cần thiết để tách electron khỏi nguyên tử trung hồ gọi lượng ion hóa
I.Z/ DE.Z/ E.Z 1/:
Đây đại lượng quan trọng phản ứng hoá học Bài toán thứ 10 Simon liên quan tới công thức tiệm cận củaI.Z/khiZ ! Nói riêng, ta có
Giả thuyết Ta cóC1 6I.Z/ 6C2với hai sốC1; C2 > 0độc lập vớiZ
Năm1990;Seco, Sigal Solovej [18] chứng minh rằngI.Z/6C Z20=21:
(27)Hình 6: Năng lượng ion hoá (ionization energy) [19]
chu kỳ Do đó, mặt tốn học, dự đốn bảng tuần hồn “vơ hạn”, lượng ion hố tuần hồn xung quanh số giá trị cố định Cũng tương tự như giả thuyết ion âm, toán lượng ion hoá kiểm chứng mơ hình xấp xỉ Tuy nhiờn, chng minh iu ny cho phng trỡnh Schrăodinger hồn chỉnh cịn tốn mở
Tài liệu tham khảo
[1] M Loss, E H Lieb,Analysis, American Mathematical Society,1997:
[2] A Burchard,A Short Course on Rearrangement Inequalities, Lecture Notes2009:
https://www.math.toronto.edu/almut/rearrange.pdf
[3] https://commons.wikimedia.org/wiki/File:Dido_purchases_Land_ for_the_Foundation_of_Carthage.jpg
[4] N Fusco, V Julin, A strong form of the Quantitative Isoperimetric inequality, Calc Var. PDEs.50 2014/;pp.925 937:
[5] C Bandle,Dido’s Problem and Its Impact on Modern Mathematics, Notices of the AMS, October2017:
https://www.ams.org/publications/journals/notices/201709/ rnoti-p980.pdf
[6] M Ashbaugh, R Benguria,The problem of Queen Dido.
(28)[7] https://www.reddit.com/r/cats/comments/71vihp/sleeping_in_ circles/
[8] https://commons.wikimedia.org/wiki/File:GravityPotential.jpg
[9] https://commons.wikimedia.org/wiki/File:DBP_1979_1020_Otto_ Hahn_Kernspaltung.jpg
[10] C B Muratov, H Knăupfer,On an isoperimetric problem with a competing nonlocal term II: The general case, Comm Pure Appl Math.67 2014/;pp.1974 1994:
[11] V Julin, Isoperimetric problem with a Coulombic repulsive term, Indiana Univ Math J. 63 2014/; 77 89:
[12] R L Frank, R, Killip, P T Nam, Nonexistence of large nuclei in the liquid drop model, Lett Math Phys.106 2016/; 1033 1036:
[13] R Choksi, C B Muratov, I Topaloglu,An Old Problem Resurfaces Nonlocally: Gamow’s Liquid Drops Inspire Today’s Research and Applications, Notices of the AMS, December 2017:
https://www.ams.org/publications/journals/notices/201711/ rnoti-p1275.pdf
[14] B Simon,Schrăodinger Operators in the Twenty-First Century,2000:
https://mathworld.wolfram.com/SimonsProblems.html
[15] P T Nam, On the number of electrons that a nucleus can bind, Proceedings of the Inter-national Congress on Mathematical Physics2012:
https://arxiv.org/pdf/1209.3642.pdf
[16] J P Solovej, The Ionization Conjecture in Hartree-Fock Theory, Ann of Math. 2/ 158 2003/; 509 576:
[17] R L Frank, P T Nam, H Van den Bosch, The ionization conjecture in Thomas-Fermi-Dirac-von Weizsăacker theory, Comm Pure Appl Math.71 2018/; 577 614:
[18] L A Seco, I M Sigal, J P Solovej, Bound on the ionization energy of large atoms, Comm Math Phys,131 1990/; 307 315:
(29)HUYỀN SỬ ĐỜI HÙNG CÓ PHẢI CỦA DÂN TỘC TA?
Nguyễn Lê Anh
GIỚI THIỆU
"Để hiểu lịch sử Việt Nam cần phải tìm hiểu tình hình địa chất theo thời gian" - Nguyễn Lê Anh.
Vẫn góc nhìn lạ với chuỗi lập luận logic dựa nghiên cứu dày công, từ khảo cứu tài liệu đến thực địa địa phương, tác giả Nguyễn Lê Anh đưa hàng loạt kết nghiên cứu lịch sử, khơng kết đánh đổ nhận định vốn tồn hàng trăm năm
Ở Epsilon số 18 này, trân trọng giới thiệu tới độc giả phần loạt "lịch sử" "Học Tốn để làm gì" với góc nhìn lạ gây "địa chấn":Liệu huyền sử Kinh Dương Vương, Hùng Vương An Dương Vương có phải dân tộc Việt Nam hay không?
Huyền sử Kinh Dương Vương hồ Động Đình
Dã sử chépKinh Dương Vương tên húy Lộc Tục, người hình thành nhà nước sơ khai đầu tiên vào năm Nhâm Tuất 2879 TCN, đặt quốc hiệu Xích Quỷ Lãnh thổ quốc gia thời Kinh Dương Vương rộng lớn, phía bắc tới sơng Dương Tử (cả vùng hồ Động Đình), phía nam tới nước Hồ Tơn (Chiêm Thành), phía đơng Đơng Hải (một phần của Thái Bình Dương), phía tây Ba Thục (Tứ Xuyên, Trung Hoa ngày nay) Kinh Dương Vương truyền cho Lạc Long Quân.
Theo Đại Việt Sử ký Toàn thư Kinh Dương Vương làm vua cai trị từ khoảng năm 2879 TCN trở Địa bàn quốc gia thời vua Kinh Dương Vương rộng lớn, phía bắc tới sơng Dương Tử (cả vùng hồ Động Đình), phía nam tới nước Hồ Tơn (Chiêm Thành), phía đơng Đơng Hải (một phần Thái Bình Dương) phía tây Ba Thục (Tứ Xuyên, Trung Quốc ngày nay).a
aTheo vi.wikipedia.org/wiki/Kinh_Dương_Vương, truy cập ngày 10 tháng năm 2020
Khoảng cách từ Cổ Loa tới hồ Động Đình 500km, tồn núi cao, khơng thể có liên kết
Kinh Dương Vương với cư dân đồng Sông Hồng
Cách khoảng 200 triệu năm, lúc với thời gian hình thành dãy Himalayas với đỉnh
(30)-Á, phía Bắc Việt Nam bị gắn vào với phía Nam Trung Quốc Cú va đập tạo dãy núi Thập Vạn Đại Sơn gồm hàng trăm dãy núi cao nhiều kilomet với bề rộng hàng trăm kilomet phía biên giới chạy dài suốt từ Himalayas đổ biển Đó tường thành chắn hai vùng đất Việt Nam với Trung Quốc Rừng rậm rạp có đường đi, trăn, rắn, rết, thú ăn thịt nhiều vô kể, khiến cho Thập Vạn Đại Sơn vùng qua Các sông bắt nguồn từ núi cao chảy khu vực dốc có nhiều ghềnh thác, thuyền mảng di chuyển xi theo dịng sơng khơng thể quay ngược
Như trước 3000năm, cư dân đồng sơng Hồng khơng thể có liên hệ nguồn gốc
tới Kinh Dương Vương hồ Động Đình!
Tổ tiên người Việt Nam đến từ đâu?
Không thể xuất phát từ tiện ích dùng hệ thống giao thơng ngày mà hình dung bang giao từ 1000 năm trước Công Nguyên Ngày chưa có “diêm” để mồi lửa, chưa có
sắt thép để làm trục xe, để đóng móng cho ngựa Rừng rậm bao trùm khắp nơi Ngay ngày nay, dù đường có ngoằn nghèo Sườn dãy núi nhìn thấy sườn dãy bên kia, vài kilomet, phải vòng hàng trăm kilomet sang Khoảng cách thẳng
500km, có đường, đường núi dài Ngay có đường rừng, vượt
đường hàng nghìn kilomet tự thồ theo lương thực, phân tích phần sau vùng đồng Sơng Hồng đầm lầy, thích hợp cho giao thơng thủy theo kênh rạch chằng chịt, người, xe ngựa vượt qua mà dàn trận đánh cướp đất
Con sông Hồng bắt nguồn từ cao nguyên Tây Tạng, nơi băng tuyết giá lạnh, bị đói rét đe doạ Vì từ trước Công Nguyên phần thượng nguồn sông Hồng lạc hậu Tuy di chuyển vùng nhiệt đới khí hậu nóng ẩm, muỗi ruồi, lạ nước, cư dân ôn đới không sống
Dữ liệu khoan băng ở2cực trái đất cho thấy nhiệt độ trái đất tuần hồn với chu kỳ100 nghìn
năm Khoảng23nghìn năm trước trái đất lạnh giá, băng đóng hai cực nhiều tới mức
nước biển cạn xuống120m so với Loài người cổ đại di cư khỏi châu Phi, men
theo ven biển Khoảng18nghìn năm trước lồi người cổ đại sinh sống khu vực Đông
Nam Á, di lên phía Bắc tới vùng Quảng Đơng - Quảng Tây Trung Quốc ngày Những nghiên cứu gen ngôn ngữ khẳng định lại
Trong khoảng 14nghìn năm tiếp theo, mực nước biển tiếp tục dâng cao120m, giữ nguyên
trong khoảng6000năm vừa qua Khi đồng Bắc Bộ ngày biển Nước biển dâng
cao đẩy người cổ đại vào sát chân núi Trong suốt thời gian6000 năm cư dân bị kẹt vùng
(31)Người Việt vùng sơng Hồng có phải tộc trong Bách Việt?
Cùng thời gian người cổ đại vùng Quảng Đông, Quảng Tây di cư vào sâu đất liền ngược theo sơng Dương Tử tới vùng Hồ Động Đình Đây vùng Ơn Đới, mùa đơng băng giá Từ 3500năm trở trước, cư dân vùng ôn đới thường xun bị đói rét đe
doạ, khơng phát triển Khoảng 3500 năm trước đây, với xuất Thời đại đồ
Đồng - người sản xuất nhiều cải mức tiêu thụ - xuất xã hội Chiếm hữu Nô Lệ Phương thức sản xuất hàng hố, phương tiện giao thơng dựa xe kéo dẫn đến phát triển thương mại, xuất tiền, xuất chữ viết, quyền trung ương pháp quyền Tuy nhiên, sử Trung Quốc ghi nhận, tận năm 200trước Công Nguyên, trước Tần
Thủy Hồng hộ, vùng phía nam dãy núi Ngũ Lĩnh khơng có cơng nghệ luyện kim sắt Công nghệ luyện kim sắt xuất vùng Địa Trung Hải, theo cư dân du mục truyền tới vùng đồng sơng Hồng Hà Có sắt tạo vũ khí, trục xe, đóng móng ngựa Chính sắt tạo sức mạnh cho phần phía Bắc dãy núi Ngũ Lĩnh Như vào thời đầu Công Nguyên, chưa có sắt làm trục xe kéo, để đóng móng cho ngựa, xe ngựa cư dân Quảng Đơng, Quảng Tây chưa thể xa tới vùng đồng sơng Hồng - mà khơng có đường để
Do thiên nhiên ưu đãi mà kinh tế cư dân sông Hồng phát triển mạnh Từ 3500 năm
về trước cư dân phía bắc dãy Thập Vạn Đại Sơn lạc hậu cư dân đồng sơng Hồng, nhiên sau thời điểm ngược lại
Con sông Hồng năm mang biển100triệu phù sa, ứng với50triệum3đất Đồng bằng Sông Hồng hình tứ diện, đỉnh Phú Thọ cao30m so với mực nước biển, mặt đáy tam giác
cân Tam giác cân có đáy bờ biển từ Quảng Ninh tới Thanh Hoá - dài 150km,
(32)thể tích tứ diện Vậy6000năm trước đồng Sơng Hồng vịnh nơng có độ
sâu trung bình10m Người cổ đại bị dạt lên ven vùng núi cao theo vòng cung Hoàng Liên
Sơn kéo lên Phú Thọ theo vịng cung Đơng Triều tới Quảng Ninh Cư dân bị kẹt gọi dân tộc Sông Hồng Theo thời gian đồng sông Hồng bồi lớn dần, tiến biển với tốc độ khoảng35km cho1000năm Đất bồi thành gò thấp, quanh gò nước tự
chảy, thuận lợi cho việc cấy lúa nước Cấu trúc gị đồi có ruộng lúa nước tự chảy bao quanh tạo kiểu giao thông thủy, sơng ngịi chằng chịt, hiệu Cư dân Sơng Hồng “cơ giới thủy” tồn kinh tế từ nhiều nghìn năm trước Cơng Ngun Thậm chí tận trước Pháp sang, thủ Hà Nội, hệ thống giao thơng thủy đóng vai trị quan trọng Với cấu trúc giao thơng thủy việc tiến quân đánh chiếm ngựa theo đường không hiệu
Cư dân gò cao, với mật độ dân số khoảng1ha đất gò vườn cho hộ5người Mật
độ cịn trì năm1960,và phương thức sản xuất nông nghiệp tự cung
tự cấp mà mật độ ổn định từ đầu Công Nguyên Pháp sang Thống kê đồ Google thấy vùng điện tích ruộng lúa nước gấp4lần diện tích gị? Tỷ lệ khơng
thay đổi nhiều vòng200năm qua Như mật độ dân số chung là1người trên1ha đất
đồng bằng, tính cho thời gian trước1800
Vào khoảng đầu Công Nguyên, bờ biển vào khoảng từ Luy Lâu tới Bát Tràng, kéo qua vùng Bình Đà Đó tam giác cân, đáy đường trung bình, diện tích
4 diện tích đồng Sông Hồng ngày Như dân số đồng Sông Hồng thời Hai Bà Trưng (40 - 43 sau Cơng Ngun) khoảng
4 ×1500000, tức khảng400 nghìn Dân vùng núi ước tính nửa, tổng dân số thời Hai Bà Trưng khoảng600nghìn Ở vào khoảng1800thì
dân số vùng đồng Sông Hồng vào khoảng2.5triệu Dân số miền Nam, ước lượng theo
tỷ lệ 14
17, là2triệu Dân số miền Trung khoảng
4 tổng dân số hai vùng đồng (theo tỷ lệ nay) Vậy dân số Việt Nam vào năm 1800 khoảng5.7triệu Sau Pháp sang, công
nghệ xây dựng dùng xi măng áp dụng, kiến thức y tế phương thức sản xuất thay đổi khiến cho tỷ lệ tăng dân số khơng cịn trước
Như vào đầu Công Nguyên dân Sông Hồng cộng đồng đơng đúc, khoảng 600
nghìn người Cho tới tận năm1800,các dòng dân tứ xứ di cư đến chủ yếu theo đường biển,
chỉ ước tính số vài nghìn người, bên Trung Quốc có biến Những dịng người bị cư dân Sơng Hồng đồng hố văn hố
Bản thân tên Việt tài liệu cổ Trung Quốc gọi theo ý họ, cư dân Sông Hồng từ nhiều nghìn năm sinh sống vùng đất khơng liên quan tới Bách Việt họ!
Về huyền sử An Dương Vương - Triệu Đà
Khoảng5000năm trước Địa Trung Hải, người ta biết công nghệ luyện kim đồng
Khoảng3500năm trước đây, công nghệ xuất lúc hạ lưu sơng Hồng Hà khu
vực Đông Nam Á, lưu vực Sông Hồng - nơi có sẵn than, sắt đồng lộ thiên
(33)Thủy Hoàng phái Triệu Đà (sinh năm257 trước Công Nguyên) đưa quân tiến đánh sang phần
khu vực sông Trường Giang, vùng họ gọi Bách Việt Khơng rõ Triệu Đà có đến lưu vực sơng Hồng hay khơng, sử nói việc dùng quà cáp lấy lòng ngoại giao với tù trưởng vùng xa để họ không quậy phá Việc cai trị khơng có
Trên thực tế vào thời gian trước Cơng Ngun chưa thể có chiến xuyên qua dãy Thập Vạn Đại Sơn: với bên Cổ Loa, bên Tây Quảng Tây Trung Quốc! Hai địa điểm hai nới khác hẳn nhau, núi cao nhiều nghìn mét vượt qua, mà Cổ Loa với Tây Quảng Tây cách nghìn kilomet đường núi quanh co Sau đánh chiếm Nam Việt, chắc Triệu Đà nhiều năm để bình định Chẳng hiểu thời cổ đại có mống quân mà dàn trận đánh chiều dài với Châu Âu vậy!
Vậy khơng thể có dã sử kiểu “Vào thời kỳ Hồng Bàng4000năm trước đây, phía Tây tỉnh
Quảng Tây (Trung Quốc) có tộc Âu Việt sống xen kẽ với người Lạc Việt Nhà nước Văn Lang Hùng Vương đứng đầu cai trị Lạc Việt Thục Phán, người Âu Việt, vua người Âu Việt đánh bại Hùng Vương vua nước Văn Lang, đổi quốc hiệu thành Âu Lạc, tự xưng An Dương Vương, đóng Phong Khê, vùng Cổ Loa, huyện Đơng Anh”.
Cũng có Lạc Việt, Âu Việt, Phong Khê bên Quảng Tây Xét phương diện giao thông, liên kết Âu Việt phía bên dãy Thập Vạn Đại Sơn với đồng sơng Hồng phía bên khơng thể có
Cổ Loa thuộc huyện Đơng Anh có, 3500 năm trước, gò đất cao16m so với
mực nước biển có cư dân thuộc lạc Sơng Hồng đến sống Các truyện xưa tích cổ Kinh Dương Vương, Lạc Long Quân, vua Hùng, Triệu Đà lấy từ vùng núi Ngũ Lĩnh bị đem dạy cho hậu duệ cư dân Sông Hồng
Cư dân Sông Hồng chưa biết đến “Nước ta trước Văn Lang, sau đổi thành Âu Lạc!?” Tất “kiến thức” quyền trung ương Trung Quốc bỏ vào mồm tầng lớp “tinh hoa” - tức biết đọc biết viết chữ Hán Họ học thuộc lòng dạy lại cho hậu duệ cư dân Sông Hồng Con cháu Bà Trưng, Bà Triệu nhận hậu duệ4000năm tận nơi,
đường núi ngoằn nghèo, cách xa Cổ Loa tới nghìn kilomet
Lịch sử Việt Nam đâu?
Vào đầu Công Nguyên, Hai Bà Trưng khởi nghĩa xưng vương mà Hán Quang Vũ Đế cử Mã Viện đưa20nghìn quân đánh phá Hậu Hán Thư, sách cổ viết rõ thời kỳ Hai
Bà Trưng, viết vào kỷ thứ V Tài liệu nói rõ, quân Mã Viện tiến đánh theo đường biển “Tháng1 âm lịch năm42, Mã Viện đem quân men theo đường biển, san núi làm
đường nghìn dặm, đánh với Trưng Nữ Vương Lãng Bạc Mã Viện thắng, tiến lên chiếm Mê Linh lại đánh bại Trưng Vương Cấm Khê” Hậu Hán Thư cho biết quân Mã Viện bị chết nửa
(34)Hậu Hán Thư sách viết kiện đầu Công Nguyên, lại người viết, Phạm Bằng, viết vào kỷ thứ V Quyển Hậu Hán Thư bị quyền Trung Quốc kiểm duyệt cơng bố nhiều năm sau Hậu Hán Thư cho nguyên nhân khởi nghĩa Hai Bà Trưng sưu cao thuế nặng Tức thời nhà Hán áp đặt ách đô hộ, Việt Nam thuộc địa Có hay khơng khởi nghĩa, ngun nhân thật khởi nghĩa gì? Việc khẳng định nguyên nhân sưu cao thuế nặng không hợp lý Việt Nam nơng nghiệp tự cung tự cấp, chưa có kinh tế hàng hố, tức dùng tiền lưu thơng để có tiền đóng thuế Nếu khơng có chuyện thu tơ ép thuế điều chứng tỏ ảnh hưởng thái thú Tô Định nhà Hán Sông Hồng thuộc dạng quan sát viên, khơng phải quyền hộ Điều hợp lý chưa Triệu Đà đặt chân tới đồng Sông Hồng
Năm40sau Công Nguyên Hai Bà Trưng xưng Vương Lịch sử dân tộc Việt Nam
Như dã sử nói vai trị Triệu Đà vị Vua việc hình thành dân tộc Việt Nam, hoang sử, dã tâm sử
Ngồi thương gia nhóm nhỏ người phương Bắc lưu lạc tới vùng đất Sông Hồng sinh sống, đồn qn Mã Viện người phương Bắc lần thức tới áp đặt chế độ đô hộ
Ngày vùng đồng sông Hồng, vùng Mê Linh - Cổ Loa có hàng nghìn đền thờ Hai Bà Trưng
Truyện cổ tích Cơ bé lọ lem Tấm Cám có cốt truyện Lịch sử khơng hoàn toàn át dã sử, dã sử thường hay mượn tích Cũng có kiện liên quan đến Sơn Tinh, Thủy Tinh - chấm dứt chế độ quần cơng xã ngun thủy, có kiện liên qua đến “bọc100 trứng” để diễn tả chia li Tuy nhiên việc dân tộc chìm đắm u mê, để
(35)CHẾ NGỰ PHOTON BẰNG TOPO
Olivier Bleu, Dmitry Solnyshkov, Guillaume Malpuech
Người dịch - Cao Chi, Hà Nội
Một laser tạo nên chùm ánh sáng mang thông tin sợi quang quãng đường nhiều ngàn km.
Như người ta thay electron bán dẫn photon thiết bị photonic làm việc nhanh gấp bội.
Để thực siêu máy tính photonique tích hợp (photonique intégré) người ta phải vượt qua nhiều trở ngại.
Trở ngại quan trọng khả chạy hai chiều cáp quang (reciproque) ánh sáng gặp vật cản Song nhờ topo tốn học người ta vượt qua trở ngại [1].
Khi hạt lan truyền khơng gian hướng tương đương Nếu photon lan truyền sợi quang (fibre optique) theo chiều gặp phải vật thể ánh sáng truyền ngược lại Điều xảy vắng mặt chiều ưu tiên lan truyền Một vật thể thay đổi chiết suất quang học làm cho ánh sáng bị phản chiếu ngược lại, làm nhiễu loạn vận hành
Muốn có lan truyền khơng đảo ngược cần phải có lực áp đặt chiều chuyển động Đối với electron cần áp đặt điện trường đủ điều hành chiều chuyển động Song photon khơng photon hạt khơng mang điện tích
Thách thức nhà vật lý tìm chuyển động khơng có chiều ngược lại photon Piotr Kapitsa (năm 1937) tìm siêu chảy (mất độ nhớt nhiệt độ thấp): Năng lượng tương tác hạt lớn động chất lỏng Chất lỏng nghiêm cấm trình phân tán trật tự môi trường
Trong chất lỏng cổ điển có vật thể gây nên trật tự (désordre) vật thể làm phát tán lượng chuyển động hạt nguyên nhân độ nhớt Những nhiễu loạn vắng mặt siêu chảy độ nhớt không, phân tử chất siêu chảy chuyển động theo chiều Đây ý tưởng quan trọng cho nhà vật lý
1 Thách thức vận chuyển không đảo ngược
(36)2 Lý thuyết giải (bande) lượng topo
Hình 1: Các giải lượng, khe, mức Fermi - dẫn điện, cách điện bán dẫn
Hình 2: Cách điện topo cách điện topo photonique
Trong mạng tinh thể lượng trải rộng giải liên tục đến giải (bande), giải cách khoảng trống gọi khe (gap), tức giải cấm
Ta lại có mức Fermi mức lượng cao chiếm electron không tuyệt đối ( 273ıC hay 0Kelvin) Nếu mức Fermi thuộc giải cho phép (hình 1) ta có vật liệu
là vật liệu dẫn điện khơng có chiều chuyển động ưu tiên: Các electronsex chuyển động bị áp đặt điện trường
(37)Trong năm1980các nhà vật lý thấy lý thuyết giải lượng chưa đủ mơ tả
tích chất electronique vật liệu Họ đưa thêm thông số số Chern, số bất biến topo Nếu số bằng0ta có vật liệu thơng thường theo lý thuyết giải, khác0(hình 2) ta có
vật liệu gọi cách điện topo
Như mức Fermi nằm khe sơ Chern khác số0thì vật liệu lại vật
liệu cách điện thể tích
Nhưng biên vật liệu trạng thái lượng tồn khe ta có dịng điện chiều chiều phụ thuộc vào dấu tổng số Chern giải nằm khe Trong vật liệu photonique (vật liệu dùng photon khơng dùng electron), photon boson nên khơng có mức Fermi ta lại có mạng photonique khả có cách điện thể tích Và số Chern khác số0thì điều dẫn đến tồn chuyển động
một chiều biên với lượng khe
3 Từ Topo đến số Chern
Với chất rắn3D bất biến topo đặc trưng số lỗ mặt, hay đường cong số
cuốn vòng (bao nhiêu vòng đường cong quanh vật) cần tính tích phân góc cực (polaire) điểm dọc theo đường cong (hình 3)
Hình 3: Số lỗ mặt số vịng quanh vật
Nói chung bất biến tích phân đại lượng định xứ (độ cong, góc cực - angle polaire) Các tích phân đặc trung tính chất tồn cục (global) hệ
Các tích phân Gauss-Bonnet topo nối liền tính chất định xứ vật với tính chất tồn cục
Ví dụ tích phân độ cong định xứ mặt cầu†của vật nối liền với số lỗcủa vật.
Khi ta làm biến dạng bề mặt vật số lỗ không thay đổi
2 Z
(38)Trong vật lý môi trường đông đặc công cụ topo không ứng dụng thân vật mà cấu trúc giải (bandes) lượng hạt chuyển động vật liệu Trước hết ta cần định nghĩa độ cong Berry Để hiều khái niệm ta xét mối tương tự với phép vận chuyển song song vector bề mặt cong Sau thực vịng kín vector ban đầu cuối có hướng không trùng Sự vận chuyển song song dẫn đến góc hai vector đầu cuối
Cũng theo cách người ta gán vector với hàm sóng mơ tả electron giải (bande) Người ta gắn liền độ cong Berry với độ biến đổi hướng vector electron chuyển động không gian nghịch đảo (espace réciproque) xung lượng Lấy tích phân độ cong tồn giải (bande) ta sử dụng định lý Gauss-Bonnet thu bất biến topo đến số Chern, số đặc trưng cho topo giải
Topo xếp hạng vật theo số lỗ theo số quanh (enroulement) - bất biến topo vật bị biến dạng liên tục
4 Topo toán học viện trợ cho môi trường đông đặc
Ý tưởng siêu chảy giúp nhà vật lý tiến đến dùng topo môi trường đông đặc Trong topo độ dài góc khơng cịn bảo tồn, bảo tồn số lỗ di qua mặt, dùng biến đổi liên tục ta khơng thể xóa bỏ lỗ
Độ nhớt đại lượng toàn cục (global) khơng phải địa phương Người ta xác định đại lượng bất biến topo topo dùng để nghiên cứu vật chất
Các vortex (những cuộn xốy vật chất) lịng siêu chảy (Onsager, Nauy1947) kích
thích có momen động lượng tử hóa nghĩa lấy giá trị gián đoạn, topo cuộn xoáy ứng với số lần mà đường cong chạy quanh vật
Nếu xét hàm sóng siêu chảy hàm sóng tiến đến tâm vortex Một vortex
đã tạo nên khơng biến đổi liên tục xóa Người ta nói khuyết tật topo Người ta sử dụng topo để mơ tả hiệu siêu chảy siêu dẫn Trong thập kỷ topo trở thành quan để nghiên cứu “cách điện topo-isolant topologique”, vật liệu cách điện lịng thể tích song dẫn điên biên. Ngồi cách điện topo electron chuyển động theo chiều
Trong chất ta có vortex loại đặc biệt Trong cách điện topo khơng cịn cuộn xốy gắn liền với vật chất mà gắn liền với hàm sóng hạt khơng gian nghích đảo (espace réciproque – từ chuyên môn chất rắn) xung lượng Đây không gian trừu tượng để mô tả sóng khơng gian thơng thường Trong khơng gian nghịch đảo hàm sóng khơng cịn hàm khơng thời gian mà hàm lượng xung lượng với vector sóng mơ tả chiều lan truyền sóng tỷ lệ nghịch với chiều dài sóng
(39)Nhà vật lý người Đức Klaus von Klitzing tìm cách điện topo nghiên cứu hệ Hall lượng tử (Nobel1985)
Xét vật liệu bán dẫn hai chiều có tác động từ trường thẳng góc gây nên quỹ đạo vịng cho electron Phía electron có quỹ đạo vịng khơng đóng góp vào dịng điện phía cách điện Ngược lại electron biên lại cho dòng điện theo chiều phụ thuộc chiều từ trường, ta có dẫn điện biên (hình 4)
Hình 4: Cách điện topo, biên quỹ đạo electron nối tiếp làm thành dịng điện Độ dẫn biên bị lượng tử hóa tỷ lệ với bất biến topo Các tính chất đặc trưng toàn cục (glogal): Một khuyết tật khơng cản trở dịng chảy
Một vật liệu dẫn điện, cách điện hay cách điện topo Muốn hiểu vấn đề phải nói đến lý thuyết giải lượng (theorie des bandes) xem phần
Trong nguyên tử riêng lẻ, electron có giá trị lượng gián đoạn, vật liệu có mạng tinh thể sóng hạt giao thoa với làm nên vùng lượng liên tục (bandes) phân cách vùng trống gọi khe (gap) Tùy theo phân bố electron giải mà có dẫn điện, cách điện hay bán dẫn
Hiện người ta thấy lý thuyết giải lượng chưa đủ để mô tả vật liệu vật liệu cách điện topo Người ta thấy cấu trúc hàm sóng electron đóng vai trị quan trọng Người ta xét đến gọi độ cong Berry mô tả electron giải (Michael Berry 1984) để mô tả tượng lượng tử Khi người ta tích phân độ cong
Berry theo tập thể electron giải toàn vẹn người ta thu đại lượng, số Chern (Mỹ gốc Trung quốc1950) Đây số bất biến topo có giá trị nguyên Số
(40)Một khe (gap) gọi topo tổng số Chern băng nằm khe khác 0:
Trong trường hợp có mức lượng cho phép khe Những electron chiếm trạng thái chuyển động vật liệu theo chiều khơng có chiều ngược Những trạng thái gọi bảo vệ topo (protégé topologique) tính chất tồn cục (global) dịng chảy khơng bị nhiễu loạn khuyết tật điểm bẩn (impurité) mạng tinh thể Đó ý tưởng siêu chảy nhiễu loạn gây nên độ nhớt bị loại bỏ
Như muốn có mơ tả tồn diện độ dẫn vật liệu người ta phải nghiên cứu không giải điện tử, cách lấp đầy mà phải biết số Chern Điều dẫn đến nhiều phát triển lý thuyết thực nghiệm với mục đích tìm cách điện topo
Năm2005Charles Kane Eugene Mele (Mỹ) tìm loại cách điện topo không biểu
bằng từ trường mà biểu nhờ tương tác spin - quỹ đạo nối liền chuyển động electron với spin hướng thẳng lên phía chúc xuống phía Như ta không cần đến từ trường mà phải xét đến đặc tính nội vật liệu
5 Cịn photon ?
Có loại cách điện topo không cần đến từ trường đề có chuyển động chiều Vậy dùng photon thay electron sử dụng mạng tinh thể photonique Ở ta có giải cho phép giải cấm, photon chuyển động giải cho phép mà không phép vào khe Và lý thuyết tương tự lý thuyết mạng electronic
Ở ta khơng có ngun lý loại trừ Pauli photon có spin Ta nói cách điện photonic nói đến tồn khe nghiêm cấm photon
Nói chung người ta tạo nên photonique tích hợp giải tốn chuyển động chiều khơng đảo ngược
6 Kết luận
Hiện người ta kết hợp khái niệm siêu chảy topo giải lượng để tạo nên cách điện topo, tương tự hệ ứng Hall lượng tử spin Vai trò spin bảo đảm số vòng quanh (enroulement) cuộn xoáy (vortex) siêu chảy ánh sáng
(41)Tài liệu tham khảo
[1] Dompter les photons grâce la topologie, Olivier Bleu, Dmitry Solnyshkov, Guillaume Malpuech, Pour la science số tháng9=2019:
(42)GĨC NHÌN MỚI CHO MỘT BÀI TỐN CŨ
Kevin Hartnett
Người dịch Trần Nguyễn Nam Trung
Hiệu đính Trần Nam Dũng, Trần Nguyễn Nam Hưng
GIỚI THIỆU
Trong thời gian phải tự cách ly Covid-19;Joshua Greene Andrew Lobb tìm cách chứng minh phiên “bài toán chốt hình chữ nhật”
Vào tháng3;các nhà tốn học Joshua Greene Andrew Lobb hoàn cảnh giống
nhau: Cách ly xã hội, phải đấu tranh để tự điều chỉnh thân đại dịch COVID-19hoành
hành Họ định vượt qua cách đắm chìm vào nghiên cứu
“Tơi nghĩ đại dịch thực loại liều thuốc kích thích” Greene, giáo sư Đại học Boston cho biết “Tất định cách tốt dựa vào số hợp tác để hỗ trợ lẫn nhau”.
Một vấn đề mà hai ông quan tâm phiên toán kỷ chưa giải hình học
“Bài tốn phát biểu đơn giản dễ hiểu, thực khó” Elizabeth Denne từ Đại học Lee Washington nói
Nó bắt đầu với đường cong khép kín - loại đường cong kết thúc nơi bắt đầu Về bản, vấn đề Greene Lobb theo đuổi dựa dự đoán, bản, phát biểu đường cong chứa bốn điểm tạo thành đỉnh hình chữ nhật với tỷ lệ mong muốn
Bài tốn “chốt hình chữ nhật” nhìn giống loại câu hỏi mà học sinh trung học giải với compa thước kẻ, cản đường nỗ lực cao nhà toán học nhiều thập kỷ Khi Greene Lobb định theo đuổi vấn đề này, họ khơng có sở đặc biệt để tin họ đạt kết tốt
Trong tất dự án khác mà ông thực hiện, Greene cho biết, “Tôi nghĩ dự án ít hứa hẹn nhất”.
Nhưng đại dịch lan rộng, Greene Lobb, công tác Đại học Durham Anh Viện Khoa học Công nghệ Okinawa, trao đổi với hàng tuần qua Zoom có bước tiến nhanh chóng Sau đó, vào ngày19tháng5; phần giới bắt đầu mở cửa
(43)Chứng minh cuối họ hình chữ nhật dự đốn thật có tồn -đưa tốn sang bối cảnh hình học hồn tồn Ở đó, câu hỏi đặt trình bày dễ dàng
“Nó thật kì lạ”, Richard Schwartz từ Đại học Brown cho biết, “Đó ý tưởng cho vấn đề này”.
Hướng suy nghĩ hình chữ nhật
Bài tốn chốt hình chữ nhật câu hỏi gần giống với câu hỏi nhà toán học người Đức Otto Toeplitz đặt vào năm1911:Ơng dự đốn đường cong kín chứa bốn
điểm bốn đỉnh hình vng “Bài tốn chốt hình vng” ơng chưa có lời giải
“Đó vấn đề cũ gai góc mà khơng giải được” Greene cho biết
Để hiểu vấn đề lại khó đến vậy, điều quan trọng phải nắm loại đường cong mà tốn chốt hình vng đặt ra, đóng vai trị quan trọng chứng minh Greene Lobb
Hai nhà toán học giải vấn đề đường cong khép kín, “liên tục” “trơn” “Liên tục” có nghĩa chúng khơng bị đứt qng, “trơn” có nghĩa chúng liên tục không bị gãy khúc (khơng tạo thành góc) Những đường cong trơn liên tục thứ mà bạn vẽ với bút chì giấy Chúng thứ “dễ dàng để ta với tay tới”, Greene cho biết Các đường cong trơn, liên tục tương phản với đường cong đơn liên tục, không trơn - loại đường cong đặc trưng đoán chốt vng Toeplitz Loại đường cong có góc nơi mà chúng xoay chuyển đột ngột theo hướng khác Một ví dụ điển hình đường cong có nhiều góc hình fractal bơng tuyết Koch, thứ mà tạo thành từ góc Bơng tuyết Koch đường cong khác tương tự, khơng thể phân tích phép tính phương pháp liên quan, điều khiến chúng đặc biệt khó nghiên cứu
“Một số đường cong liên tục [không trơn] thực khó chịu”, Denne nói
Nhưng lần nữa, vấn đề Greene Lobb giải liên quan đến đường cong trơn, liên tục Và thay xác định liệu đường cong ln có bốn điểm tạo thành hình vng hay không - câu hỏi giải cho đường cong trơn, liên tục vào năm1929- họ nghiên cứu xem liệu đường cong ln có bốn điểm tạo thành hình
chữ nhật tất “tỷ lệ khung hình” (có nghĩa tỉ lệ cạnh) Đối với hình vng, tỷ lệ khung hình là1W1;trong nhiều TV có độ phân giải cao, tỉ lệ là16W9:
Những bước tiến đáng kể toán chốt hình chữ nhật trình bày chứng minh từ cuối năm1970 Herbert Vaughan Chứng minh mở cách suy
(44)“Tất người biết chứng minh này”, Greene cho biết “Nó thứ giống văn hóa dân gian thứ mà bạn tiếp nhận thảo luận buổi ăn trưa phịng ăn tập thể”
Thay hình dung hình chữ nhật dạng bốn điểm kết nối, Vaughan nghĩ hai cặp điểm có mối quan hệ đặc biệt với
Hình ảnh hình chữ nhật có đỉnh kí hiệu làABCD;theo chiều kim đồng hồ từ
cùng bên trái Trong hình chữ nhật này, khoảng cách hai điểm A C (dọc theo đường
chéo hình chữ nhật) với khoảng cách hai điểmB vàD(dọc theo đường chéo
(45)Vì vậy, bạn tìm kiếm hình chữ nhật vịng khép kín, có cách để tìm chúng tìm cặp điểm có chung tính chất này: Chúng tạo thành đoạn thẳng có độ dài với trung điểm Và để tìm chúng, điều quan trọng phải tìm cách suy nghĩ có hệ thống
Để hiểu điều có nghĩa gì, thứ đơn giản Chúng ta chọn hai điểm trục số, giả sử số7và8- biểu diễn chúng thành điểm
trong mặt phẳng Oxy 7; 8/: Các cặp hai điểm trùng nhau, ví dụ.7; 7/cũng chấp nhận
(46)khác để phát biểu điều nói mặt phẳng Oxy “tham số hóa”, lấy theo thứ tự tất cặp điểm trục số
(47)và điểm tọa độy- ta không thu mặt phẳng Oxy Thay vào đó, ta thu
hình dng ỏng ngc nhiờn: Mt di Măobius, mt mt hai chiều có mặt
Theo cách điều lại có ý nghĩa Để hiểu sao, ta chọn cặp điểm đường cong đặt tên chúng làxvà y:Bây ta từxđếny dọc theo cung đường
cong từyđếnxdọc theo cung lại đường cong Bằng cách làm vậy, ta
đã di chuyển qua tất cặp điểm đường cong, bắt đầu kết thúc với cặp khơng có thứ tự.x; y/:Nhưng làm vậy, ta trở lại nơi bắt đầu, theo hướng lật ngược lại Vòng lặp định
hướng lật điểm khơng có thứ tự tạo thnh lừi ca di Măobius
Di Măobius ny cung cấp cho nhà toán học đối tượng để phân tích nhằm giải vấn đề chốt hình chữ nhật Vaughan sử dụng để chứng minh đường cong chứa bốn điểm tạo thành hình chữ nhật
Những câu trả lời bốn chiều
Chứng minh Greene Lobb xây dựng dựa cơng trình Vaughan Nó kết hợp số kết bổ sung, vài số tìm gần Chứng minh cuối công cụ xác, có kết hợp đắn ý tưởng để tạo kết mà họ mong muốn
Một phần quan trọng chứng minh họ xuất vào tháng11năm 2019khi sinh viên tốt nghiệp Princeton Cole Hugelmeyer công bố báo giới thiệu
cách phõn tớch di Măobius ca Vaughan Cụng vic ny liên quan đến quy trình tốn học gọi nhúng, bạn lấy vật thể ghép vào khơng gian hình học Greene Lobb cuối sử dụng kỹ thuật Hugelmeyer chuyển vào khơng gian hình học khác Nhưng để xem họ (Greene Lobb) làm trước tiên bạn cần biết anh sinh viên (Cole Hugelmeyer) thực
Dưới ví dụ đơn giản việc nhúng
Chúng ta bắt đầu với đường thẳng chiều Mỗi điểm đường thẳng xác định số Bây giờ, “nhúng” đường thẳng vào khơng gian hai chiều - có nghĩa là, bạn minh họa đường thẳng mặt phẳng
Khi nhúng đường thẳng vào mặt phẳng Oxy, vị trí điểm xác định xác hai số - gồm hoành độ x tung độ y:Với cách làm này, ta bắt đầu phân tích
đường thẳng cách sử dụng kỹ thuật hình học hai chiều
Ý tưởng Hugelmeyer làm tương tự cho di Măobius, nhng bng cỏch nhỳng nú vo khụng gian bốn chiều, nơi sử dụng đặc trưng hình học bốn chiều để chứng minh kết mà mong muốn hình chữ nhật
(48)có thể nghĩ đến việc anh gán điểm đường cong cho tiểu bang, thành phố, tên đường số nhà
Để làm điều này, bt u vi mt im cho trc trờn di Măobius tập trung vào hai điểm đường cong khép kín ban đầu mà biểu diễn Sau đó, tìm trung điểm cặp điểm xác định tọa độ xvà y Đó hai giá trị “địa bốn chiều”
(tiểu bang thành phố)
Tiếp theo, đo khoảng cách (theo đường thẳng) hai điểm ban đầu đường cong Độ dài trở thành giá trị thứ ba “địa bốn chiều” (tên đường) Cuối cùng, tính góc tạo thành đường thẳng qua hai điểm ban đầu trụcx:Góc trở thành giá trị thứ tư
trong “địa bốn chiều” (số nhà) Bốn giá trị cho ta biết tất thứ cặp điểm đường cong cách hiệu
Các bước phức tạp, Hugelmeyer thu kết Anh ta ly di Măobius ó c nhỳng v xoay nó, theo cách ta tưởng tượng giữ khối trước mặt vặn chút sang trỏi Di Măobius mi sau xoay cú v trớ bự vi di Măobius gc, ú hai di cắt (Vì phép quay diễn khơng gian bốn chiều, nên hình ảnh xác hai di Măobius rt khú hỡnh dung, nhng nú d dng để tiếp cận mặt toán học)
Giao điểm có vai trị quan trọng Bất nơi no hai di Măobius chng lờn nhau, ta s tỡm thấy hai cặp điểm tương ứng đường cong khép kín ban đầu tạo thành bốn đỉnh hình chữ nhật
(49)Đầu tiên, nhớ hình chữ nhật hiểu hai cặp điểm có chung điểm khoảng cách hai điểm cặp Đây xác thơng tin mã hóa ba giá trị “địa bốn chiều” c gỏn cho tng im trờn di Măobius c nhỳng
Th hai, ta cú th xoay di Măobius không gian bốn chiều để thay đổi bốn tọa độ “địa bốn chiều” điểm - thay đổi số nhà tất nhà khu vực, để lại tên đường, tên thành phố tên tiểu bang (Xét ví dụ mang tính hình học hơn, tưởng tượng cách giữ khối trước mặt bạn dịch chuyển sang bên phải, tức thay đổi hồnh độxcủa nó, cịn tung độyvà cao độz giữ nguyên)
Hugelmeyer giải thích cách xoay dải Măobius khụng gian bn chiu cho hai ta độ trung điểm cặp điểm tọa độ khoảng cách chúng giữ nguyên Phép xoay thay đổi tọa độ cuối - góc đoạn thẳng cặp điểm
Kết là, giao im ca di Măobius sau c xoay v di ban đầu tương ứng với hai cặp điểm phân biệt đường cong kín có trung điểm cách khoảng Ta kết luận, giao điểm tương ứng với bốn đỉnh hình chữ nhật đường cong Kĩ thuật sử dụng giao điểm hai khơng gian để tìm điểm mà ta mong muốn, từ lâu sử dụng việc nghiên cứu vấn đề chốt hình vng hình chữ nhật
“Nơi [khơng gian] giao nơi bạn có thứ mà bạn tìm kiếm”, Denne cho biết “Rất nhiều chứng minh lịch sử vấn đề chốt hình vng dựa ý tưởng đó” Hugelmeyer sử dụng kĩ thuật giao không gian bốn chiều thu nhiều kết trước Dải Măobius cú th c xoay theo bt k gúc no khoảng từ
0đến360độ, chứng minh phần ba số phép quay tạo
giao điểm gốc xoay Điều hóa lại tương đương với việc nói đường cong kín, ta tìm thấy hình chữ nhật ứng với phần ba số tỷ lệ khung hình
“Nhờ Cole mà chúng tơi nhận rằng, nên suy nghĩ việc đặt dải Măobius khụng gian bn chiu v s dng cỏc kỹ thuật bốn chiều tương ứng”, Greene nói.
Đồng thời, kết Hugelmeyer đáng ý: Nếu không gian bốn chiều cách hiệu để cơng tốn, hữu ích cho phần ba hình chữ nhật có thể? “Bạn giải hai phần ba cịn lại, chúa”, Greene nói “Nhưng cách nào?”
Giữ cho chúng đối xứng Symplectic
(50)“Chúng tơi khơng ngừng nói vấn đề này”, Lobb nói “Chúng tơi đến nhà hàng, quán cà phê, bảo tàng suy nghĩ vấn đề này”.
Họ tiếp tục trò chuyện sau bị cách ly nhà Họ hi vọng chứng minh vịng quay có th ca di Măobius u mang li mt im giao - tương đương với việc chứng minh ta tìm thấy hình chữ nhật với tất tỷ lệ khung hình
Vào tháng4;họ phát triển chiến lược Nó liên quan đến việc nhỳng di Măobius vo
trong mt phiờn bn c biệt không gian bốn chiều Bằng cách nhúng thông thường, bạn đặt đối tượng nhúng theo cách bạn muốn Hãy suy nghĩ việc nhúng vịng khép kín chiều mặt phẳng hai chiều Số cách bạn làm vơ hạn, tương tự số cách bạn xếp vòng lặp chuỗi bảng
Nhưng giả sử rằng, bề mặt hai chiều mà bạn muốn nhúng vịng lặp vào có cấu trúc định Hãy nghĩ, ví dụ, đồ xếp lớp với mũi tên (được gọi vectơ) cho biết gió thổi vào điểm Trái Đất theo hướng với tốc độ Bây bạn có bề mặt hai chiều với thông tin cấu trúc bổ sung điểm Sau đó, bạn đặt quy tắc vịng khép kín chiều cần nhúng vào đồ để ln tn theo hướng mũi tên mà nhúng vào
“Khó khăn bạn bạn phải cố gắng đặt đường cong theo vectơ đó”, ơng Schwartz nói Có cách để đặt vịng lặp chuỗi
Các loại khơng gian hình học khác mở hướng suy nghĩ cách khác để đặt giới hạn Một điều quan trọng nghiên cứu Greene Lobb khơng gian đối xứng
Kiểu thiết lập hình học lần xuất vào kỷ XIX nghiên cứu hệ thống vật lý, ví dụ quỹ đạo quay hành tinh Khi hành tinh chuyển động không gian ba chiều, vị trí xác định ba tọa độ Nhưng nhà toán học người Ireland William Rowan Hamilton quan sát thấy điểm chuyển động hành tinh, người ta đặt vectơ biểu diễn động lượng hành tinh
Vào năm1980;nhà toán học Vladimir Arnold xây dựng nghiên cứu hình học
đối xứng Ơng thấy khơng gian hình học với cấu trúc đối xứng tự giao xoay thường xuyên so với khơng gian khơng có cấu trúc
Điều hoàn hảo cho Greene Lobb, người muốn giải vấn đề chốt hình chữ nhật cho tất tỷ lệ khung hình cách chứng minh xoay di Măobius c tham s húa cng t giao rt nhiu Vỡ vy, h c gng nhỳng di Măobius hai chiều vào không gian đối xứng bốn chiều
“Cái nhìn sâu sắc quan trọng giúp ta xem xét vấn đề từ góc nhìn hình học đối xứng”, Greene nói, “Đó thứ thay đổi chơi”.
Cuối tháng4;Greene Lobb xác định nhúng dải Măobius vo khụng gian
(51)Nu bn cú th lm cho [di Măobius] tuõn theo cỏc quy tắc đối xứng, bạn tận dụng số định lý đối xứng”, Lobb nói.
Nhờ điều này, Greene Lobb tự tin họ cải thiện kết Hugelmeyer - có nghĩa họ chứng minh có nhiều phần ba số tất phép quay tạo giao điểm Điều có nghĩa số hình chữ nhật tìm thấy dạng điểm đường cong kín nhiều phần ba tổng số hình chữ nhật với tỉ lệ khác
“Một chắn xảy chúng tơi có ý tưởng này”, Lobb nói
(52)Sự kết nối chai Klein
Chai Klein mặt cong hai chiều trông ging nh mt bỡnh nc Ging nh di Măobius, nú có mặt ta tạo mt bng cỏch dỏn hai di Măobius li vi Bất kỳ chai Klein mà ta tạo đặt bàn, nhiều nhà toán học làm, tự giao với Khơng có cách để nhúng chai Klein vào không gian ba chiều mà khơng tự giao
“Chai Klein cho mặt cong, để từ bên vào bên trong, ta phải xuyên qua chai”, Schwartz nói.
Tuy nhiên, khơng phải ln ln Trong khơng gian bốn chiều, ta nhúng chai Klein cho khơng tự giao Chiều thứ tư cung cấp thêm công cụ để điều khiển chai Klein tự tránh Nó tương tự việc hai người phía đường chiều tránh khỏi va chạm, hai người phía sàn hai chiều dễ dàng tránh
Vào tháng5;Greene Lobb tình cờ nhớ thật thú vị chai Klein: Nó khơng thể
nhúng vào không gian đối xứng bốn chiều mà không tự giao với Nói cách khác, khơng thể có chai Klein không tự giao tuân thủ quy tắc đặc biệt không gian đối xứng Điều chìa khóa cho chứng minh “Đó viên đạn kỳ diệu”, Greene cho biết
(53)Hai di Măobius giao tng ng vi mt chai Klein, loại bề mặt tự giao loại không gian ny V nu ta xoay mt di Măobius cho xoay không giao với gốc, chất, ta tạo chai Klein không tự giao Nhưng chai Klein tồn không gian đối xứng bốn chiều Do đó, phép quay dải Măobius c nhỳng cng phi t giao - ngha đường cong khép kín, trơn phải chứa bốn điểm tạo thành hình chữ nhật với tất tỷ lệ khung hình
Kết luận cuối đến trận tuyết lở
“Nó giống việc liên tục thực hiện, thực hiện, thực hiện, sau điều kì diệu xuất hiện chứng minh hồn thành”, ơng Denne nói.
Chứng minh Greene Lobb ví dụ điển hình để thấy rằng, việc giải vấn đề thường phụ thuộc nhiều vào việc tìm hướng phù hợp để xem xét Nhiều hệ nhà toán học thất bại việc xử lý phiên toán chốt hình chữ nhật họ cố gắng giải bối cảnh hình học truyền thống Khi Greene Lobb chuyển vào giới đối xứng, vấn đề giải
“Những vấn đề đưa năm1910và1920;họ cơng
(54)THƠ VÀ KHOA HỌC - VIẾT CHO TRẺ EM
Hideki Yukawa
Người dịch Doãn Hồng Trang
Thơ khoa học trông xa mà gần, trông gần mà xa Tại ta thấy hai thứ xa nhau? Bởi khoa học giống người thầy giáo nghiêm khác: Ta trả lời thầy qua loa, mà phải cẩn thận làm thí nghiệm phức tạp, phải giải tốn khó Cịn thơ giống người mẹ hiền: Ta có nói nữa, mẹ lúc lắng nghe Trong giới thơ, hoa thơm,
Như dù ta thấy thơ khoa học gần Vì sao? Vì xuất phát điểm chúng giống Cả hai việc nhìn lắng nghe thiên nhiên Khơng có khác nhiều cảm giác ta ngửi hoa hồng ngợi ca vẻ đẹp nó, cảm giác lúc ta nghiên cứu hình dạng bơng hoa
Nhưng làm thơ hoa hồng mang kính hiển vi soi khác Khoa học ngày tiến nhanh chia thành nhiều chun ngành nhỏ Trong phịng thí nghiệm la liệt thiết bị, sách đầy rẫy cơng thức khó hiểu, ta khơng cịn thấy hình bóng thơ Nhà khoa học người quên thơ, người đánh thơ
Nếu tơi khơng biết thơ lần bị có quay trở giới khoa học hay không Cái ta gọi thơ thứ có tính thất thường Khơng thiết ta tìm thơ nơi ta cặm cụi tìm Đơi khi, góc phịng thí nghiệm bừa bộn, nhà khoa học lại bất ngờ phát thơ Trong công thức mà người bình thường khơng thấy hay ho gì, mắt người làm chun mơn lại thấy hình hài tự nhiên đẹp đố hoa hồng mà mắt nhìn thấy Nhưng nhà khoa học cảm nhận vẻ đẹp ẩn giấu tự nhiên Có thể việc tìm lại vẻ đẹp tự nhiên lịng khoa học đặc quyền số học giả xuất sắc Nhưng thơ, người tìm ra, chia sẻ cho người khác
(55)Hình 1: Hideki Yukawa
Giới thiệu tác giả
Hideki Yukawa (tên khai sinh Hideki Ogawa) sinh ngày 23 tháng1 năm1907 tại Tokyo,
Nhật Bản Bố mẹ ông xuất phát gia đình truyền thống võ sĩ đạo (samurai) Nhật Ông người thứ năm bảy anh chị em Lúc Hideki sinh ra, bố ông, Takuji Okawa, làm việc Uỷ ban điều tra địa chất Tokyo, lúc Hideki tuổi ông nhận chức Giáo sư Địa Lí Đại học Kyoto mang gia đình thành phố Kyoto, cố Nhật Bản Lúc mới5 6tuổi Hideki ông ngoại dạy đọc chữ Hán (kanji) qua sách Đại học, Luận
ngữ Mạnh Tử Cách dạy ông ngoại ông lấy que vào chữ sách, Hideki phải theo ông đọc to thành tiếng chữ Mặc dù lúc Hideki khơng hiểu tí ý nghĩa thứ ơng ngoại dạy đọc, tập đọc giúp cho ông đọc sách sớm bạn tuổi
Những năm đầu học phổ thơng ơng khơng khơng có thiên hướng vật lý mà lại say mê văn học khoa học Nhưng trường trung học, ông bắt đầu làm quen với vật lý đại (thuyết tương đối học lượng tử) qua sách thư viện trường bị môn lôi Năm1926Hideki Ogawa trúng tuyển vào khoa Vật Lí trường Đại học Kyoto Một
người bạn học phổ thông trung học với ông, Shin-Ichiro Tomonaga, trúng tuyển vào trường Trong thời gian học đại học hai người bạn giúp đỡ học Sau tốt nghiệp, hai người lại trường làm trợ giảng vài năm Năm1932Hideki Ogawa thành giảng
viên Kyoto, cịn bạn ơng, Tomonaga, chuyển Tokyo Cũng vào năm 1932Hideki cưới vợ
Vợ ông, Sumi Yukawa, nghệ sĩ múa Gia đình vợ ơng khơng có trai, theo truyền thống Nhật, bố mẹ vợ ông nhận ông làm nuôi từ ông mang họ nhà vợ, Yukawa
(56)Những năm Hideki Yukawa bắt đầu làm lúc Cơ học lượng tử định hình nhà vật lý bắt đầu quan tâm đến vấn đề mới: Cấu trúc hạt nhân nguyên tử Lúc người ta biết hạt nhân biến thành hạt nhân khác qua phân rã alpha phân rã beta, qua phản ứng hạt nhân, người ta chưa biết hạt nhân cấu tạo từ hạt Tới năm 1931người ta nghĩ hạt nhân cấu thành từ hạt proton (hạt nhân nguyên tử
hidro) electron Chỉ tới năm1932;sau Chadwick phát hạt neutron, nhà vật lý Đức
Heisenberg nhà vật lí Nga Ivanenko đưa giả thuyết hạt nhân cấu tạo từ hạt proton hạt neutron Câu hỏi lực gắn hạt proton neutron lại với nhau? Lúc người ta biết lực điện từ lực hạt mang điện tích lực giải thích trao đổi hạt photon hạt mang điện Người ta biết tương tác yếu gây phân rã beta số hạt nhân Nhưng lực gắn hạt proton neutron hạt nhân lớn nhiều khơng thể quy vào hai tương tác
Cũng nhà vật lý châu Âu, Yukawa suy nghĩ nhiều năm vấn đề Lời giải cho câu đố chất lực hạt nhân đến với Yukawa vào buổi tối cuối tháng10năm1934W Lực hạt nhân phải mang hạt Nếu hạt có khối lượng, ta giải thích lực hạt nhân lại tác dụng bán kính nhỏ Nói đơn giản ra, ý tưởng Yukawa sau Một hạt có khối lượng m mang lượng E D mc2;trong đóc vận
tốc ánh sáng Nếu khơng có đủ lượng E ta sinh hạt này, theo
học lượng tử, hạt sinh lại biến khoảng thời gian ngắn, độ dài khoảng thời gian h
E;trong đóhlà số Planck (theo hệ thức bất định Heisenberg) Do vận tốc ánh sángclà vận tốc tối đa, khoảng thời gian quãng đường tối đa hạt có
thể chuyển động h E c D
h
mc:Biết lực hạt nhân có bán kính tác dụng khoảng1 femtomet (10 15m), Yukawa tính hạt phải có khối lượng lớn khối lượng electron 200lần Yukawa công bố phát kiến báo “Tương tác hạt bản”, in
năm1935:
Việc Yukawa dùng kiến thức bán kính tương tác lực hạt nhân mà đưa giả thuyết hạt chưa tìm bước nhảy mang tính cách mạng cho vật lý thời Lúc nhà vật lý dè dặt việc đưa giả thuyết hạt Trước Yukawa có hai người làm việc này: Pauli đưa giả thuyết hạt neutrino năm1930;và
Dirac đưa giả thuyết hạt positron (phản hạt electron) năm1931:Việc tìm tịi hạt mang
lực hạt nhân (nay ta gọi hạt pion, kí hiệu là) ban đầu khơng đưa đến kết Năm1937một
hạt có khối lượng khoảng200lần khối lượng electron tìm tia vũ trụ, người
ta nhanh chóng xác định khơng thể hạt mà Yukawa đưa ra, tương tác yếu với proton neutron Hạt ta gọi hạt muon (ký hiệu là) Chỉ đến năm1947hạt pion
mới tìm tia vũ trụ Năm1949 ông trở thành người Nhật Bản giải
thưởng Nobel Bạn học ông, Shin-Ichiro Tomonaga, giải Nobel Vật lý năm 1965;
và người Nhật Bản thứ hai giải thưởng
Từ năm1948đến năm1949;Hideki Yukawa Giáo sư mời Viện nghiên cứu cao cấp Đại
học Princeton năm 1949ông trở thành Giáo sư Vật lý Đại học Colombia Năm1953;để
(57)Hideki Yukawa người đọc rộng chịu ảnh hưởng văn hố phương Đơng lẫn phương Tây Năm1966khi ông đưa ý tưởng không-thời gian gián đoạn, ông liên hệ ý
tưởng với câu văn Lý Bạch: “Trời đất quán trọ muôn vật, thời gian khách ghé trăm đời” Ngoài báo khoa học, ơng cịn để lại nhiều tác phẩm khác, đó có sách“Tabibito”(Người lữ hành), viết đời thân từ nhỏ đến năm
1935;lúc ông hoàn thành xong báo sau mang lại giải Nobel cho ông Kết thúc
sách, ông viết cảm xúc sau khám phá chất lực hạt nhân:
Tôi cảm thấy người lữ hành ngồi nghỉ quán nước nhỏ đỉnh núi. Lúc tơi khơng suy nghĩ đằng trước tơi cịn có núi không.
Tài liệu tham khảo
(58)THUẬT TOÁN GIÚP TĂNG TỐC ĐỘ
XÉT NGHIỆM CORONA
Smriti Mallapaty
Lược dịch - Trịnh Diễm Anh
Các nhà khoa học nói việc xét nghiệm rộng rãi điều cần thiết để kiểm soát việc lây lan virus Tuy nhiên nhiều khu vực thiếu hụt hóa chất cần thiết thực xét nghiệm chẩn đoán Ở số quốc gia, quan chức y tế bắt đầu sử dụng thuật toán mà lần đưa chiến tranh giới thứ hai, phương pháp xét nghiệm theo nhóm Xét nghiệm theo nhóm là1lần xét nghiệm cho1mẫu nhiều người Các nhà nghiên cứu cho biết phương pháp tiết kiệm thời gian, hóa chất chi phí
Roy Kishony nhà sinh học dịch tễ Technion - Israel Institute of Technology, Haifa phát biểu: “Trong đại dịch toàn cầu nay, làm xét nghiệm cho số lượng người bệnh lớn là cần, gộp nhóm lựa chọn đáng quan tâm”.
Các nước Trung Quốc, Ấn Độ, Mỹ, sử dụng phương pháp xét nghiệm nhóm Có nhiều cách để tiến hành xét nghiệm nhóm, nhà khoa học số nước thử nghiệm với cách tốt đại dịch lần lần Phần lớn ý tưởng họ xuất phát từ lĩnh vực toán học, áp dụng cho số toán quy mô như: Phát đèn bị lỗi Giáng Sinh, ước lượng tỷ lệ nhiễm HIV dân số
Dror Baron, nhà khoa học thông tin đại học tọa lạc Raleigh tiểu bang North Carolina, Hoa Kỳ phát biểu: “Đã có hang loạt đổi lĩnh vực này”
Có4phương pháp bật thử nghiệm
Phương pháp 2
Phương pháp sử dụng từ giang mai đến corona Phương pháp đơn giản xét nghiệm nhóm nhà kinh tế học Robert Dorfman đề xướng vào năm1940để kiểm tra người lính mắc bệnh giang mai
(59)riêng lẻ Các nhà khoa học ước tính kích cỡ nhóm hiệu nhất, số lần xét nghiệm dựa tỷ lệ nhiễm virus cộng đồng
Tháng5vừa qua, quyền thành phố Vũ Hán, Trung Quốc sử dụng phương pháp1như nỗ lực xét nghiệm phần lớn cư dân thành phố, đạt được10triệu người sau 2tuần triển khai Các mẫu khoảng2; triệu người sử dụng phương pháp xét nghiệm nhóm, với5mẫu nhóm kết là56người bị nhiễm corona tìm thấy
(60)Krishna Narayanan, thuyết gia thông tin trường đại học Texas A&M cho “Đây phương pháp đơn giản nhất” ơng nói thêm “Nhưng lượt xét nghiệm lần 2;vẫn có nhiều cách hiệu xét nghiệm người riêng lẻ”.
Thêm phiên tốt hơn, có thêm lượt xét nghiệm nhóm trước xét nghiệm riêng lẻ (xem phương pháp trong hình bên dưới) Thêm lượt xét nghiệm nhóm giảm số người cần xét nghiệm riêng lẻ
Tuy nhiên cách tiếp cận chậm, phải vài để có kết xét nghiệm nhóm Wilfred Ndifon, nhà sinh học thuộc viện Khoa học toán Châu Phi, Kigali, Rwanda phát biểu “Đây một bệnh lây lan bùng phát nhanh Chúng ta cần câu trả lời nhanh nhiều so với phương pháp có”.
Phương pháp 3: Đa chiều
Ndifon đồng nghiệp cải tiến phương pháp Dorfman, họ thử nghiệm Rwanda, giảm số lượng xét nghiệm cần làm Ở lượt xét nghiệm đầu, họ làm giống cách Dorfman Tuy nhiên nhóm có kết dương tính, lượt xét nghiệm thứ2;họ phân chia mẫu trùng vào nhóm
(61)Theo lời Neil Turok, nhà vật lý lý thuyết đại học Edinburgh, anh người đồng nghiên cứu việc tăng đa chiều, ví dụ từ hình vng thành khối lập phương cho phép kích cỡ nhóm lớn hiệu cao
Ndifon, thành viên lực lượng đặc nhiệm Covid Rwanda nói xét nghiệm nhóm phần kế hoạch phủ nhằm xác định cách ly nhanh chóng người bị nhiễm Ơng đồng ước tính phương pháp họ giảm chi phí xét nghiệm người từ USD xuống còn0:75USD Các nhà nghiên cứu thực thí nghiệm để biết liệu thực tế số mẫu xét nghiệm mà đảm bảo kết dương tính Leon Mutesa nhà di truyền học Rwanda nhà đồng nghiên cứu thành viên lực lượng đặc nhiệm Covid Rwand bảo phịng thí nghiệm, ơng xác được mẫu dương tính nhóm hỗn hợp gồm100mẫu
Tuy nhiên Sigrun Smola, chuyên gia virus phân tử Trung tâm y tế Saarland Hom-burg, Đức người tiến hành xét nghiệm lên tới 20 mẫu nhóm nói để bảo đảm độ xác, bà khơng khơng khuyến khích có nhiều hơn30mẫu nhóm cho xét nghiệm Một nhóm lớn khó khăn việc tìm virus đồng thời làm giảm hội tìm trường hợp dương tính Smola hoài nghi ứng dựng kỹ cắt khối xét nghiệm thơng thường Bà nói thêm “Nếu bạn nói điều với kỹ thuật viên xét nghiệm, họ nói lộn xộn, họ muốn cách đơn giản hơn”.
Ndifon cịn nói thêm nhóm ơng phát triển phần mềm ứng dụng để tự động xếp mẫu
Phương pháp 4: Giải pháp lượt
(62)chờ kết từ vòng đến cuối cùng, điều làm chậm trình, Manoj Gopalkrishnan khoa học máy tính viện cơng nghệ Bombay, Ấn Độ phát biểu
Thay vào đó, Gopalkrishnan đề xuất thực xét nghiệm1 lượt với nhiều nhóm chồng chéo Điều tăng số lượng xét nghiệm lên lại giảm thời gian chờ kết xét nghiệm, nhiên nhiều thời gian cho việc tạo lập nhóm mẫu có thêm nhiều nhóm nghĩa có thêm nhiều mẫu cần trích xuất
Cách tiếp cận Gopalkrishnan liên quan đến việc trộn lẫn mẫu nhiều nhóm xét nghiệm khác nhau, dùng kỹ thuật Kirkman tức dùng quy tắc cách phân phối mẫu Hãy tưởng tượng ma trận phẳng hàng đại diện cho xét nghiệm cột đại diện cho xét nghiệm (xem Phương pháp4trong hình bên trên). Nói chung, xét nghiệm phải có số lượng mẫu số lần xét nghiệm mẫu giống
Nhưng Narayanan nói phương pháp đòi hỏi nhiều xét nghiệm để bảo đảm mức độ xác cách xét nghiệm nhiều vòng Cách tiếp cận phương pháp lượt xét nghiệm số lượng lớn mẫu lúc, điều gây khó khăn Ơng nói thêm “Một kỹ thuật viên xét nghiệm làm điều này, điều cần hệ thống robot”.
Để đơn giản phịng thí nghiệm, Gopalkrishnan đồng nghiệp phát triển ứng dụng di động, ứng dụng cho người dùng cách thức để trộn mẫu Theo kết chưa công bố rộng rãi từ phòng thử nghiệm lâm sàng Mumbai (Ấn Độ), Gopalkrishnan nói với48xét nghiệm, xác định được5mẫu dương tính trong320mẫu
Các nhà nghiên cứu Israel sử dụng hệ thống tự động ứng dụng tương tự hệ thống test lượt Moran Szwarcwort - Cohen, người đứng đầu phịng thí nghiệm virus Rambam Health Care Campus, Haifa nói nhóm bà đánh giá lại hệ thống kết khả quan
Tài liệu tham khảo
(63)ĐUỔI BẮT VÀ CHẠY THOÁT
Trịnh Đào Chiến
Gia Lai
GIỚI THIỆU
Lý thuyết trị chơi nhánh Tốn học ứng dụng Ngành nghiên cứu tình chiến thuật, đối thủ lựa chọn hành động khác để cố gắng làm tối ưu kết nhận Bắt đầu từ năm 1970;Lý thuyết trò chơi bắt đầu
được áp dụng cho nghiên cứu hành vi động vật, có phát triển loài qua chọn lọc tự nhiên, chẳng hạn hành viĐuổi bắt Chạy thoát
Bài viết đề cập vài câu chuyện vậy, từ dân gian đến đại
Từ tốn dân gian
Chuyện dân gian kể rằng, có cáo thỏ sống trong17cái hang động dọc theo đỉnh núi tạo thành vòng tròn lớn Khoảng cách hang xa Cáo ln muốn tìm cách để ăn thịt thỏ
Một hơm, cáo thỏ nhìn từ xa Thỏ nói với cáo: “Anh khơng cần lúc phải tìm cách ăn thịt tơi Chúng ta đánh số thứ tự cho 17cái hang, từ số 1đến số17: Tôi chọn ba hang gần nhau, hang ở10ngày Anh xuất phát từ hang số17:Lần thứ cách hang, đến hang số1 tìm tơi Lần thứ hai cách hai hang, đến hang số3tìm tơi Lần thứ ba cách 3cái hang, đến hang số tìm tơi Cứ vậy, vịng30 ngày, khơng quan trọng anh vào hang lần, cần anh tìm thấy tơi anh ăn thịt tơi ngay.”
Cáo nghĩ: “Trong vịng tháng, ta định tìm mi, ta ăn thịt mi” Vì cáo đồng ý Nhưng cáo giảo hoạt tìm suốt tháng mà khơng tìm thấy thỏ Bạn cho biết, thỏ thông minh trong3chiếc hang liền kề không?
Trước tiên, phân tích lần cáo vào hang, hang số Lần thứ nhất, cáo cách hang, vào hang số1:
(64)Lần thứ tư, cáo cách bốn hang, vào hang số1C2C3C4D10:
Lần thứn, cáo cáchncái hang, vào hang số1C2C3C CnD n.n2C1/: Sau bảng số dư củan,nC1, n.n2C1/ chia cho17
n nC1 n.n2C1/
1
2 3
3
4 10
5 15
6
7 11
8
9 10 11
10 11
11 12
12 13 10
13 14
14 15
15 16
16 17
17 18
18 19
19 20
(65)
đến phương pháp “Chạy thoát”
Tiếp tục câu chuyện cáo thỏ Một cáo nhẩn nha khu vực hình trịn rộng lớn khép kín, thấy thỏ bắt đầu chạy khu vực Ngay lập tức, cáo nhắm hướng thỏ chạy đuổi theo với vận tốc tương đương với thỏ tội nghiệp Xem vật điểm di dộng hình trịn Câu hỏi đặt là, liệu cáo bắt thỏ khoảng thời gian hữu hạn hay khơng? Và thỏ tinh ranh tội nghiệp có cách chạy thoát để cáo không vồ
Vấn đề đặt tưởng chừng đơn giản vào cuối năm30của kỷ trước, trở thành đề tài trao đổi sơi số nhà toán học quan tâm
Thời trẻ con, tham gia vào trò chơi đuổi bắt, bé bị rượt đuổi thường tinh ranh chạy dích dắc loằng ngoằng để bé sau khơng bắt Có phải từ phương pháp “Chạy thốt” trị chơi dân gian mà nhà tốn học A S Besicovitch đề xuất hướng giải độc đáo cho tốn
Lời giải Khơng tính tổng quát, ta giả sử đuổi bắt bắt đầu, thỏ
ở điểmT0nằm bên hình trịn bán kính bằng1:Để theo dõi diễn biến khoảng cách, ta đặt1 " < 1là độ dài khoảng cáchOT0:
Hình
Giả sử ta dựng điểmTk thỏ nTk thỡ cỏo nSk ÔTk 1: tTk choTk 1Tk vng góc vớiOTk 1;có chiều dàilk, giả sửSk 1không nằm nửa mặt phẳng mở xác định đườngOTk 1chứaSk (xem hình 1)
(66)Tổng chiều dài đường gấp khúcT0T1T2 P1
kD1
lk Nếu tổng vô hạn, nghĩa thỏ chạy dọc theo đường gấp khúc mãi, khơng bị bắt Đây giả thuyết vì, tồn hay khơng đường gấp khúc mà nằm bên hình trịn?
Ta có
OTn2 D.1 "/2C n
X
kD1
lk2< "C n
X
kD1 lk2: Do đó, đường gấp khúcT0T1T2 ln nằm bên hình trịn
1 X
kD1
lk2 < ":
Vấn đề lại là, dãyl1; l2; : : :thỏa mãn điều kiện Đặt lk Dck 34;
trong đóc D "2:Khi
1 P
kD1
lk vô hạn
1 X
kD1
lk2 Dc2
1 X
kD1
k 32 < c2
0
@1C Z
1
x 32dx
1
AD3c
2 < ":
Vậy dãy.lk/xác định dãy thỏa mãn điều kiện Bài toán giải
Từ trò chơi vi phân
Lại câu chuyện cáo thỏ Một cáo nhẩn nha, thấy thỏ bắt đầu chạy băng qua cánh đồng, đường thẳng vng góc với đường thẳng nối vị trí ban đầu cáo thỏ Ngay lập tức, cáo nhắm hướng thỏ chạy đuổi theo với vận tốc tương đương với thỏ tội nghiệp
Trong hệ tọa độ vng góc (hình 2) giả sử rằng, thời điểm đầu tiên, thỏ gốc tọa độ cáo điểm.L; 0/ ; L > 0:Chú thỏ chạy lên theo trụcOyvà cáo nhắm thẳng hướng thỏ chạy mà đuổi theo với tốc độ Khi cáo chạy đến điểm.x; y/thì thỏ chạy đến điểm.0; y1/,y1 > 0:
Ta nhắc lại rằng, độ dốc đường thẳng i qua hai im.x1; y1/v.x2; y2/ ;vix1 Ôx2;l m D y2 y1
(67)Hình
Lưu ý rằng, hai vật chạy với tốc độ nêny1 (quãng đường chạy thỏ) độ dài cung (quãng đường chạy chó) từ điểm L; 0/đến điểm x; y/ :Hơn nữa, ta biết độ dài cung tính theo cơng thức tích phân sau
L Z x s 1C dy dx dx: Vì độ dốcmtạix ddyx;nên ta có
dy dx D
y L
R
x
q
1C ddyx
2
dx
x ;
hay
x dy dx Dy
L Z x s 1C dy dx dx: Bây giờ, để loại bỏ tích phân, ta cần lấy đạo hàm hai vế Ta có
d dx
x dy dx D d dx @y L Z x s 1C dy dx dx A;
biến đổi tương đương thành dy dx Cx
d2y dx2 D
(68)x d 2y dx2 D
d dx @ L Z x s 1C dy dx dx A;
x d 2y dx2 D
d dx @ x Z L s 1C dy dx dx A:
Bây giờ, sử dụng cơng thức tích phân d dx @ x Z a
f t /dt
1
ADf x/ ;
ta có
x d 2y dx2 D
s 1C dy dx :
Giải phương trình vi phân phương pháp tách biến cách đặtz D ddyx: Ta có phương trình
x dz dx D
p
1Cz2;
do Z
dz p
1Cz2 D
Z dx
x : Lại đặtz Dtan;ta có
1Cz2 D
cos2 Dsec
dz Dsec2d:Vì vậy
Z sec2 d
sec DlnxCC;
Z
secd DlnxCC;
lnjsecCtanj DlnxCC: Để xác định sec tan theoz, ta xét tam giác vng hình Ta cóz Dtan sec Dp1Cz2:Do phương trình
lnˇˇ ˇ
p
z2C1Czˇˇ
ˇDlnxCC;
tương đương với
elnj
p
z2C1Cz
j DelnxCC; hay
ˇ ˇ ˇ
p
z2C1Czˇˇ ˇDe
(69)Hình
p
z2C1Cz D ˙eC x: ĐặtAD ˙eC;phương trình trở thành
p
z2C1Cz DAx: Giải phương trình ta
z D Ax
1 2Ax: Suy
dy dx D
Ax
1
2Ax ,y D
Z
Ax
1 2Ax
dx ,y x/D Ax
4
1
2AlnxCD: Cuối sử dụng điều kiệny L/D0; y0.L/ D0để tìm số thích hợp Ta có
y0.L/D AL
1
2AL D0: Suy raAD L1;hơn lại có
y L/D AL
4
1
2AlnLCD D L
4 L
2 lnLCD D0: cho nênD D L2 lnL L4:
Vì
y x/D x
4LC L
2 lnL L
4:
(70)Hình
đến tốn IMO
Đó tốn số3của IMO2017;một tốn “sát thủ”! Nó khó đến mức, trong112quốc gia tham dự với hơn600thí sinh, thí sinh Australia giải trọn vẹn (điểm7) vài em điểm1Š
Bài tốn sau (bạn đọc xem lời giải bình luận [1]):
Bài tốn Thỏ thợ săn chơi trị đuổi bắt trốn tìm mặt phẳng Euclid Tại thời điểm0; cả hai vị trí.A0 DB0/ :Tại thời điểmnthì có việc sau xảy ra
Thỏ nhảy từ vị tríAn 1đến vị trí bí mậtAncáchAn 1một khoảng bằng1:
Thợ săn có máy dị tìm vị trí thỏ với sai số1đơn vị, cho thợ săn biết vị tríPn cáchAnkhơng q1(ngồi khơng biết thêm) thời điểmn:
Thợ săn phải chọn nhảy đến vị tríBncáchBn một khoảng bằng1:
(71)Nếu dự đoán câu trả lời khẳng định, tốn khó có hai lần bất định Nếu dự đoán câu trả lời phủ định, tốn dễ chịu theo logic phủ định, ta cần trường hợp mà thỏ chạy xa người thợ săn, có nghĩa ta chủ động đặt thiết bị định vị đâu tuỳ ý, cần thoả mãn điều kiện khoảng cách
Giáo sư Nguyễn Tiến Dũng (Đại học Toulouse, Pháp - Huy chương Vàng IMO 1985) hóm hỉnh bình luận: Khơng rõ học sinh phổ thơng mà gặp phải mà“nhằn”được! Đọc đề có cảm giác liên quan đến Toán học mức cao cấp, học đại học, Điều khiển tối ưu, Giải tích tiệm cận, Min-Max, Trò chơi vi phân,
Nhà thơ VCH (Tây nguyên) nghe “lóm” câu chuyện tốn này, phá lên cười: Chà, ơng thợ săn thỏ rảnh hè!
Tài liệu tham khảo
[1] Lời giải bình luận IMO 2017, Ban biên tập Epsilon.
https://www.fb.com/TapchiEpsilon/posts/1702604733368559
[2] Bài tốn sư tử đấu sĩ, Hồng Đức Tân, Tạp chí Tốn học Tuổi trẻ, số 515. [3] Calculus: Early Transcendentals, James Stewart,
(72)THẶNG DƯ TOÀN PHƯƠNG
BỔ ĐỀ GAUSS
VÀ LUẬT THUẬN NGHỊCH BẬC HAI
Nguyễn Thái Vũ
Hà Nội
GIỚI THIỆU
Năm1801đánh dấu mốc quan trọng lịch sử phát triển toán học Gauss
trình làng cuốnDisquisitiones Arithmeticae, đánh dấu đời số học đại Phần lớn trongDisquisitiones Arithmeticaetập trung vào thặng dư toàn phương vấn đề xoay quanh Các nghiên cứu dẫn đến ý tưởng chủ chốt xuất khắp nơi số học ngày
Định nghĩa Với số nguyênm > 1và số nguyênanguyên tố vớimđược gọi một thặng dư tồn phương modulomnếu phương trình đồng dưx2a mod m/có nghiệm. Nhận xét:
Nếualà thặng dư tồn phương modulom; b a mod m/thì hiển nhiênbcũng thặng dư toàn phương modulom:
Với a; m/ D 1; hiển nhiên nghiệm phương trình x2 a mod m/ nguyên tố với m; việc bình phương thặng dư hệ thặng dư thu gọn modulo m ta tìm tất thặng dư tồn phương modulo m: Khi xét modulo p nguyên tố lẻ, ta kiểm tra bình phương phần tử tập f1; 2; 3; : : : ; p 1g đơn giản ˚
˙1; ˙2;˙3; : : : ;˙p 12 (cũng hệ
thặng dư thu gọn modulop) Tập hợp bình phương hệ gồm có p 12 phần tử đơi khơng đồng dư modulop:Từ ta có định lý sau
Định lý Với số nguyên tố lẻp;số thặng dư toàn phương số bất thặng dư toàn phương modulopcùng là p 12 :
Định nghĩa (Kí hiệu Legendre) Với số nguyên tố lẻp;số nguyênathỏa mãn.a; p/ D1; kí hiệu Legendrea
p
(73)
Việc sử dụng kí hiệu Legendre giúp ta đưa mối quan hệ định tính trở thành định lượng, thuận tiện cho đánh giá, tính tốn dẫn đến nhiều tính chất đẹp Trước tiên, ta xem lại tiêu chuẩn Euler quen thuộc
Chop là số nguyên tố lẻ số nguyênathỏa mãn.a; p/ D 1:Khi đóap21 mod p/
nếu phương trình x2 a mod p/có nghiệm, vàap21 1 mod p/nếu phương
trìnhx2 a mod p/vơ nghiệm.
Sử dụng kí hiệu Legendre, ta có phát biểu tương đương
Định lý (Tiêu chuẩn Euler) Vớiplà số nguyên tố lẻ số nguyênathỏa mãn.a; p/ D1; khi đó
a
p
ap21 mod p/ :
Tính chất Ta có số tính chất sau
i) ab modp/thìpaDpb: ii) ab
p
Dap b p
: iii) a2
p
D1: iv) a2b
p
Dpb
:
Các tính chất (i) (iii) hiển nhiên theo định nghĩa Chứng minh (ii): Ta có
ab p
.ab/p21 Da p
2 b p a p b p
.mod p/ : Vìab
p
vàa p
b p
đều nhn giỏ tr l1hoc 1;mp Ô 2nờn khụng th xy trường hợp1 mod p/ ;do
ab p D a p b p : Chứng minh (iv): Ta có
a2b p D a2 p b p D b p :
Việc xây dựng tiêu chuẩn để kiểm tra số có phải thặng dư tồn phương modulophay khơng có ý nghĩa quan trọng, ta xem xét vài số dư thường gặp
(74)Chứng minh Theo thiêu chuẩn Euler
1
p
1/p21.mod p/ ;
suy
1/p21 1 modp/ ;
màplẻ nên 1/p21 D1;do p
2 chẵn hayp mod 4/ :
Định lý Vớiplà số nguyên tố lẻ Khi đó2là thặng dư toàn phương modulopnếu thỏa mãnp ˙1 mod 8/và bất thặng dư toàn phương modulop nếup ˙3 mod 8/ :
Chứng minh Vớip mod 8/ ; p mod 8/ ;khi đó4j p 1/ :Xét khai triển sau 2p21
p
2
ŠD246 p 1/246
p p
5/ 3/ 1/ 1/p41
p
2
Š mod p/ : Vì p
2
Š; p D1nên chia hai vế cho p 12 Š;ta 2p21 1/
p
4 .mod p/ :
Áp dụng tiêu chuẩn Euler suy
2
p
1/p41.mod p/ ;
màplẻ nênp2D 1/p41:Do nếu2là thặng dư tồn phương modulop thì 1/ p
4
chẵn, dẫn đến p mod 8/ ;nếu2là bất thặng dư tồn phương modulo p 1/p41
lẻ, dẫn đếnp mod 8/ :
Vớip mod8/ ; p mod8/ ;khi đó4j pC1/ ;tương tự ta xét khai triển sau 2p21
p
2
ŠD246 p 1/246
p p
5/ 3/ 1/ 1/pC41
p 1
2
Š mod p/ : Vì p
2
Š; pD1nên chia hai vế cho p 12 Šta được2p21 1/ pC1
4 .mod p/ :Áp dụng
tiêu chuẩn Euler suy
2
p
1/pC41 .mod p/ ;
màp lẻ nênp2D 1/pC41:Do nếu2là thặng dư tồn phương modulopthì 1/ pC1
4
chẵn, dẫn đếnp mod 8/ ;nếu2là bất thặng dư tồn phương modulop 1/pC41
(75)Tính chất Vớiplà số nguyên tố lẻ, đó
(i) 2là thặng dư toàn phương modulop khi khip 1; mod 8/ : (ii) 3là thặng dư toàn phương modulop khi khip ˙1 mod 12/ : (iii) 3là thặng dư toàn phương modulop khi khip mod 6/ :
(iv) 5là thặng dư toàn phương modulop khi khip ˙1 mod 5/ :
Sau xem xong chứng minh “định lý 3” đa phần cảm thấy chứng minh đẹp đầy kĩ thuật, không thật tự nhiên (tại lại module 8‹/: Vậy việc chứng minh “định lý 4” tìm thêm tiêu chuẩn cần khó khăn tương tự? Bổ đề Gauss giới thiệu, với mục đích đánh giá giá trị củaa
p
vớipcho trước, để tìm số nguyên tố lẻpmà nhậnacho trước làm thặng dư toàn phương Phần viết làm người đọc thấy chứng minh sáng sủa nhiều
Định lý (Bổ đề Gauss) Vớiplà số nguyên tố lẻ số nguyênathỏa.a; p/ D1:Xét tập các thặng dư dương nhỏ modulo p củaa; 2a; 3a; : : : ;.p 1/a2 :GọiN là số thặng dư lớn hơn p
2;khi đó
a p
D 1/N:
Chứng minh Nhận thấy a; 2a; 3a; : : : ;.p 1/a2 nguyên tố với p; đôi không đồng dư modulop:Đặtu1; u2; u3; : : : ; uN số dư lớn p2 vàv1; v2; v3; : : : ; vM
là số dư nhỏ p
2:Khi đóM CN D p
2 :
Xét dãy sốp u1; p u2; p u3; : : : ; p uN số dương nhỏ p2; nguyên tố
nhau vớip đôi không ng d modulop: Hn nap ui Ô vj; 8i; j (nếuui D ar;
vj Dasthì cóui Cvj Da r Cs/hiển nhiên nguyên tố vớip vìr Cs < p/:
Do đóp u1; p u2; p u3; : : : ; p uN;vàv1; v2; v3; : : : ; vMchính tập hợp
˚
1; 2; 3; 4; : : : ; p 12 : Ta có
p 1
2
ŠD.p u1/.p u2/.p u3/ p uN/v1v2v3 vM
u1/ u2/ u3/ uN/v1v2v3 vM.mod p/
1/Na2a3a
p 1
2 a
.mod p/ 1/Nap21
p 1
2
Š mod p/ :
Mà p Š; p D1;
nên 1/N ap21 mod p/, áp dụng tiêu chuẩn Euler, ta 1/N
a p
1 modp/ ; màa
p
(76)
Ta nhận thấy phát biểu bổ đề Gauss giá trị sốN không quan trọng, điều mà ta thực quan tâm tính chẵn lẻ (parity) củaN:Ví dụ sau làm rõ ý nghĩa bổ đề Gauss
Ý nghĩa bổ đề Gauss Ta sử dụng bổ đề Gauss để tìm số nguyên tố nhận thặng dư toàn phương Ta xét dãy số5; 10; 15; : : : ;5.p 1/2 nhỏ 5p2 :Trong dãy này, dễ thấy số có số dư lớn p
2 chia p số 5j nằm khoảng p
2; p
3p ; 2p
:Do đó, ta cần đánh giá số cách chọnj cho p2 < 5j < p; 3p2 < 5j < 2p: Đặtp D20kCr vớir 2 f1; 3; 5; 7; 11; 13; 17; 19g;bất đẳng thức trở thành
10k C r
2 < 5j < 20kCr; 30kC 3r
2 < 5j < 40kC2r:
(Sau đọc hết phần này, người đọc tự giải thích xem lại chọn số20‹) Tương đương với
2kC r
10 < j < 4kC r
5; 6kC 3r
10 < j < 8kC 2r
5 :
Đến đây, lưu ý ta quan tâm đến tính chẵn lẻ số cách chọnj:Ta có nhận xét sau: Kí hiệu số số nguyên khoảng a; b/ là n a; b/ ; khi đó n aCt; bCt / D n a; b/ và n a; bCt /Dn a; b/Ct:
Do đó, vớit chẵn tính chẵn lẻ củan a; bCt /vàn a; b/là nhau, nên để xét tính chẵn lẻ số cách chọnj thỏa mãn2kC10r < j < 4kC
r
5;thực chất ta cần xét với r
10 < j < r 5:
Tương tự với6kC 3r10 < j < 8kC 2r5 ta đưa xét 3r10 < j < 2r5:
Vớir D 1;có0cách chọn 10r < j < 5r; 0cách chọn 3r10 < j < 2r5 nên tổng số cách chọnj 0;dẫn đếnN D0;nên theo bổ đề Gauss thì5là thặng dư tồn phương củap:
Vớir D 3;có0cách chọn 10r < j < 5r; 1cách chọn 3r10 < j < 2r5 nên tổng số cách chọnj 1;dẫn đếnN D1;nên theo bổ đề Gauss thì5khơng thặng dư toàn phương củap:
Dễ dàng kiểm tra qua tất giá trị củar;ta rút kết luận5là thặng dư toàn phương củap khir D1; 9; 11; 19hay khip ˙1; ˙9 mod 20/ :
Điều tương đương với kết luận “định lý 4” trước, thặng dư toàn phương modulop khip ˙1 mod 5/ :
Qua ví dụ này, ta thấy giá trị của5 p
chỉ phụ thuộc vào số dư chiap cho20D45;tương tự giá trị của7
p
chỉ phụ thuộc vào số dư chiap cho28 D 47hay giá trị củap2phụ thuộc vào số dư chia pcho 8D 42mà ta xử lý “định lý 3” từ dẫn đến cách tiếp cận tự nhiên nhiều cho định lý này.
(77)Chứng minh Xét dãy2; 4; 6; : : : ; p 1là số nhỏ p nguyên tố vớip:Dễ thấy số có số dư chiaplớn hơnp2 số có dạng2j mà p2 < 2j < p:Đặtp D8kCr được4kCr2 < 2j < 8kCr ,2kC
r
4 < j < 4kC r
2:Ta đưa xét tính chẵn lẻ số cách
chọnj thỏa mãn r4 < j < r2:Thử với r D 1; 3; 5; 7dễ dàng nhận kết quảr D 1; 7thì N chẵn đóp2D1:
Hãy chứng minh định lý sau: Vớip là số nguyên tố lẻ, đó
(v) 2là thặng dư toàn phương modulop khi khip 1; mod 8/ : (vi) 3là thặng dư toàn phương modulop khi khip ˙1 mod 12/ : (vii) 3là thặng dư toàn phương modulop khi khip mod 6/ :
Sau đọc kĩ mục phần ý nghĩa bổ đề Gauss có lẽ người đọc có câu trả lời cho câu hỏi: “Tại lại viếtpdưới dạng20kCr;hay tổng quát lại xét với modulo 4a‹” Ý nghĩa nằm việc đảm bảo gia sốt công thứcn a; bCt /Dn a; b/Ct chẵn để có đượcn a; b/vàn a; bCt /cùng tính chẵn lẻ Cụ thể
p
2 < 5j < p,
20kCr
5 < j <
20kCr
5 ;
,2kC r
10 < j < 4k C r 5; có gia sốt D2k chẵn nênn 2kC 10r I4kC r5
vàn 10r Ir5
cùng tính chẵn lẻ, nên toán quy xác định số cách chọn j thỏa mãn 10r < j < r5:Từ ta nhận thấy, thay xét modulo4a dùng8a; 16a; : : :cũng Con số4là cách chọn “vừa đủ tốt” Qua ví dụ này, ta rút nhận xét: “Giá trị củaa
p
hoàn toàn phụ thuộc vào số dư chia p cho4a” Và đó, nếuq p mod 4a/thìapDaq
:Ta có định lý sau
Định lý Vớip; qlà số nguyên tố lẻ,alà số nguyên dương thỏa.a; p/ D.a; q/D1;khi đó nếup q mod4a/hoặcp q mod 4a/thìapDaq:
Ý tưởng phép chứng minh định lý trình bày phần ý nghĩa bổ đề Gauss, xin nhường lại cho người đọc Đến ta đủ cơng cụ để chứng minh luật thuận nghịch bậc hai, kết quan trọng số học
Định lý (Luật thuận nghịch bậc hai Gauss) Vớip; qlà hai số nguyên tố lẻ phân biệt, đó pq
Dqp
nếu hai số có dạng4kC1: pq
D qp
(78)
Chứng minh Ta xét hai trường hợp:p q mod 4/vàp q mod 4/ : Vớip q mod4/ ;tồn số nguyênmđểp DqC4m:Khi
p q D
qC4m q
; màqC4m4m mod q/nên theo “tính chất 1”
p q D q C4m q D 4m q D m q : Tương tự q p D p 4m p D 4m p D m p D 1 p m p : Nhân theo vế ta
p q q p D 1 p m p m q ; theo “định lý 7” thìm
p
Dmq;do đómp mqD1haypq pqDp1:
Từ suy p mod4/ q mod 4// pq pq D (theo “định lý 3”) hay
p q
Dpq
;nếup mod 4/thìpq pqD 1haypqD pq
: Vớip q mod4/thìp D qC4m:Khi
p q D q C4m q D 4m q D m q ; q p D p C4m p D 4m p D m p : Nhân theo vế ta đượcp
q q p
Dmq m p
D1(theo “định lý 7”), đópqDpq
: Kết hợp hai trường hợp ta có chứng minh hoàn tất
Để ý rằng, p và
q
2 cùng lẻ khip; qđều có dạng4kC3:Do ta có phát biểu
ngắn gọn sau luật thuận nghịch Gauss.
Định lý (Luật thuận nghịch bậc hai) Vớip; qlà số nguyên tố lẻ phân biệt, đó
p
q q p
D 1/p21 q
2 :
Luật thuận nghịch bậc hai cho phép ta đưa việc tínha p
về việc tính tích kí hiệu Legendre với mẫu nhỏ thơng qua “tính chất (ii)” Tuy nhiên q trình bao gồm việc phân tícha thành tích thừa số ngun tố, khơng khả thi với sốarất lớn
(79)Định nghĩa (Kí hiệu Jacobi) Với sốm D Q
piki vớipi là số nguyên tố lẻ,.a; m/ D 1;
khi kí hiệu Jacobi a m
được định nghĩa là
a m DY a pi ki ; vớia
pi
là kí hiệu Legendre.
Tính chất Do cách định nghĩa nên a m
D khơng có nghĩa phương trình x2 a mod m/ có nghiệm, bù lại luật thuận nghịch bậc hai tính chất quan trọng của kí hiệu Legendre giữ nguyên.
Vớim; nlà số nguyên dương, đó
(i) Nếu.a; m/D1,ab mod m/thì maD mb
: (ii) Nếu.a; m/D.b; m/D1thì abm
D ma
b m : (iii) m
D1khi khim1 mod 4/ : (iv)
m
D1khi khim ˙1 mod 8/ : (v) Với.m; n/D1thì mn mn D 1/m21
n
2 (luật thuận nghịch bậc hai).
Bên cạnh đó, ta có thêm tính chất từ định nghĩa (vi) Nếu.a; m/D.a; n/D1thì mna
D ma
a n
:
Vớialà số nguyên lớn hơn1;ta viếtpadưới dạng
2k
p
Y
q
iti
p
; vớiqi số nguyên tố lẻ Với trường hợpaâm, ta viết
a p
D p1 a p
nên kí hiệu Legendre quy việc tính
p
;p2vàqpvớiqlà số nguyên tố lẻ Để minh họa cho điều này, xem xét ví dụ sau, toán tiếng L.Euler đưa dạng giả thuyết kỉ XVIII
Bài tốn (Euler) Chứng minh phương trình sau khơng có nghiệm nguyên dương
(80)Lời giải Phương trình tương đương
4zt2C1D.4xz 1/ 4yz 1/ ; suy ra.4yz 1/ j.2zt /2Cznên kí hiệu Jacobi4yz 1z D1: Đặtz D2kr vớir lẻ, ta có
z 4yz D 4yz 4yz k r 4yz : Theo “tính chất 3”
4yz
D 1:Áp dụng luật thuận nghịch bậc hai
r 4yz D 4yz 1 r
1/4yz2 2: r
2 D
4yz 1
r
1/r21
D
4y
2kr 1
r
1/r21 D
1
r
1/r21 D1:
Do z 4yz
D 4yz 12
k
:
Nếukchẵn thì4yz 1z D 1;mâu thuẫn
Nếuklẻ thì4yz 1D42kry 1 mod 8/ ;suy ra4yz 12 D1nên
z 4yz D 4yz k
D 1; (mâu thuẫn): Vậy phương trình vơ nghiệm
1 Các ví dụ minh họa
Ví dụ Cho p; q là số nguyên tố lẻ thỏa mãn p q mod 26/ : Chứng minh rằng
13 p
D13q :
Lời giải Vì13D4kC1;áp dụng luật thuận nghịch Gauss, ta
13
p
Dp 13 ; 13 q
D q 13
: Vìp q mod 26/nênq D26mCp;do 13q
D 26m13Cp
D 13p
:
Ví dụ Với kí hiệu Jacobi ma
(81)Ví dụ Với hai số nguyêna; bvà số nguyên tốqcó dạng4kC3sao choqj a2Cb2:Chứng minh rằngqjavàqjb:
Lời giải Nếu.a; q/D1thì.b; q/D1:Khi qj a2Cb2
,a2 b2.mod q/ : Suy
q
D1nênqD4k C1(theo “định lý 3”), mâu thuẫn
Ví dụ Chứng minh với số nguyên tốp D3kC2thì pj m2CmnCn2,
(
pjm pjn
Lời giải Nếu.p; m/D.p; n/D1thì từpj m2CmnCn2suy rapj4 m2CmnCn2; ,pj 2mCn/2C3n2, 3n2 2mCn/2.mod p/ ;
nên p
D1;theo “tính chất 2” thìp mod 6/ ;mâu thuẫn
Ví dụ Chứng minh có vơ số số nguyên tố có hàng đơn vị là9:
Lời giải XétN D5.nŠ/2 1:Ta chứng minh tồn ước nguyên tố củaN lớn hơnnvà có dạng10k 1:
Gọiplà ước nguyên tố củaN;suy ra5.nŠ/2 mod p/nên 5.nŠ/2
p
!
D1: Suy ra1D5.nŠ/p
Dp5
theo “tính chất 1”, đóp ˙1 mod 5/theo “tính chất 2” Do N có dạng 5k 1nên tồn ước nguyên tố p có dạng 5k 1;màp khơng thể có dạng 10k C4nênp D10k 1:Rõ ràngp > nvì nếup < nthì5.nŠ/2 ::: p:
Do đó, với số tự nhiênnbất kì ln có ước ngun tố dạng10k 1của5.nŠ/2 1lớn hơnn;hay số số ngun tố có tận là9là vơ hạn
Ví dụ (Korean 2000) Cho số nguyên tốp D4kC1:Tính
p
X
xD1
2x2 p
2
x2 p
:
Gợi ý: Sử dụng “định lý 1”
(82)Lời giải Giả sử tồn tạip ước nguyên tố dạng8k 1của2nC1;khi
Nếun chẵn 2n C1 có dạng a2C1; suy a2.mod p/hay p1 D 1; theo “định lý 3” thìp mod 4/ ;mâu thuẫn
Nếun lẻ thì2nC1 có dạng 2a2 C1;suy ra1 2a2.mod p/hay p2 D 1; theo “định lý 2” thìp 1; mod 8/ ;mâu thuẫn
Ta có điều phải chứng minh
Ví dụ (Serbian 2008) Tìm nghiệm ngun khơng âm phương trình
12x Cy4 D2008z:
Lời giải Nếuy D0;suy đượcxDz D0:Xét trường hợpy >
Nếux chẵn, vế trái có dạnga2Cb2;vì251là ước ngun tố dạng4kC3của2008nên 251ja; 251jb;vơ lý vì.12; 251/D1:
Nếu x lẻ, vế trái có dạng 3a2 C b2; 2513
D 1: Tuy nhiên 2513
D 113
mà
3 11
D 1, mâu thuẫn Do phương trình vơ nghiệm khiy > 0: Vậy.x; y; z/D.0; 0; 0/là nghiệm
Ta tính 251
bằng cách dùng luật thuận nghịch bậc hai:
3 251 D 1 251 251 D 3 251
; 251D4kC3: Áp dụng luật thuận nghịch bậc hai
3
251
251
D 1/321 251
2 D 1:
Do 3 251 D 251 ) 3 251 D 3 251 D 251 D 2 D 1:
Ví dụ Tìm số ngun tốpsao chopŠCplà số phương.
Lời giải Dễ thấyp D2;hoặcp D3thỏa mãn đề
Xétp > 3; ta cópŠCp Dp p 1/ŠC1/suy ra.p 1/ŠC1 Dpx2vớix 2Z:Vìp > nên3j p 1/Š;suy
(83)Lấyalà bất thặng dư toàn phương lẻ modulop; < a < p suy raaj p 1/Š;từ p 1/ŠC1Dpx2 )px2 mod a/)p
a
D1: Áp dụng luật thuận nghịch bậc hai cho kí hiệu Jacobi
p
a
a
p
D 1/p21 a
2 )
p
a
D
a p
1/p21 a
2 ;
màp D4kC1;apD 1nên pa D1;mâu thuẫn Do khơng tồn sốp > 3thỏa mãn đề
2 Bài tập tự luyện
Bài Chop số nguyên tố lẻ, q bất thặng dư toàn phương nhỏ p:Chứng minh rằngqlà số nguyên tố
Bài Choalà thặng dư toàn phương modulop: Chứng minh acũng thặng dư toàn phương modulop khip mod 4/ :
Bài Chứng minh vớip số nguyên tố lẻ
p
P
iD1
.i=p/D0:
Bài Chop số nguyên tố lẻ, q bất thặng dư toàn phương nhỏ p:Chứng minh rằngqlà số nguyên tố
Bài Chop số nguyên tố lớn hơn5:Chứng minh ba số 2; 5; 10là thặng dư toàn phương modulop:
Bài ChoP x/ D x2 2
x2 3
x2 6
: Chứng minh với số nguyên tố p; tồn số nguyênksao chopjP k/ :
Bài Xét đa thứcP x/ D x3C14x2 2xC1:Chứng minh tồn số tự nhiên nsao
cho với mọixnguyên
P P P P x////
„ ƒ‚ …
nlần
x mod 101/ :
Bài Chứng minh không tồn số nguyên dươnga; b; c cho 3.aba2CCbbc2CCcca/2 số nguyên
Bài (Iranian TST 2013) Chứng minh không tồn số nguyên dương a; b; c choa2Cb2Cc2chia hết cho2013 abCbcCca/ :
Bài 10 Tìm tất số nguyên dươngnsao cho.2n 1/j.3n 1/ : Bài 11 Tìm số nguyên dươngm; nthỏa mãn7n Dm2CmC1:
(84)Bài 13 Tìm ước nguyên tố nhỏ của122 C1:
Bài 14 Chứng minh phương trìnhx2 Dy3 5khơng có nghiệm nguyên.
Bài 15 (Sierpinski) Chứng minh không tồn số nguyên dươngn > 1thỏa mãnnj 2n 1C1:
Bài 16 Cho ba số nguyên a; b; c: Chứng minh a; b; c; abc khơng số phương tồn vô số số nguyên tốpsao chopaDpb
Dpc
:
Bài 17 (Polish MO 2013) Cho số nguyêna; bsao cho3CaCb2chia hết cho6a:Chứng minh rằngaâm
Bài 18 (Taiwanese MO 1997) Chok D 22n
C1vớinlà số nguyên dương Chứng minh klà số nguyên tố khikj3k21 C1
:
Bài 19 (IMO Shortlist 1998) Tìm số nguyên dương n cho tồn số nguyên m để 2n 1/j m2C9
:
Bài 20 Choa; blà hai số nguyên dương thỏa mãnabkhơng số phương Chứng minh tồn vô số số tự nhiênnsao cho.an 1/ bn 1/khơng số phương
Bài 21 Cho cặp số nguyên dương m; n/ thỏa mãn 5m 1/ D 5n 1:Chứng minh rằng
gcd.m; n/ > 1:
Bài 22 (Bulgarian MO 1998) Chom; nlà số nguyên dương thỏa mãnAD mC3m3/nC1 số nguyên Chứng minh rằngAlẻ
Bài 23 Chứng minh rằng23n C1có nhấtnước ngun tố có dạng8kC3:
Bài 24 (Iranian TST 2004) Cho trước số nguyên tốpvà số nguyên dươngk;chứng minh tồn số nguyên dươngnsao cho
n
p
D
n
Ck p
:
Bài 25 (Korean 1999) Tìm số nguyên dươngnsao cho3j.2n 1/và tồn số nguyênm cho 2n 1
3
ˇ
ˇ4m2C1:
Tài liệu tham khảo
[1] The theory of number, a text and source book of problems, Andrew Adler. [2] Quadratic residue and non-residue, Steve Wright.
[3] An introduction to the theory of numbers, G.H Hardy. [4] Elementary theory of numbers, W Sierpinski.
(85)[6] Chuyên đề thặng dư bình phương, Nguyễn Văn Sơn.
[7] Chuyên đề thặng dư bình phương, Nguyễn Huy Hồng, Trần Hy Đông. [8] Bài giảng Số học, Nguyễn Vũ Lương.
[9] https://artofproblemsolving.com
(86)CÂU CHUYỆN VỀ MỘT BẤT ĐẲNG THỨC
Trần Nam Dũng
Đại học KHTN, ĐHQG thành phố Hồ Chí Minh
Mới đây, nhận tin nhắn qua Facebook với nội dung sau:
Em thưa thầy thầy cho em hỏi kiến thức học trình ơn thi đội tuyển quốc gia liệu có ích sau khơng ạ, nghĩa liệu sau lên đại học có dùng khơng học đam mê để thi quốc gia thơi ạ? Tại em có hỏi số anh chị trước Đại học Bách khoa anh chị có nói kiến thức tốn cổ điển sau lên đại học khơng cịn dùng lại Nhưng mà em cảm thấy khó mà chấp nhận ạ, chả lẽ công học bục mặt học cao siêu để kiếm giải quốc gia mà sau chả để làm ạ?
Nhận thấy vấn đề quan trọng, không với bạn học sinh mà cịn với nhiều bạn vất vả ơn luyện để thi học sinh giỏi quốc gia, ghi lại câu chuyện chúng tơi Có Tốn tiếp nối mà Anh chị nói thiếu hiểu biết
Liệu thầy nói cụ thể chút khơng ạ?
Tốn học đại học lại đại số, giải tích, số học, tổ hợp, hình học mà Tùy chuyên ngành học khác Tại em thú thật em cảm thấy khó mà có động lực học nghĩ thứ học khơng giúp ích sau ý Vậy có phải thứ pt hàm, định lý số học sau tiếp tục đào sâu thêm không ạ? Hiển nhiên Định lý Trung hoa số dư, định lý Euler, đồng dư ứng dụng ầm ầm
Rồi định lý giải tích, hình học giải tích
Dạ em cảm ơn thầy Em nghĩ em có thêm động lực để học hành chăm Okie Theo dõi facebook thầy có kỹ vấn đề
Thật tình cờ, sau tơi lên facebook thấy có Lê Quang Nẫm, học trò cũ thời PTNK, giáo sư toán Indiana, mở đầu câu đố thú vị bất đẳng thức sơ cấp Và phần giải đố thật bất ngờ kết tảng Garding Chern Nẫm dẫn dắt tốt nên việc chép lại đăng
Tên Bất Đẳng Thức gì?
(87)Bài toán GọiGlà tập hợp bốn số.x; y; z; w/sao cho:
xCyCzCw > 0; xyCxzCxwCyzCywCzw > 0; xyzCxywCxzwCyzw > 0:
Xét hai bộ.x; y; z; w/và.a; b; c; d /thuộcG:Chứng minh rằng:
.aywCazwCayzCbxzCbxwCcywCcxyCcxwCdxyCdxzCdyz/3
>27.xyzCxywCxzwCyzw/2.abcCabd Cacd Cbcd /:
Các câu hỏi là:
(1) Bất đẳng thức chứng minh lần đầu tiên? (2) Nó xuất đâu?
(3) Ảnh hưởng tốn học?
1 Bất đẳng thức Garding, Tạp chí Tốn Đại học Indiana, và Chern
Lars Garding.1919 2014/hoàn thành luận án lý thuyết biểu diễn nhóm năm1944dưới
hướng dẫn Marcel Riesz
Sau viết vài công trình liên quan đến luận án, Garding chuyển sang nghiên cứu phương trình đạo hàm riêng hyperbolic mà phương trình truyền sóng ví dụ Một học trị xuất chúng Garding Lars Hormander 1931 2012/;chủ nhân giải Fields năm1962: Garding
được mời báo cáo toàn thể Đại hội Toán học giới năm1958:
Năm1959;Garding có gửi đến tạp chí tốn học Đại học Indiana, Indiana University
Math-ematics Journal (IUMJ) (lúc cịn mang tên Journal of Mathematics and Mechanics, trang
web tại:https://www.iumj.indiana.edu) báo ngắn bất đẳng thức cho đa thức hyperbolic
Thành viên ban biên tập xử lí Garding lúc nhà hình học lừng danh Shiing-Shen Chern 1911 2004/: Lúc Chern làm việc cho Đại học Chicago Chắc thấy
Garding hay nên Chern dùng kết Garding để chứng minh tính siêu mặt lồi nhiều chiều mà trước năm1938Aleksandrov-Fenchel-Jessen chứng
minh hai chiều Cuối Garding nhận vào IUMJ Kết là: (1) Hai Garding Chern xuất số IUMJ
(2) Bài Chern (trang 947-955) trước Garding (trang 957-965)
(88)(3) Chern trích dẫn Garding; Garding có nói việc Chern dùng kết khơng trích dẫn
Chúng ta đối chiếu tồn văn hai báo liên quan [1], [2]
Bất đẳng thức Garding đa thức hyperbolic cơng cụ nhiều ngành tốn học, từ Giải tích, Phương trình Đạo hàm riêng, Lý thuyết tối ưu, Hình học lồi, Tổ hợp, Chúng ta xem vài trích dẫn tiêu biểu Google Scholar:
https://scholar.google.com/scholar?hl=en&as_sdt=0%2C15&q=hyperbolic+ polynomials&oq=hyperbolic+pol
https://scholar.google.com/scholar?cites=16253416580996898076&as_ sdt=800005&sciodt=0,15
Bài Filipe Rincon, Cynthia Vinzat Josephine Yu (đều cựu IMO/Putnam) dùng đa thức hyperbolic [3] Bài cũ dùng bất đẳng thức Garding [4]
Một trường hợp cụ thể đa thức hyperbolic đa thức đối xứng Pm bậc m n biến số x D.x1; : : : ; xn/W
P1.x/Dx1Cx2C Cxn;
P2.x/Dx1x2Cx1x3C Cx1xnCx2x3C Cx2xnC Cxn 1xn;
Pn.x/Dx1x2 xn:
Chúng ta cố địnhn(trong phần mở đầu,nD4; mD3/W
GọiGmlà tập bộnsốx D x1; : : : ; xn/sao cho giá trị củaP1; : : : ; Pmtạix dương
Có thể thấy khim > 1;có nhiều sốx D.x1; : : : ; xn/trongGmvới nhiều sốxi âm
Ta “trộn”y D y1; : : : ; yn/vào Pm.x1; : : : ; xn/ sau: Với i D 1; 2; : : : ; n;
ta thay yi vào vị trí củaxi trongPm.x1; : : : ; xn/(các biến khác giữ ngun xố số
hạng khơng chứaxi/:Sau cộng hết lại, ta biểu thứcQm.y; x/:
Dễ thấyQm.x; x/DmPm.x/:Nếu bạn biết đạo hàm riêng phần thấyQm.y; x/D lấy tổng theoi từ1tớincủa tích (yi nhân với đạo hàm riêng phần củaPmtheo biếnxi)
Bất đẳng thức Garding phát biểu:
Nếux; ylà hai số trongGmthì ta ln có
.Qm.y; x//m >mmPm.y/.Pm.x//m 1:
Dấu đẳng thức xảy có số dươngc choy Dcx;tức làyi Dcxi; 8i:
Cách chứng minh dùng tính lõm hàm số.Pm/1=mtrênGm:Chi tiết có báo
(89)Tài liệu tham khảo
[1] http://www.iumj.indiana.edu/IUMJ/FULLTEXT/1959/8/58060
[2] http://www.iumj.indiana.edu/IUMJ/FULLTEXT/1959/8/58061
[3] https://arxiv.org/pdf/1907.08545.pdf
[4] https://arxiv.org/pdf/1212.6696.pdf
(90)CÁC BÀI TOÁN TỔ HỢP LIÊN QUAN SỐ HỌC
Lê Phúc Lữ
(Cao học Khoa học tự nhiên TPHCM)
TÓM TẮT
Trong viết này, tác giả giới thiệu số tốn tổ hợp có dùng định lý, công thức liên quan đến số học như: số mũ đúng, tính tổng ước,
1 Kiến thức cần nhớ
Các tính chất sau xét với số nguyên dươnga, b, nvàplà số nguyên tố
1 vp(ab) = vp(a) +vp(b), vp(an) =n·vp(a)
2 vp(ab) =vp(a)−vp(b)nếub |a
3 vp(a)≥vp(b)với số nguyên tốpkhi khib|a.Tính chất cho ta hướng
tiếp cận định lượng cho tốn chia hết thơng thường
4 vp(a)chẵn với số nguyên tốpkhi khinlà số phương
5 Cơng thức Legendre để tính số mũ củaptrongn!là
vp(n!) =
+∞
X
k=1
n pk
Tiếp theo số công thức hàm số học Chon =pa1
1 p
a2 p
ak
k vớip1, p2, , pklà số nguyên tố số mũa1, a2, , aknguyên
dương Khi
1 Tổng số ước củanlàσ(n) =Q(1 +pi+· · ·+paii)với tích lấy ước củan
(91)2 Các toán chọn lọc
Bài 1. GọiAlà tập hợp ước dương của3010.Hai sốx, y ∈ A gọi liên kết với nếu tồn tạik ∈Z+sao choxy|(xk+yk).Hỏi có cặp có tính thứ tự, không thiết phân
biệt(x, y)liên kết với trongA?
Lời giải. Ta chứng minh điều kiện cần đủ để có hai sốx, y liên kết làx, ycó tập ước
nguyên tố Thật vậy, chiều thuận hiển nhiên vìa|bk và b|ak chứng tỏa, bkhơng thể có ước
ngun tố riêng; chiều đảo cần chọnk đủ lớn để vớiplà ước nguyên tố củaabthì vp(ab) =vp(a) +vp(b)≤kmin{vp(a), vp(b)} ≤vp(ak+bk)
Xét có mặt ước nguyên tố2trong hai sốa, b:
1 Nếu chúng có số mũ0thì có1cách chọn
2 Nếu chúng có số mũ lớn hơn0thì số có10cách chọn nên tổng cộng có102+1 = 101cách
Do ước nguyên tố2,3,5độc lập nên ta đếm có tất (102+ 1)3 = 1013
cặp liên kết Ngoài cách xử lý trên, ta đếm thủ cơng cách chia nhỏ trường hợp
Bài 2. (USA MTS 2015) Cho số nguyên dươngn≥3 Người ta xếpnsố nguyên dương
trên đường trịn cho: i Tích hai số khơng nằm cạnh chia hết cho2015·2016
ii Tích hai số nằm cạnh khơng chia hết cho2015·2016 Tìm giá trị lớn củan
Lời giải. Ta có
2015 = 5·13·31,2016 = 25·32·7
ĐặtM = 2015·2016vàN ={2,3,5,7,13,31}
Giả sử tồn tạinsố nguyên dươngx1, x2, x3, , xnxếp vòng tròn thỏa mãn tính chất nêu,
quy ướcxn+1 ≡x1 vàxn+2 ≡x2 Doxixi+1không chia hết choM vớii = 1, n nên tồn số nguyên tốpi ∈N tương ứng mà
vpi(xixi+1)< vpi(M)⇔vpi(xi) +vpi(xi+1)< vpi(M) (∗) Chú ý sốpi trùng Ta chứng minh rằngpi 6=pj vớii, j hai số
không kề Thật vậy, giả sử tồn tạii, j không kề cho pi = pj =p(don ≥ 4nên
tồn sối, j thế) Đặta =vp(M)thì theo (*), ta có
(92)Suy
vp(xi) +vp(xi+1) +vp(xj) +vp(xj+1)<2a (1) Mặt khác, do(i, j),(i+1, j+1)là cặp số không kề nênvp(xi)+vp(xj)≥a, vp(xi+1)+
vp(xj+1)≥a Suy
vp(xi) +vp(xi+1) +vp(xj) +vp(xj+1)≥2a (2) Ta thấy (1), (2) mâu thuẫn nên nhận xét chứng minh
Tiếp theo, giả sử tồn i mà pi = pi+1 = p (với số pi, pj định nghĩa trên) Nếu
vp(M) = 1thì cặpxixi+1, xi+1xi+2đều khơng chia hết choM nên dẫn đếnxi, xi+1, xi+2 không chia hết chop Suy raxixi+2 M, mâu thuẫn vìxi, xi+2 khơng kề Do đó, ta phải cóvp(M)≥2 Từ suy ra, dãy sốp1, p2, , pnvớipi ∈N, số nguyên tố
xuất không lần xuất lần bình phương phải ước củaM
Nói cách khác,M chia hết cho tíchp1p2 pn
DoM = 25·32·5·7·13·31nên số lượngnlớn có thể, ta cho số xuất lần, số xuất lần, ước nguyên tố lại xuất lần, tức có tổng cộng số Ta lại chọn số sau:
M
25,
M
2·3,
M
32,
M
3·5,
M
5·7,
M
7·13,
M
13·31,
M
31·2
tương ứng với dãy số nguyên tố
p1 = 2, p2 = 3, p3 = 3, p4 = 5, p5 = 7, p6 = 13, p7 = 31, p8 = Rõ ràng dãy số thỏa mãn điều kiện cho Vậy giá trị lớn củanlà
Bài 3. (Chuyên Sư phạm HN, 2014) Xâu nhị phân dãy liên tiếp ký tự0,1được xếp
thành hàng, cịn bậc số xâu nhị phân khơng trùng với có số lượng ký tự0,1với
Ví dụ: xâu nhị phân001có bậc2vì có hai xâu nhị phân100,010có số lượng0,1với nó.
Hỏi có cách xếp 66số0và 33số1thành dãy cho với mọik = 1,99thì xâu
nhị phân tính từ vị trí thứ1đến vị trí thứk dãy có bậc chẵn?
Lời giải. Sử dụng định lý Legendre, ta chứng minh công thức mạnh định giá p-adic Cụ thể làvp(n!) = n−ps−p1(n) vớisp(n)là tổng chữ số củantrong hệpphân Khi đó,
vp(Cnk) = vp(n!)−vp(k!)−vp((n−k)!)
= n−sp(n)
p−1 −
k−sp(k)
p−1 −
n−k−sp(n−k)
p−1 = sp(k) +sp(n−k)−sp(n)
p−1
(93)Trở lại tốn, ta thấy xâu nhị phân cóasố0vàbsố1thì có bậc làCa
a+b−1(chính
là số cách xếp số 0,1lên đường thẳng) Do đó, ta cần xếp số0,1phù hợp
cho tính từ đầu dãy đến vị trí thứk, cólsố0thìCl klà lẻ
Nhận xét 1:64số 0.Thật vậy, trong64số có cả0lẫn thìCi
64 số chẵn, khơng thỏa Do đó,64đầu tiên phải giống nhau, là0
NNhận xét 2:32số là1.Tương tự trên, ý rằng25|96nên có số0rơi vào giữa32số từ vị trí65→96thì ta có hai sốC65
96 hoặcC9666đều chẵn
Nhận xét 3: ba số cuối là001.Ta cịn ba vị trí với ba số 0,0,1và dễ dàng kiểm tra
rằng xếp theo hai cách là1,0,0thì dẫn đếnC65
98 số chẵn, khơng thỏa Vậy có cách xếp thỏa mãn đề
Bài 4. (TPHCM 2017) Cho số nguyên dươngn chẵn bảng hình chữ nhậtn cột, nhiều
hàng điền số thỏa mãn đồng thời điều kiện sau: i) Tổng tất số hàng
ii) Mỗi hàng gán bậc avà hàng, ta điền số là0vớia
(khơng có ràng buộc vị trí số hàng) Khơng có hai hàng có bậc
iii) Khơng thể thêm hàng vào bảng mà i), ii) thỏa mãn
Chứng minh bảng có lẻ số0thì số lẻ lớn bảng phải số phương
Lời giải. Giả sử hàng có bậc làa, có tất cảbsốađược điền tổng số hàng
làn=abhayalà ước củan.Theo điều kiện ii, iii) số hàng bảng số ước dương
củan
Gọiσ(n), f(n)lần lượt tổng ước dương củanvà số ước dương củan.Số số khác0
trên hàng có bậc bằngalà n
a nên số số bằng0trên hàng làn− n a
Tổng số0trên bảng
X
a|n
n−n a
=X
a|n
(n−a) = n·f(n)−X
a|n
a=n·f(n)−σ(n)
Don chẵn nênσ(n)phải lẻ Đặtn = 2k·qvới qlà số lẻ thì qchính ước lẻ lớn của n,
cũng số lớn có bảng Tổng ướcσ(n)củancó dạng Q
pαkn
σ(pα).
1 Nếup= 2thì rõ ràngσ(2k)ln số lẻ.
(94)Từ suy tất ước nguyên tố lẻ n có số mũ chẵn Từ suy qlà số
phương, ta có đpcm
Bài 5. (IMO Shortlist 2016) Cho số ngun dươngn > 1.Xét bảng vng kích thướcr×s
thỏa mãn điều kiện sau:
i) Tất ước nguyên dương n điền 1lần bảng khơng có
trống
ii) Tổng số hàng iii) Tổng số cột
Lời giải. Trước hết, ta có nhận xét rằngr, s >1vì ước phân biệt
Tiếp theo, bảng có sốnnên tất tổng hàng, cột lớn hơnn
Giả sửc ≤ rvà gọi a1, a2, , ar ước lớn hàng1,2, , r Ta giả sử tiếp ar
số nhỏ thìar khơng vượt q ước lớn thứr củan tất nhiên không vượt
quá n r
Suy tất số cột tương ứng không vượt n
r, cócsố nên tổng số
cột khơng vượt qc· n r ≤r·
n
r =n, mâu thuẫn
Bài 6. (Hàn Quốc, 1997) Cho số nguyên tốp1, p2, , pk phân biệt số tự nhiên bất
kìn1, n2, , nk lớn Chứng minh số cặp số(x, y)không có thứ tự, nguyên tố
cùng thỏa mãn đẳng thức sau
x3+y3 =pn1
1 pn22· · ·p
nk k
sẽ không vượt quá2k+1.
Lời giải. Ta có x3 +y3 = (x+y)x3+y3
x+y
và gcdx+y,xx3++yy3 = dthì d ∈ {1,3} Ta
chứng minh số cặp(x, y)không có thứ tự thỏa mãn khơng vượt q số cách chọnx+y
và x3+y3
x+y Thật vậy:
(
x+y=a
x3+y3 =b(a, b∈Z)⇔
(
y=a−x
x3+ (a−x)3 =b
Ta có
x3+ (a−x)3 =b ⇔x2 −ax+a
3−b
3a =
(95)• Nếu3∈ {/ p1, p2, p3, , pk}thì với mỗipnii thuộc số xx++yy hoặcx+y,tức
có2cách chọn đó, số cặp(x, y)sẽ khơng vượt q2k.
• Nếu3 ∈ {p1, p2, , pk}thì giả sử khơng tính tổng quát, giả sửp1 = Khi đó, với1 < i ≤ k có cách chọn tương tự cho số pni
i vớii = 1,ta có
cách chọn là3n1 thuộc hồn toàn hai số hoặc3thuộc số và3n1−1 thuộc số cịn lại Do đó, trường hợp này, số cặp(x, y)sẽ khơng q2k+1. Do đó, trường hợp, ta có đpcm
Bài 7. (VMO Mock test 2020) Vớiklà số nguyên lớn hơn1, bảng có viết sẵn số 1,2,3, ,10k,10k+
Ở bước, cho phép xóa hai số a, bbất kỳ có sẵn bảng thay f(a, b), f hàm hai biến Gọixlà số nguyên dương thu sau10k bước Chứng minh với
mọi cách chọn cặp số bước
1 Nếuf(a, b) = gcd(2020ab, a2+ 506ab+b2)thìx= 1.
2 Nếuf(a, b) = gcd(a2b2+ 2019, a2+b2+ 2021)thìxkhơng phải số phương. Lời giải.
1) Ta xét trường hợp sau:
• Nếua, bcùng lẻ thìf(a, b)chẵn
• Nếua, bcùng chẵn thìf(a, b)chẵn
• Nếua, bkhác tính chẵn lẻ thìf(a, b)lẻ
Suy số lượng số lẻ giữ nguyên, giảm 2đơn vị; mà ban đầu có lẻ số lẻ nên sốx
cuối phải lẻ Giả sửx >1, gọiplà ước nguyên tố củax
Nếup= 5thì vì5|gcd(2020ab, a2+ 506ab+b2)nên5|a2+ab+b2
Ta có bổ đề quen thuộc sau.Nếup= 3k+ 2là số nguyên tố vàp|a2+ab+b2thìp|a, b.(chứng minh dễ dàng định lý Fermat nhỏ, ýa3 ≡b3 (mod p).)
Từ suy số trước sinh raa, bcũng chia hết cho5,kéo theo tất số ban đầu
chia hết cho5;vô lý Tương tự với101
Cịn nếup 6= 5,101thìp|ab, p|a2 + 506ab+b2 sẽ kéo theop | a, b.Tương tự suy ra vô lý
Vậy nênxlà số lẻ ước nguyên tố lẻ nào, chứng tỏ rằngx=
(96)• Nếu3|a, 3|bthì3-f(a, b)
• Nếu có hai sốa, bchia hết cho3thì3|f(a, b)
• Nếu3-a, 3-bthì3-f(a, b)
GọiT số lượng số chia hết cho3.Sau bước tính chẵn lẻ T không đổi, mà ban đầu T = 10
k−1
3 lẻ nênxchia hết cho3 Ngồi ra,xkhơng chia hết cho9vì ngược lại, tồn
tạia, bsao chox=f(a, b)|a2b2+ 2019nên2019chia hết cho9, điều vơ lí. Vậy nênxkhơng phải số phương
Bài 8. (Đề nghị HSG TPHCM 2019) Xét tam giác Pascal có 2019dịng bố trí theo
dạng tam giác với số1≤k ≤2019thì dịng thứkcó đúngksố Biết ngoại trừ
các số dòng số tổng hai số kề với hàng liền GọiSlà số đỉnh tam giác vàa0, a1, , a2018là số dòng
(a) Chứng minh rằngS =C0
2018a0+C20181 a1 +· · ·+C20182018a2018
(b) Hỏi có cách cho sốa0, a1, , a2018 nhận giá trị0hoặc1đểSchia hết cho
1009?
Lời giải.
(a) Ta chứng minh quy nạp tam giác Pascal cón+ dịng số dịng
cuối làa0, a1, , anthì số dòng
S =Cn0a0+Cn1a1+· · ·+Cnnan
Vớin = 1, ta thấy tam giác có hai dòng hiển nhiên
S =a0+a1 =C10a0+C11a1
Giả sử khẳng định vớin.Ta thêm vào bên tam giác Pascal cón+ 1dịng dịng
mới gồmn+ 2số làb0, b1, , bn+1 Khi đó, theo giả thiết quy nạp
S =Cn0a0+Cn1a1+· · ·+Cnnan=Cn0(b0+b1) +Cn1(b1+b2) +· · ·+Cnn(bn+bn+1) Với1≤i≤nthì hệ số củabitrongSsẽ làCni−1+Cni =Cni+1nên từ suy
S =C0
n+1b0+Cn1+1b1+· · ·+Cnn+1+1bn
Theo nguyên lý quy nạp khẳng định Thayn= 2018, ta có điều phải chứng minh
(b) Vớiplà số nguyên tố lẻ, ta có tính chất sau:
(97)ii) Ck
2p ≡0 (mod p)với1≤k ≤2p−1vàk 6=p
i) Ta có:
C2pp−2 = (2p)!
p!p! −2 =
(p+ 1)(p+ 2) .(2p−1)(2p)
p! −2
= 2(p+ 1)(p+ 2) .(2p−1)−(p−1)! (p−1)!
Tử số đồng dư với1·2·3·(p−1)−(p−1)! = 0theo modulopnênC2
2p ≡2 (mod p)
ii) DoCk
2p =C
2p−k
2p nên ta giả sử1≤k < p.Biến đổi tương tự
C2kp = (2p−k+ 1)(2p−k+ 2) .(2p−1)(2p)
k!
Ta thấy tử số chia hết choptrong khi(p, k!) = 1vì1≤ k < pnên mẫu số không chia hết cho p, tức biểu thức chia hết chop
Từ ta thấy rằng, để1009|S thìa0+a2018+ 2a1009 phải chia hết cho 1009.Điều xảy tất số bằng0.Các giá trị cịn lại ta chọn tùy ý là0hoặc1nên có tất 22016 cách.
Bài 9. (Nữ sinh Trung Quốc 2012) Mỗi bảng hình chữ nhậtn×4được điền số 1và
−1.Biết tổng số hàng cột bằng0.Gọif(n)là số cách điền thỏa mãn
(a) Tínhf(4)
(b) Chứng minh với mọiplà số nguyên tố lẻ thìf(2p)≡6 (mod 2p)
Lời giải. Ta nhận xét cách điền số1,−1ở cột bảng có
dạng là:
XXOO, OOXX, XOXO, OXOX, XOOX, OXXO
Gọi số cột có dạng làa, b, c, d, e, f với tổng là2p.So sánh số lượng số0ở
hàng, ta có hệ:
a+c+e =b+d+f a+d+f =b+c+e a+d+e=b+c+f a+c+f =b+d+e
Từ hệ dễ dàng suy raa=b, c=d, e =f.Số cách điền có dạng
M = X
a=b,c=d,e=f,a+b+c+d+e+f=2p
(2p)!
a!b!c!d!e!f! =
X
a+c+e=p
(2p)! (a!c!e!)2
(a) Ta cóf(4) =Pa+c+e=2 (a!c24!e!)2 Xét trực tiếp trường hợp (một số bằng2và hai số
(98)(b) Dopnguyên tố nên có số (trong ba sốa, c, e) bằngpvà hai số bằng0thì phân
số khơng chia hết chiap,cịn lại chia hết chop
Do đó, theo modp,sốM đồng dư với3C2pp Bằng cách khai triển trực tiếp, dễ thấyC2pp−2chia
hết chopnên ta cóM ≡6 (mod p), ngồi dễ thấyM chẵn nên M đồng dư6 (mod 2p)
Bài 10. (Dựa theo IMO Shortlist 2001) Một bộ5số nguyên dương có thứ tự(m, n, p, q, r)được
gọi “tốt” chúng số hạng liên tiếp cấp số cộng có cơng sai khơng chia hết cho5
1 Hỏi từ100 số nguyên dương đầu tiên, ta cần chọn số để chắn
rằng đó, ln tồn tốt?
2 Hỏi dãy(a1, a2, , a100)gồm100số nguyên dương (không thiết phân biệt), chọn nhiều tốt? Hai xem khác có phần tử có số thứ tự khác tất số thứ tự phần tử
Lời giải. Gọiklà công sai cấp số cộng đề rõ ràng(k,5) = Ta có
xb =xa+k, xc =xa+ 2k, xd=xa+ 3k, xe =xa+ 4k
Suy số lập thành hệ thặng dư đầy đủ modulo5,tức có số chia5và
có số dư là0,1,2,3,4
(a) Trong số nguyên dương → 100,ta chọn tất số chia 5dư 0,1,2,3thì thu 80số rõ ràng khơng có hệ thặng dư đầy đủ modulo5nào tạo thành nên chắn không
có tốt Do đó, cần lấy nhất81số
Ta cần chứng minh lấy81số ln thỏa mãn Thật vậy, số1→100,ta chia
thành20bộ, gồm5số liên tiếp lấy81số, ta bỏ đi19số; tức có
cịn giữ nguyên phần tử nó, rõ ràng tốt Vì nên GTNN cần tìm là81
(b) GọiA, B, C, D, Elần lượt số lượng số dãy chia dư0,1,2,3,4thì
A+B +C+D+E = 100
Khi đó, số bộ5số thỏa mãn đề
m≤ABCDE ≤
A+B +C+D+E
5
5
= 205
(99)GIỚI THIỆU MỘT SỐ ĐỀ THI OLYMPIC ONLINE
Lê Phúc Lữ
(Cao học Khoa học tự nhiên TPHCM) Trần Bá Đạt
(GV THPT Gia Định TPHCM)
TÓM TẮT
Trong thời gian nửa năm qua, với tình hình dịch bệnh khắp giới, nhiều kỳ thi Olympic truyền thống chuyển sang hình thức online để đảm bảo an tồn cho thí sinh Đó vừa thách thức, địi hỏi phải tổ chức kỳ thi công bằng, khách quan, vừa hội, việc tổ chức linh động hơn, thuận tiện Trong viết này, tác giả xin điểm qua tổng cộng10kỳ thi online
1 Cyberspace MC - thi toán học trực tuyến
American Mathematics Competitions kết hợp với AoPS tổ chức CMC - Cyberspace Mathe-matical Competition Đây thi online miễn phí, uy tín, vừa mang tính giải trí vừa thử thách cao, tối đa học sinh nước đăng ký Thí sinh giải tốn vịng ngày
Ngày thi thứ (13/07/2020)
BÀI1 Xét bảng vng kích thướcn×n đường chéo lànơ vng dọc theo
đường chéo từ góc bên trái xuống góc bên phải Xét hình chữLlà bảng vng2×2
bỏ vng góc Tìm tất giá trịn ≥2sao cho phủ bảng hình
chữ Lsao cho đường chéo khơng phủ, cịn lại bảng thuộc
đúng hình chữL
BÀI Cho hàm số f(x) = 3x2 + 1 Chứng minh với số ngun dương n thì tích
f(1)f(2)· · ·f(n)có tối đanước nguyên tố phân biệt
BÀI Cho tam giácABC có AB > BC điểm D di chuyển đoạn BC Xét điểm E
(100)lượt tâm nội tiếp tam giácABDvàACE.Chứng minh đường thẳngIJ qua
điểm cố định khiDthay đổi
BÀI4.Chonlà số nguyên dương lẻ Một số vng bảngn×nđược tơ màu xanh
cho quân vua di chuyển qua tất ô tô xanh cho bước qua vng xanh kề đỉnh với mà đứng Chứng minh quân vua ln thực việc di chuyển với không n2−1
2 bước
Ngày thi thứ hai (14/07/2020)
BÀI5.Có2020số nguyên dương viết lên bảng Mỗi phút, Zuliet xóa hai sốa, b
nào bảng thay số:a+b, a−b, b−a, ab,a b,
b
a Sau2019phút, Zoom
viết số −2020 Chứng minh với2020 số nguyên dương đó, Zoom viết
được số2020bằng cách thực thao tác cách thích hợp
BÀI Tìm tất số nguyên dươngn ≥ 3sao cho: đa giácP giác lồi màn−1
cạnh có độ dài nhau, n−1góc có số đo thìP đa giác
đều
BÀI7.Mỗi vng bảngn×nđược tơ đen trắng Gọiai số ô vuông đen hàng thứ
ivàbi số vng trắng cột thứi.Tìm giá trị lớn tổng n
P
i=1
aibi BÀI8.Choa1, a2, dãy vô hạn số thực dương cho với mọin∈Z+thì
a1+a2+· · ·+an
n ≥
s
a2
1+a22+· · ·+a2n+1
n+
Chứng minh dãy cho
2 Senior MO of USA 2020
(101)Ngày thi thứ (19/06/2020)
BÀI (Zuming Feng) Cho ABC nhọn, cố định với tâm ngoại tiếp O Điểm X thay đổi
cung nhỏ AB (O) CX cắt AB D Gọi O1, O2 tâm hai đường trịn ngoại tiếp tam giác ADX, BDX Tìm tất vị trí củaX để tam giácOO1O2 có diện tích nhỏ
BÀI2.(Alex Zhai)Một khối gỗ hình lập phương kích thước2020×2020×2020được chia thành
các khối lập phương đơn vị Các xà hình hộp chữ nhật kích thước1×1×2020được đặt
bên khối gỗ thỏa mãn
1 Thanh xà khối liên tiếp khối lập phương đơn vị hai đầu xà thuộc hai mặt đối diện khối gỗ (sẽ có tối đa3·20202thanh)
2 Hai thành xà khơng có chung khối lập phương
3 Mỗi mặt1×2020của xà nằm biên khối gỗ kề với mặt
của xà khác
Tìm số xà đặt vào bên khối gỗ
BÀI3 ( Richard Stong and Toni Bluher)Cho số nguyên tố lẻpvàxlà bất thặng dư
phương modp, nghĩa không tồn tnguyên chox−t2 chia hết chop.Ký hiệuAlà tập hợp tất sốasao cho1≤a < pvàa,4−ađều bất thặng dư phương Tính số dư
của tích tất số trongAkhi chia chop
Ngày thi thứ hai (20/06/2020)
BÀI (Ankan Bhattacharya) Giả sử (a1, b1), (a2, b2), , (a100, b100) cặp có thứ tự số tự nhiên phân biệt Gọi N số cặp số (i, j) cho ≤ i < j ≤ 100
|aibj −ajbi|= Xác định giá trị lớn củaN
BÀI5.(Carl Schildkraut)Một tập hữu hạnScác điểm mặt phẳng gọi "xác định"
nếu |S| ≥ 2và tồn đa thức hệ số thựcP(t) có bậc khơng vượt q|S| −2, thỏa mãn
P(x) =yvới mọi(x, y)∈S
Với số nguyên dương n ≥ 2, tìm số nguyên k lớn cho tồn tập hợp có n
điểm phân biệt khơng "tốt" cóktập tốt
BÀI6.(David Speyer and Kiran Kedlaya)Cho số nguyên dươngn ≥2 Gọix1 ≥x2 ≥ ≥xn
vày1 ≥y2 ≥ ≥ynlà2nsố thực0 =x1+x2+ +xn=y1+y2+ +ynvà
1 = x21+x22 + +x2n =y12+y22+ +yn2
Chứng minh rằngPn
(102)3 Junior MO of USA 2020
USAJMO kỳ thi HSG toàn quốc Mỹ dành cho học sinh lớp 10 nhỏ hơn, nhằm tuyển chọn học sinh "Red level" cho trường hè Kỳ thi tổ chức lần vào năm 2010, năm tổ chức online
Ngày thi thứ (19/06/2020)
BÀI1 (Milan Haiman)Cho số nguyên dươngn ≥ Carl cón sách hình chữ nhật xếp
trên kệ, có kích thước dài - rộng đơi phân biệt Ban đầu, sách xếp tăng dần theo chiều dài bước, Carl chọn hai cạnh mà bên trái có chiều rộng nhỏ hơn, chiều dài lớn so với bên phải, đổi chỗ chúng Carl lặp lại thao tác nhiều lần đến không thực Chứng minh q trình dừng lại, đó, sách xếp tăng dần theo chiều rộng từ trái sang phải
BÀI2.(Titu Andreescu and Waldemar Pompe)Choωlà đường trịn nội tiếp tam giác đềuABC
có`là đường thẳng thay đổi tiếp xúc vớiωvà cắt đoạn thẳngBC, CAlần lượt ởP, Q
Tìm quỹ tích điểmRthỏa mãnP R=P AvàQR =QB
BÀI3.(Alex Zhai)Một khối gỗ hình lập phương kích thước2020×2020×2020được chia thành
các khối lập phương đơn vị Các xà hình hộp chữ nhật kích thước1×1×2020được đặt
bên khối gỗ thỏa mãn
1 Thanh xà khối liên tiếp khối lập phương đơn vị hai đầu xà thuộc hai mặt đối diện khối gỗ (sẽ có tối đa3·20202thanh).
2 Hai thành xà khơng có chung khối lập phương
3 Mỗi mặt1×2020của xà nằm biên khối gỗ kề với mặt
của xà khác
Tìm số xà đặt vào bên khối gỗ
Ngày thi thứ hai (20/06/2020)
BÀI 4.(Milan Haiman)Cho tứ giác lồiABCD nội tiếp đường tròn thỏa mãnDA < AB = BC < CD Điểm E, F chọn CD, AB cho BE ⊥ AC EF kBC Chứng minh rằngF B =F D
BÀI (Ankan Bhattacharya) Giả sử (a1, b1), (a2, b2), , (a100, b100) cặp có thứ tự số tự nhiên phân biệt Gọi N số cặp số (i, j) cho ≤ i < j ≤ 100
(103)BÀI (Ankan Bhattacharya) Chon ≥ 2là số nguyên xét P(x1, x2, , xn) đa thứcn
biến hệ số thực Giả sử với bộn số thựcr1, r2, , rn mà có hai chúng
bằng ta có P(r1, r2, , rn) = Chứng minh P(x1, x2, , xn)
viết thành tổng hơnn!đơn thức có dạngcxd1
1 xd22 xdnn, vớic6= 0vàd1,d2, .,dnlà
số tự nhiên
4 Nữ sinh châu Âu
Olympic Nữ sinh châu Âu EGMO Hà Lan tiến hành thi online EGMO có cấu trúc ngày, ngày làm tiếng rưỡi So với APMO đề có hình thức giống đề VN hơn, gần gũi hơn, đẹp thú vị
Ngày thi thứ (17/04/2020)
BÀI1.Cho số nguyên dươnga0, a1, , a3030 thỏa mãn điều kiện
2an+2 =an+1+ 4anvới mọin= 0,1,2, ,3028
Chứng minh có sốa0, a1, , a3030 chia hết cho22020
BÀI2.Tìm tất số thực không âm(x1, x2, , x2020)thỏa mãn đồng thời ba điều kiện i) x1 ≤x2 ≤ .≤x2020;
ii) x2020 ≤x1+ 1;
iii) có hốn vị(y1, y2, , y2020)của(x1, x2, , x2020)sao cho 2020
X
i=1
((xi+ 1)(yi+ 1))2 =
2020 X
i=1
x3i
BÀI Cho lục giác lồi ABCDEF thỏa mãn điều kiện ∠A = ∠C = ∠E ∠B = ∠D =
∠F,đồng thời phân giác góc∠A,∠Cvà ∠E đồng quy Chứng minh
đó, phân giác góc∠B,∠Dvà∠F đồng quy
Ngày thi thứ hai (18/04/2020)
(104)BÀI5.Xét tam giácABC có gócC tù Đường trịnΓngoại tiếp tam giácABC có bán kính R.Giả sử có điểmP nằm đoạn thẳng ABsao choP B =P C vàP A =R.Đường trung
trực củaP BcắtΓtại điểmDvàE.Chứng minh rằngP tâm đường tròn nội tiếp tam
giácCDE
BÀI6.Cho số nguyênm >1.Một dãy sốa1, a2, a3, định nghĩa bởia1 =a2 = 1, a3 = với mọin ≥4thì
an =m(an−1+an−2)−an−3
Tìm tất sốmsao cho số hạng dãy số phương
5 APMO
Olympic châu Á Thái Bình Dương kỳ thi thường niên diễn theo hình thức thi "bán tập trung" nước, năm việc thi online không ảnh hưởng BTC phát đề tiến hành chấm tháng công bố kết đầu tháng vừa qua
Bài 1. GọiΓlà đường tròn ngoại tiếp tam giácABC.GọiDlà điểm nằm cạnhBC.Tiếp
tuyến củaΓtạiAcắt đường thẳng quaD,song song vớiBAtạiE.CạnhCE cắtΓlần
F.Giả sử rằngB, D, F, E nằm đường tròn, chứng minh rằngAC, BF, DE đồng
quy
Bài 2. Chứng minh rằngr= 2là số thựcrlớn thỏa mãn điều kiện sau:
Nếu dãy số nguyêna1, a2, thỏa mãn bất đẳng thức
an ≤an+2 ≤ p
a2
n+ran+1
với số nguyên dươngnthì tồn số nguyên dươngM choan+2 =anvới mọin ≥N
Bài 3. Xác định tất số nguyên dươngk thỏa mãn tồn số nguyên dươngmlà tập
hợpSgồm số nguyên dương cho số nguyênn > mbất kì viết
dạng tổng phần tử phân biệt trongS theo đúngkcách
Bài 4. GọiZ tập hợp tất số nguyên Tìm tất đa thứcP(x) hệ số nguyên thỏa
mãn tính chất sau:
Với dãy số nguyêna1, a2, mà với số nguyên xuất lần dãy, tồn sối < jvà số nguyênkthỏa
ai+ai+1+ .+aj =P(k)
Bài 5. Cho số nguyên dương n ≥ 3.Số1được viếtnlần bảng Phía bảng hai
xơ trống Ở bước, thực xóa hai sốavàbtrên bảng, thay bởi1vàa+bđồng thời thêm
1viên sỏi vào xô thứ nhất,gcd(a, b)viên sỏi vào xô thứ hai Sau số hữu hạn bước, cóscục
(105)6 Cá tháng tư MO
April Fool MO kỳ thi "ăn theo" APMO, viết tắt AFMO; toán lạ đảm bảo tính chun mơn thử thách
Bài 1. Một số nguyên dươngnđược gọi "đẹp" tổng chữ số của2nnhỏ tổng
chữ số 3n đúng đơn vị Có thể thấy rằng1và 3là hai số "đẹp" Liệu cịn có số "đẹp"
nào khác không?
Bài 2. Hàm tâm tam giác phức hàm số cho
f(a, b, c) = P(a, b, c)
Q(a, b, c)
trong P, Qlà đa thức đối xứng có tất hệ số vàdeg(P) = deg(Q) + Nếu
|a| =|b| = |c| = 1thìf biểu diễn điểmX =f(a, b, c)trong mặt phẳng phức Chẳng hạn,
nếuf(a, b, c) =a+b+cthìXlà trực tâm tam giác có tọa độ ba đỉnh mặt phẳng phức a, b, c
Hàm f gọi "đẹp" với cách chọn số phân biệt a, b, cvới |a| =
|b|=|c|, điểmX =f(a, b, c)nằm tam giácABC.Liệu có phải có hữu hạn hàm tâm tam
giác phứcf?
Bài 3. Cho tam giác nhọn khơng cânABC với tâm đường trịn ngoại tiếp làO trực tâm H.TiaOH cắt đường tròn ngoại tiếp tam giácABC lần tạiX,và ảnh củaXqua phép đối
xứng trụcOA, OB, OC làR, S, T.GọiGlà trọng tâm tam giácRST vàP điểm
tiếp xúc đường tròn nội tiếp đường tròn chín điểm RST Liệu X, G, P có thẳng
hàng hay khơng?
Bài 4. Có2020 thành phố số nước A B tạo đường chiều
thành phố theo lượt, Andrew trước (có thể mở đường từ thành phố tới thành phố người chơi muốn) Trong lượt, người chơi tạo đường thành phố khơng phép có nhiều đường đến hay nhiều đường
Khi tạo thêm đường nữa, thành phố bắt đầu giao dịch với cho giao dịch diễn tuyến đường (hai thành phố giao dịch với tồn dãy đường nối hai thành phố với nhau)
Cuối cùng, thành phố chia thành quốc gia cho thành phố giao thương với phải quốc gia, thành phố giao thương với thành phố khác trở thành quốc gia riêng biệt A thắng trị chơi có số chẵn quốc gia, cịn B thắng có số lẻ quốc gia Giả sử người chơi biết cách chơi tối ưu, liệu B thắng hay khơng?
Bài 5. Có hữu hạn hình trịn mặt phẳng Một cặp đường tròn(Γ, γ)được gọi "đẹp"
tồn tam giác nhận Γlàm đường tròn ngoại tiếp,γ làm đường tròn nội tiếp ngược
(106)Bài 6. Một hàm sốf : N → Nđược gọi "đẹp" tồn hàm khác hằngg : N → N
thỏa mãn với số tự nhiênn,ta ln có
f(n) +g(n) = fg(n)(n) +gf(n)(n)
Chú ý rằng, fa(x) = f(f(· · ·f(x)· · ·))
| {z }
aa lầnf
Liệu có tồn hàm f0 : N → N thỏa mãn tính chất sau
• Nếuf(n)≤f0(n)với mọinthìf "đẹp"
• Nếuf(n)> ef0(n)với mọinthìf không "đẹp"
7 USE MO
Đây kỳ thi mang tính “ăn theo” USAMO xịn, nói phiên thử nghiệm cho thí sinh hướng tới kỳ thi USAMO thức Năm 2020 năm có kỳ thi USEMO diễn
Ngày thi thứ nhất.
Bài 1. Cho tứ giác nội tiếpABCD.Một đường trịn có tâm làO,đi quaB, Dvà cắtBA, BC
lần E, F khác A, B, C Gọi H trực tâm tam giác DEF Chứng minh AC, DO, EF đồng quy hai tam giácABC vàEHF đồng dạng với
Bài 2. GọiZ[x]là tập hợp đa thức theoxvới hệ số nguyên Tìm tất hàmθ :Z[x]→
Z[x]thỏa mãn
• Vớip, q ∈Z[x],ta ln cóθ(p+q) =θ(p) +θ(q)
• Với mỗip∈Z[x], pcó nghiệm nguyên khiθ(p)có nghiệm nguyên
Bài 3. GọiGlà lưới vô hạn chứa ô vuông đơn vị Mộtđa giác bàn cờlà đa giác có cạnh nằm lưới Nikolai chọn mộtđa giác bàn cờF yêu cầu bạn tô số ô củaG màu xanh cho mỗiđa giác bàn cờđồng dạng vớiF có nhất1và nhiều nhất2020ơ xanh
Liệu Nikolai chọnF để khiến bạn thực điều hay không?
Ngày thi thứ hai.
Bài 4. Chứng minh với số nguyên tốp,tồn số nguyên dươngnthỏa mãn
(107)Bài 5. Cho đác đềuP vàV tập hợp định củaP.Mỗi điểm trongV tô
ba màu đỏ, trắng, xanh Một tập củaVđược gọi làđẹpnếu số đỉnh tơ màu Một cạnh củaP gọi làsángnếu điểm đầu điểm cuối khác màu Giả sử rằngVlàđẹpvà số cạnhsángtrongP chẵn Chứng minh tồn đường thẳng không qua tất đỉnh củaV chiaV thành hai tập conđẹp.
Bài 6. ChoABC tam giác nhọn khơng cân với tâm đường trịn ngoại tiếpO ba đường cao AD, BE, CF.GọiX, Y, Zlần lượt trung điểm củaAD, BE, CF ADvàY Zcắt tạiP, BE vàZX cắt tạiQ, CF vàXY cắt tạiR
Giả sử rằngY ZvàBCcắt tạiA0,QRvàEF cắt tạiD0.Chứng minh bốn đường
vng góc kẻ từA, B, C, O tớiQR, RP, P Q, A0D0 tương ứng đồng quy
8 OMO Spring
Kỳ thi OMO năm tổ chức lần, vào mùa thu (cuối T10-đầu T11) vào mùa xuân (cuối T3-đầu T4) Đề gồm 30 câu (từ dễ đến khó, khó khủng, hình thức lẫn nội dung), thí sinh tham gia theo hình thức đăng ký team điền đáp số Đề thi đa dạng nhiều mảng kiến thức, nhiều kỹ thuật hay, mới, đáng ý Bên Ban biên tập chọn giới thiệu15câu số
Bài 1. Cho đường thẳngl vàA, B, C nằm trênlthỏa mãnAB = 7, BC = 5.Gọimlà đường
thẳng quaA,vng góc vớil.LấyP nằm trênm.Tìm giá trị nhỏ củaP B+P C
Bài 2. P viết vào giấy năm số tự nhiên liên tiếp xóa số chúng Khi tổng
bốn số cịn lại là153.Tính số làP xóa
Bài 3. Cho hình vng ABCD với độ dài cạnh 16và tâm O.Gọi S nửa đường trịn
đường kínhABnằm ngồiABCDvàP điểm nằm trênS choOP = 12.Tính diện tích
tam giácCDP
Bài 4. Tìm tố nguyên dươngnnhỏ cho không tồn số nguyênx, y cho n =x3+ 3y3
Bài 5. A có 2020 vẽ, vẽ thứ i hình chữ nhật có độ dài ×i, với i = 1,2, ,2020.Tìm số tự nhiên nnhỏ cho ta đặt tất tranh
chiếc bàn kích thướcn×nvà khơng bị chồng lên
Bài 6. Cho hai số nguyên dươnga > b.Tính giá trị nguyên nhỏ biểu thức ab!+1!+1
Bài 7. Một nhà ảo thuật có mũ chứaacon thỏ trắng vàb thỏ đen Nhà ảo thuật
liên tục rút cặp thỏ chọn mũ Gọi cặp thỏ rút hoàn hảo cặp gồm thỏ đen thỏ trắng Biết nhà ảo thuậ gia rút tất thỏ theo đơi (khơng có thỏ rút mình) số cặphồn hảolà2020
(108)Bài 8. Tính số hàmf: {1, ,15} → {1, ,15}thỏa mãn với mọix ∈ {1, ,15}, ta ln
có
f(f(x))−2f(x) +x
15
là số nguyên
Bài 9. Một giá sách gỗ bốn sách giống hệt với số trang là200, 400, 600,
and800 Velma chọn ngẫu nhiên sách giá sách, lật ngẫu nhiêu trang để đọc
và đặt sách lại kệ Sau đó, Daphne chọn ngẫu nhiên sách kệ lật tới trang để đọc Biết Velma đọc trang 122 Daphne đọc trang304, xác
suất để họ chọn chung sách m
n vớim, nlà hai số nguyên tố Tính100m+n
Bài 10. Với số nguyên không âmp, q, r, đặt
f(p, q, r) = (p!)p(q!)q(r!)r
Tìm số nguyên dươngn nhỏ cho với ba (a, b, c) và(x, y, z)các số không âm
thỏa mãna+b+c= 2020vàx+y+z =n, f(x, y, z)chia hết chof(a, b, c)
Bài 11. Tính số cặp số nguyên dương có thứ tự(m, n)thỏa mãn(2m−1)(2n−1)|210!−1.
Bài 12. Tính số số nguyênnthỏa1 ≤ n ≤ 1024thỏa mãn dãy [n],[n/2],[n/4],[n/8],
không chứa bội của5
Bài 13. Vincent có xúc sắc cân bằng6mặt Ban đầu, Vincent tung xúc sắc ghi
lại số chấm lên tờ giấy Sau đó, giây, anh lại tung lại lần Nếu kết lần tung sau khác lần tung trước anh ghi lại kết vào tờ giấy Nếu kết lần tung sau giống với lần tung trước anh dừng lại khơng ghi kết lần cuối Biết lượt tung Vincent là1, gọiE kì vọng kết sau lần tung Biết tồn
các số hữu tỉr, s, t >0thỏa mãnE =r−slntvàtkhông phải lũy thừa số tự nhiên
Nếur+s+t = mn vớim, nlà hai số nguyên tố nhau, tính100m+n
Bài 14. ChoABC tam giác không cân Đường tròn nội tiếp tam giácABCcắt cạnh BC, AC, vàABtại điểmD,E, andF, tương ứng, đường trònA-bàng tiếp tiếp xúc với BC,AC, vàABlần lượt tạiD1,E1, vàF1, tương ứng Giả sử rằngAD,BE, vàCF đồng quy
tạiG, đường thẳng AD1,BE1,CF1 đồng quy tạiG1 Đường thẳngGG1 cắt phân giác góc BAC tạiX Giả sử AX = 1, cos∠BAC = √3−1, vàBC = 8√4
3 Khi
AB·AC = j+kn√m với số nguyênj,k,m, vànthỏa mãngcd(j, k, n) = 1vàmkhông chia
hết cho số phương lớn hơn1 Tính1000j+ 100k+ 10m+n
Bài 15. Với số nguyên dươngi= 2,3, ,2020, đặt
ai =
√
3i2+ 2i−1
i3 −i
Đặtx2, .,x2020 số thực dương thỏax42+x43+· · ·+x42020 = 1− 1010·20201 ·2021 GọiS giá trị lớn
2020 X
i=2
aixi(√ai−2−2.25xi)
và đặtmlà số nguyên dương nhỏ choSm hữu tỉ KhiSm được viết dạng phân số
tối giản, giả sử phân tích tiêu chuẩn mẫu số pα1
1 pα22· · ·p
αk
k với p1 < · · · < pk số
(109)9 Global Quarantine MO
Đây thi online Thụy Sĩ tổ chức, diễn đợt cách ly dịch bệnh vừa qua, giúp hs số nước có hội trải nghiệm nhiều kỳ thi khác bị hủy
9.1 Mức độ khó
Ngày 1.
Bài 1. Cho tam giácABC với tâm nội tiếp I.Đường tròn nội tiếp tam giácABC tiếp xúc với AC AB E, F Gọi lB, lC tiếp tuyến đường tròn ngoại tiếp tam giác
BIC tạiB, C tương ứng Chứng minh tồn đường tròn tiếp xúc vớiEF, lB, lC với
tâm nằm trênBC
Bài 2. Geoff có vơ hạn túi kẹo, cónvị Anh phân phối số kẹo chon đứa bé (một đứa
bé lấy kẹo có hương vị nào) Gọi cách phân phối viên kẹo k−tốt
mọi nhóm k đứa trẻ có k hương vị kẹo khác Tìm tất tập S
{1,2, , n}thỏa mãn cách phân phối kẹo làs−tốt với mọis∈Sthì làs−tốt
với mọis ∈ {1,2, , n}
Bài 3. Ta gọi số nguyên làđặc biệt chứa nhất4phần tử phân hoạch
thành hai tập hợp rời nhau{a, b}và{c, d}sao choab−cd = 1.Với số nguyên dươngn,
chứng minh tập hợp{1,2, ,4n}không thể phân hoạch thànhntậpđặc biệt.
Bài 4. Chứng minh với số nguyên đủ lớn n, tồn n số nguyêna1, a2, , an thỏa
mãn ba điều kiện sau:
• Mỗi sốai bằng−1,0hoặc1
• Có nhất2n/5số tronga1, a2, , ankhác0
• a1/1 +a2/2 + .+an/n=
Ngày 2.
Bài 5. Tìm tất hàmf :Q→Qthỏa mãn
f(x)f(y+ 1) =f(xf(y)) +f(x), với mọix, y ∈Q
Bài 6. Liệu có tồn vơ hạn ba số nguyên dương(a, b, c)thỏa mãn tất ước nguyên tố
củaa! +b! +c!nhỏ hơn2020hay không?
Bài 7. Mỗi số nguyên {1,2, ,2020} tô màu theo cho với số nguyên dươnga, bthỏa mãna+b ≤2020,các sốa, b,vàa+bkhông tô ba màu khác
Xác định số màu tối đa dùng
Bài 8. GọiABC tam giác nhọn không cân với chân đường cao hạ từA, B, C hạ xuống BC, CA, AB làD, E, F.GọiW điểm nằm tam giácABC vàWa, Wb, Wc
lần lượt ảnh đối xứng W qua BC, CA, AB tương ứng Cuối cùng, gọi N I tâm
đường tròn ngoại tiếp tâm đường tròn nội tiếp tam giácWaWbWc Chứng minh
(110)9.2 Mức độ dễ
Bài 1. Tìm tất bốn(a, b, c, d)sao cho đẳng thức
X2+aX+b= (X−a)(X−c) vàX2+cX+d= (X−b)(X−d)
đúng với mọiX
Bài 2. Ngân hàng Zulrich phát hành đồng xu có mặt in chữH mặt in chữT
Alice cónđồng tiền xếp từ trái sang phải Cô liên tiếp thực thao tác
sau: đồng tiền mặtH,Alice chọn số đồng xu liên tiếp (ít xu)
lật chúng lại; trường hợp tất đồng xu mặtT Alice dừng lại Chẳng hạn,
nếun = 3,Alice thực sau:T HT →HT H →HHH →T T H →T T T Alice
cũng chọn cách thao tácT HT →T T T
Với cấu hìnhC ban đầu, gọi m(C)là số nhỏ thao tác mà Alice thực
Chẳng hạn,m(T HT) = 1vàm(T T T) = 0.Với số nguyênn≥1,hãy xác định giá trị lớn
nhất củam(C)
Bài 3. GọiAvàBlà hai điểm phân biệt mặt phẳng GọiM trung điểm củaABvàωlà
đường tròn quaAvàM.GọiT điểm nằm trênωthỏa mãnBT tiếp xúc vớiω GọiX
là điểm khácB cạnhABthỏa mãnT B =T X,và gọiY chân đường cao hạ từAxuống BT.Chứng minh rằngAT vàXY song song
Bài 4. Với số thựcx,ta gọi[x]là số ngun lớn khơng vượt qx.Tìm tất hàm
đi từRvàoRthỏa mãn
f(x+y) = (−1)[y]f(x) + (−1)[x]f(y), với mọix, y ∈R.
Bài 5. Gọi n, k số tự nhiên thỏa mãnk ≤ 2n B C chơi trò chơi sau.
Đầu tiên, B bí mật chọn số tự nhiênxtrong khoảng từ1đếnn C cố gắng đoánxbằng
cách hỏi câu hỏi mô tả sau Trong lượt, C chọn k tập khác
{1,2, , n}và với tập conS, C hỏi "Liệuxcó nằm trongS hay khơng?".B chọn
trongk câu hỏi, đọc câu hỏi lên trả lời xác Tìm tất cặp(n, k)sao cho với
mọi hành động B, C xác định đượcxsau hữu hạn bước
Bài 6. Với số tự nhiên khác1và−1,gọiS(n)là số tự nhiên nhỏ lớn hơn1và ước
n.Đặc biệt,S(0) = 2.Ta định nghĩaS(0) =S(−1) =
Gọi f đa thức khác với hệ số nguyên thỏa mãnS(f(n)) ≤ S(n) với số nguyên
dươngn.Chứng minh rằngf(0) =
10 Fake USAMO
(111)Bài 1. ChoClà số nguyên dương vàa1, a2, a3, dãy vô hạn số nguyên dương thỏa mãn
an+1 = p
a3
n−Can
với mọinnguyên dương Chứng minh tồn số nguyên dươngN thỏa mãnaN =aN+1 =
aN+2 =
Bài 2. Chonlà số nguyên Ankan có1 + + .+ 2nđiểm xếp thành mảng
tam giác hướng lên gồm2n hàng Ta gọi đoạn thẳng nối điểm với hai điểm
phía liên kết
Cho dãya1, a2, , a2ncó tổng lànvàai ≤ivới mọii= 1,2, ,2n, Ankan thách đố Bankan
lấy n điểm cho hàng thứi có điểm bị lấy đi, với i = 1,2, ,2n Khi
Ankansẽ cố gắng lấyn2liên kết cho điểm kết nối với liên kết Nếu anh ta thành công Bankan loại bỏ điểm nào, trình tựa1, a2, , a2nđược gọi
đẹp Tìm số dãyđẹp.
Bài 3. Cho4ABC không cân với tâm đường tròn ngoại tiếpO, tâm đường tròn nội tiếp I,
đường tròn nội tiếpω Biếtωtiếp xúc vớiBC,CA, vàABtạiD,E, andF GọiT hình chiếu
củaDlênEF Đường thẳngAT cắt đường tròn ngoại tiếp tam giác4ABClần tạiX 6=A
Các đường tròn ngoại tiếp tam giác4AEXvà4AF X cắtωlần tạiP 6=EvàQ6=F
Chứng minh đường thẳngEQ,F P, vàOI đồng quy
Bài 4. Cónhọc sinh lớp học Ban đầu, học sinh có0đồng xu Thầy giáo Evan
đến thực thao tác sau phút: thầy chọn hai học sinh có số xu nhau, giả sử k.Nếuk = 0,thầy cho học sinh1 xu học sinh cịn lại khơng nhận xu Ngược
lại, thầy lấy đồng xu từ bạn chuyển qua bạn khác
Đến thời điểm, Evan thực thao tác Chứng minh thời điểm đó, học sinh giữ số xu là0,1,2,3, , n−1theo trật tự
Bài 5. Xác định tất hàm không bị chặnf : Z → Zthỏa mãn với số nguyên a, b, c
lập thành cấp số cộng, hoán vị củaf(a), f(b), f(c)cũng cấp số cộng
Bài 6. GọiZ+ là tập hợp tất số nguyên dương Giả sửf : Z+ → Z+thỏa mãn hai điều kiện sau:
• Với dãy hữu hạn số nguyên dươnga1, a2, , ak, ta ln có
f(a1) +f(a2) + .+f(ak)chia hếtf(a1+a2+ .+ak)
• Tồn số nguyên dươngmthỏaf(m)6=mf(1)
Chứng minh tồn số nguyên dươngnthỏa mãn
(112)MỘT SỐ BÀI TOÁN PHƯƠNG TRÌNH HÀM
TRONG ĐỀ THI OLYMPIC TỐN
Võ Quốc Bá Cẩn GIỚI THIỆU
Trong viết này, tác giả xin giới thiệu toán tác giả sử dụng giảng Phương trình hàm Gặp gỡ Tốn học2020:
Bài tốn Tìm tất hàm sốf WR!Rthỏa mãn f xCyf x/
Cf y f x/
D2xf y/; 8x; y 2R: 1/
Lời giải Đặta D f 0/:Nếua D 0thì cách thayx D 0vào.1/; ta đượcf y/ D 0với mọiy 2Rvà hàm sốf x/D0thỏa mãn yờu cu ca bi toỏn
Xột trng hpaÔ0:Thayx D0vo.1/;ta
f ay/Cf y a/D0; 8y 2R: 2/
Thayy D0vào.1/;ta có
f x/Cf f x/
D2ax; 8x2 R: 3/
Từ.3/;ta dễ thấyf đơn ánh vàf a/D a:ThayxD avào.1/;ta f a.yC1/Cf yCa/D 2af y/; 8y 2R: Mặt khác, từ.1/;ta cóf a.yC1/D f y a 1/:Do
f yCa/ f y a 1/D 2af y/; 8y 2R: Thayy D a 12 vào phương trình trên, ta đượcf a 12
D0:Tiếp tục, thayx D a 12 vào phương trình 3/; ta đượca D 2a aC 12
; suy a D (doa Ô 0) v f 12 D 0: ThayaD 1và thayy 12 yvào phương trình.2/;ta
f 1
2 Cy
Cf 1
2 y
D0; 8y 2R: Bây giờ, thayy D 12 vào phương trình.1/và sử dụng kết trên, ta
f
xC 2f x/
D f
2 f x/
Df
1
2 Cf x/
(113)Dof đơn ánh nên từ đây, ta cóxC 12f x/ D 12 Cf x/hayf x/ D 2x 1:Thử lại, ta thấy thỏa mãn
Vậy có hai hàm số thỏa mãn yêu cầu đề làf x/D0vàf x/D2x 1:
Bài tốn Tìm tất hàm liên tụcf WR!Rthỏa mãn
f xCy/Cf xy/Df xyCx/Cf y/; 8x; y 2R: 1/
Lời giải Thayx yCx1 vớix; y >0vào phương trình.1/;ta f x/Cf y/Df
xy yC1
Cf
x
yC1 Cy
; 8x; y >0: 2/ Cố địnhx; y >0:Xét hai dãy.xn/; yn/được xác định bởix0 Dx; y0 Dyvà
xnC1D xnyn
ynC1; ynC1 D xn
ynC1 Cyn; 8n2N:
Khi đó, dễ thấyxnCyn D xCy với mọin:Phương trình xác định dãy.yn/có thể viết lại thành
ynC1 D
xCy yn ynC1 Cyn:
Từ đây, quy nạp, ta dễ dàng chứng minh được06yn 6xCyvới số tự nhiênn:Từ suy dãy.yn/khơng giảm bị chặn bởixCy;do tồn giới hạn hữu hạn limynDL: Chuyển phương trình xác định dãy sang giới hạn, ta đượcLDxCy:Vậy limyn DxCy;suy limxn D0:
Mặt khác, từ phương trình.2/;dễ thấyf x/Cf y/ Df xn/Cf yn/với số tự nhiênn: Chon! C1với ý tính liên tục hàm sốf;ta
f x/Cf y/ Df xCy/Cf 0/; 8x; y >0:
Dof liên tục nên từ đây, ta dễ dàng suy raf x/DaxCbvới mọix >0;trong đóa; blà số thực Bây giờ, thayy D 1vào phương trình cho, ta
f x/Cf x 1/DbCf 1/; 8x2R:
Trong phương trình này, chox >1;ta đượcf x/D axCaCf 1/:Suy raf x/DaxCc với mọix 1;trong đóc DaCf 1/:Tiếp tục, thayy D1vào phương trình.1/;ta f x/Df 2x/ f xC1/CaCb; 8x 2R: 3/ Dựa vào phương trình.3/;ta chứng minh quy nạp theonrằng
f x/DaxCc; 8x
1 2n;
1 2nC1
: 4/
Xét x
1; 12
(114)Bây giờ, giả sử khẳng định đến n n > 0/: Xét x 2nC1;
1 2nC2
; ta có 2x
2n;
1 2nC1
vàxC1>0nênf 2x/D2ax Ccvàf xC1/Da.xC1/Cb: Do
f x/Df 2x/ f xC1/CaCbaxCc:
Nên khẳng định vớinC1:Theo nguyên lý quy nạp, khẳng định với mọin: Từ.4/;ta suy raf x/DaxCc với mọix 2n1C1:Chon! C1;ta đượcf x/ DaxCc
với mọix < 0:Vậyf x/DaxCbvới mọix >0vàf x/DaxCbvới mọix < 0:
Vì hàm số f liên tục điểm nên limx!0 f x/ D limx!0Cf x/; tức b D c: Vậy
f x/DaxCbvới mọix 2R:Thử lại, ta thấy hàm thỏa mãn yêu cầu toán
Bài tốn (Iran, 2018) Tìm tất hàm sốf WR!Rthỏa mãn
f xCy/f x2 xyCy2/Dx3Cy3; 8x; y 2R: 1/
Lời giải Thayy D0vào phương trình.1/;ta
f x/f x2/Dx3; 8x 2R: 2/
Từ đóf 1/D1hoặcf 1/ D 1:Mặt khác, dễ thấy nếuf nghiệm hàm f nghiệm hàm Do đó, không tổng quát, ta cần xét trường hợpf 1/ D1là đủ Bây giờ, thayy D1 xvào.1/;ta
f 3x2 3xC1/D3x2 3xC1; 8x 2R: Vì3x2 3xC1có thể nhận giá trị miền14; C1
nên từ kết trên, ta suy f x/Dx; 8x >
4:
Kết hợp với.2/;bằng quy nạp, ta dễ dàng chứng minh f x/ D xvới mọix > 412n (ntự
nhiên) Từ đó, chon ! C1;ta đượcf x/ Dx với mọix > 0:Từ đó, kết hợp với.2/;ta suy raf x/ D x vi mi x Ô 0:Mt khác, từ.2/; dễ thấy f 0/ D 0: Do đóf x/ D x với mọix2 R:Thử lại, ta thấy hàm thỏa mãn yêu cầu toán
Tóm lại, có hai hàm số thỏa mãn yêu cầu làf x/Dxvàf x/D x:
Bài toán (APMO, 2016) Tìm tất hàm sốf WRC !RC thỏa mãn zC1/f xCy/Df xf z/Cy
Cf yf z/Cx
(115)Lời giải Rõ ràng hàm số cho đơn ánh RC: Thay x D y D z D 1 vào .1/; ta được f 1/D1:ThayxDy vào.1/;ta
.zC1/f 2x/D2f x
f z/C1
; 8x; z > 0: 2/
Thayx D 12 vào phương trình trên, ta f
f z/ C1
D zC1
2 ; 8z > 0: 3/
Suy hàm sốf x/toàn ánh miền 12; C1/
:Kết hợp hai phương trình.2/và.3/;ta f x
f z/C1
Df 2x/f
f z/C1
; 8x; z > 0: Vì f z/C1
2 nhận giá trị miền 4; C1
nên từ kết trên, ta suy f xy/Df x/f y/; 8x > 0; y >
4:
Với số thực dươngx; y;tồn số thựcz > 34 choyz > 34:Sử dụng kết trên, ta có f xyz/
f z/ D
f xy/f z/
f z/ Df xy/;
f xyz/ f z/ D
f x/f yz/ f z/ D
f x/f y/f z/
f z/ Df x/f y/:
Do đóf xy/D f x/f y/với mọix; y > 0;hay nói cách khác, hàmf nhân tính Nói riêng, ta có
f x2/D f x/2
; 8x > 0:
Từ đây, quy nạp, ta dễ dàng chứng minh f x/ nhận giá trị miền
1
22n; C1
với mọintự nhiên Chon ! C1;ta suy hàmf toàn ánh trênRC:Vậyf song ánh
Như vậy, với số thực dươngavàb;tồn số thực dươngz chof z/ < min˚
1; ba; ab tồn hai số thực dương x; y cho xf z/Cy D a; yf z/ C x D b (cụ thể x D b af z/
1 f z/
2; y D
a bf z/ f z/
2) Suy
f a/Cf b/ Df xf z/Cy
Cf yf z/Cx
D.zC1/f xCy/ D2f
f z/ C1
f xCy/D2f
f z/C1
.x Cy/
!
D2f
aCb
:
Nói riêng, ta có2f 2/Df 3/Cf 1/Df 3/C1và2f 3/Df 4/Cf 2/D f 2/2Cf 2/: Từ đây, với ýf song ánh, ta đượcf 2/D2vàf 3/D3:Và
f a/Cf b/ D2f
aCb
D 2f aCb/
(116)Suy hàm f cộng tính trênRC:Kết hợp vớif 1/ D 1;ta suy f x/ D x với x > 0: Thử lại, ta thấy hàm thỏa mãn yêu cầu toán
Bài toán (Korea, 2015) Cho số nguyên lẻn > 1:Tìm tất hàm sốf WR!Rthỏa f xnC f y/nDynC f x/n; 8x; y2 R: 1/
Lời giải ĐặtaDf 0/:Từ giả thiết, ta có
f f xnC f y/nDf ynC f x/nDxnC f y/n; 8x; y 2R: VìxnC f y/n
có thể nhận giá trị trênRnên từ đây, ta suy f f x/
Dx; 8x 2R: Từ đây, thayy bởif y/vào.1/;ta
f xnCyn/D f x/n
C f y/n
; 8x; y 2R: 2/
Thayy D0vào.2/;ta đượcf xn/D f x/nCanvới mọix 2R:Nói riêng, ta cóaD2an: Phương trình.2/có thể viết lại thành
f xnCyn/Df xn/Cf yn/ 2an; hay
f x Cy/Df x/Cf y/ a; 8x; y2 R: 3/ Thayx Dy Davào.3/;ta đượcf 2a/D2f a/ aD2f f 0/
aD a:Tiếp tục, thay x Davày D 2a vào.1/;ta
aDf 0/Df anC f 2a/n D.2a/nC f a/nD.2a/n:
Kết hợp vớiaD2an;ta dễ dàng suy raaD0:Từ suy hàmf cộng tính trênRvà f xn/D f x/n
; 8x 2R
Từ đây, dễ dàng chứng minh có2hàm số thỏa mãn làf x/Dxvàf x/D x:
Bài tốn Tìm tất hàm sốf WRC !RCthỏa mãn f
xCyC xy xCy
Df x Cy/Cf x/
y
xCy
; 1/
với số thực dươngx; y: Ở đâyRCđược ký hiệu tập số thực dương./
Lời giải Đảo vị trí củaxvàytrong.1/rồi đối chiếu với phương trình.1/;ta f x/
f y/ D
f xCxy f xCyy
(117)Từ suy
f x/ f 1/ D
f xCx1
f xC11; 8x >
f xxyCy f xCyy
D f
xy xCy xy xCyC
y xCy
f
y xCy xy xCyC
y xCy
D
f xCx1
f xC11; 8x; y > 0:
Kết hợp lại, ta
f x/f
y xCy
Df 1/f
xy xCy
; 8x; y > 0: Từ đó, phương trình.1/có thể viết lại thành
f
xCyC xy xCy
Df xCy/Cf
xy
xCy
f 1/; 8x; y > 0:
Để ý với số thực dươnga; b màa> 4b;luôn tồn hai số thực dươngx; ysao cho xCy Davà xxyCy Db:Từ đây, kết hợp với kết trên, ta
f aCb/Df a/Cf b/f 1/; 8a; b > 0Wa>4b:
Bây giờ, cố địnhx; y > 0:Ta thấy tồn số thực dươngz choz > 4.xCy/:Khi đó, theo kết vừa chứng minh, ta có
f zCxCy/Df z/Cf xCy/f 1/
f zCxCy/Df zCx/Cf y/f 1/ Df z/Cf x/f 1/Cf y/f 1/: So sánh hai kết quả, ta
f xCy/Df x/Cf y/; 8x; y > 0:
Suy hàmf cộng tính trênRC:Từ đóf x/Dkxvới mọix > 0;trong đóklà số dương Thay trở lại phương trình.1/;ta đượckD1:Vậy có hàm số thỏa mãn yêu cầu đề làf x/Dx:
Bài tốn (IMO Shortlist, 2018) Tìm tất hàm sốf WRC!Rthỏa mãn
xC x
f y/Df xy/Cf y x
.1/ với số thực dươngx; y: Ở đâyRCđược ký hiệu tập số thực dương./
Lời giải Đặtf 1/Da:ThayxDy vào phương trình.1/;ta
xC x
(118)
Thayy bởixy2vào phương trình.1/;ta
xC x
f y2x/ Df x2y2/Cf y2/; 8x; y > 0: 3/ Thayxbởiyvà thayy bởixyvào phương trình.1/;ta
yC
y
f xy/ Df y2x/Cf x/; 8x; y > 0: 4/ Kết hợp kết quả.2/; 3/và.4/;ta
xC
x yC
1 y
f xy/ f x/
D
xy C xy
f xy/C
yC y
f y/ 2a; hay x y C y x
f xy/D
xC x
f x/C
yC y
f y/ 2a; 8x; y > 0: 5/ Bây giờ, đảo vị trí xvà y phương trình.1/ cộng kết vừa thu với.1/; ta
xC
x
f y/C
yC y
f x/D2f xy/Cf
x y
Cf y x
; 8x; y > 0: Mặt khác, cách thay x yx thay y D vào 1/; ta có f xyC f yx D axy C yx:Do đó, kết hợp với kết trên, ta
xC
x
f y/C
yC y
f x/D2f xy/Ca x
y C y x
; 8x; y > 0: Nhân hai vế phương trình với x
y C y
x sử dụng kết quả.5/;ta x
y C y
x xC
1 x
f y/C x
y C y
x yC
1 y f x/ D2 xC
x
f x/C2
yC y
f y/ 4aCa x y C y x ; hay
.y2 1/.y2 x2/
xy2 f x/C
.x2 1/.x2 y2/
x2y f y/D
a.x2 y2/2
x2y2 ; 8x; y > 0: Thay y D 2vào phương trình trên, ta đượcf x/ D mxC xn vi mix > 0; x Ô 2;trong ú m; nlà số Mặt khác, cách thayx D2vào.2/;ta cóf 2/D2mCn2: Do đó, ta ln cóf x/ D mx C nx với mọix > 0:Thử lại, ta thấy thỏa mãn Vậy hàm số thỏa mãn yêu cầu đề có dạngf x/DmxCxn vớim; nlà số thực
Bài toán (Korea, 2020) Tìm tất hàm sốf WQC !Rthỏa mãn f x/Cf y/Cf z/D1
(119)Lời giải Dễ thấyx Cy Cz C1 D 4xyz tương đương với 2x1C1 C 2y1C1 C 2z1C1 D 1: Đặt a D 2x1C1; b D
1
2yC1 c D
2zC1 ta có < a; b; c < 1; aCb Cc D x D a
2a ; y D b2b ; zD c2c :Do đó, giả thiết tốn viết lại thành
f 1 a 2a Cf 1 b 2b Cf 1 c 2c D1 với số hữu tỉ dươnga; b; c thỏa mãnaCbCc D1:
Đặtg x 13
Df x2x
3 ta có g a Cg b Cg c D0
với số hữu tỉ dươnga; b; c thỏa mãnaCbCc D1:Một cách tương đương, ta có
g.x/Cg.y/Cg.z/D0 1/
với số hữu tỉx; y; z2 13; 23thỏa mãnxCyCz D0:
Bây giờ, thayx Dy Dz D0vào.1/;ta đượcg.0/D0:Lần lượt thayx; y xC2y; xC2y vào 1/rồi đối chiếu với phương trình.1/;ta
g x
Cy
D g.x/Cg.y/
2 2/
với số hữu tỉx; y 13;
thỏa mãnxCy 23;
:Nói riêng, phương trình.2/được thỏa mãn với số hữu tỉx; y 13; 16:
Đặth.x/Dg x 121 g 121:Khi đó, với số hữu tỉx; y2 14; 14;ta cóx 121; y
12 thuộc khoảng 3;
1
:Do đó, sử dụng kết quả.2/;ta h x Cy Dg x Cy 12 g 1 12
Dg x
1 12 Cy
1 12 ! g 1 12
D g x 12
Cg y 121
2 g
12
D h.x/Ch.y/
2 :
Rõ ràngh.0/ D0nên từ phương trình trên, ta dễ dàng chứng minh quy nạp để suy h.x/D2nhx
2n
.3/ với số hữu tỉx 14;
1
và với số tự nhiênn:
Bây giờ, với số hữu tỉ x;ta thấy tồn số nguyên dương n cho 2xn
1 4; : Ngồi ra, với số nnày thì2nh 2xn
nhận giá trị (theox) Thật vậy, giả sử tồn hai số nguyên dươngm; nvớim > nsao cho 2xm;
x 2n
1 4;
1
:Khi đó, theo.3/;ta có 2nhx
2n
D2n2m nh
x
2n
2m n
D2mh x 2m
(120)Từ kết suy ta định nghĩa hàmWQ!Rđược xác định bởi.x/D2nh 2xn
; đónlà số nguyên dương thỏa mãn 2xn
1 4;
1
:Hàm.x/được xác định theo cách hàm cộng tính Q:Thật vậy, với hai số hữu tỉx; y cho trước, rõ ràng tồn số nguyên dươngnsao cho 2xn;
y 2n;
xCy 2n
1 4;
1
:Từ đó, ta có xCy/D2nC1h
x Cy 2nC1
D2nhhx 2n
Chy 2n
i
D2nhx 2n
C2nhy 2n
D.x/C.y/: Vì.x/là hàm cộng tính trênQnên.x/Dkx vớiklà số thực Từ suy với số hữu tỉx 14;
1
thìh.x/D.x/Dkx;do với số hữu tỉx 13;
thì g.x/Dh
xC
12 Cg 1 12 Dk
xC 12 Cg 1 12
DkxC`; đó`D 12k Cg 121:Màg.0/D0nên` D0;suy
g.x/Dkx; 8x 2QWx2 3; : 4/
Bây giờ, phương trình.1/;xétx2 6;
2
và thayy D 13 " x; z D
3C"với"là số hữu tỉ thuộc 0; 23 x
;ta g.x/Cg
1
3 " x
Cg
1
3 C"
D0: Với cách chọn"như trên, ta có 13 " x; 13 C"2 13; 16
:Do đó, theo kết quả.4/thì g
3 " x
Cg
1 C"
Dk
3 " x
Ck
1 C"
D kx:
Từ suy g.x/ D kx với số hữu tỉx 6;
2
:Tóm lại, ta có g.x/ D kx với số hữu tỉx 13;
2
:Từ suy ra, với số hữu tỉx 2.0; 1/thì f
1 x
2x
1
3 Dg x Dk x : Thayxbởi 2x1C1;ta đượcf x/D 2xkC1 C
1 k
3 với số hữu tỉ dươngx:Thử lại, ta thấy hàm thỏa mãn yêu cầu toán
Bài tốn (Chinese IMO TST, 2019) Tìm tất hàm sốf WQ!Qthỏa mãn f
2xyC
Cf x y/D4f x/f y/C
2; 8x; y2 Q: 1/
Lời giải Thay y D vào 1/; ta
4f 0/
f x/ D f 12
(121)Bây giờ, trong.1/;lần lượt thayx; y x2; y2;ta f xy C1
Cf x y
2
D4f x
f y
C
2; 8x; y 2Q: Đặtg.x/D4f x2thì từ phương trình, ta có
g.xyC1/Cg.x y/Dg.x/g.y/C2; 8x; y 2Q: 2/ Từ cách đặt, dễ thấyg.0/D1vàg.1/D2:Thayy D1vào.2/;ta
g.xC1/Cg.x 1/D2g.x/C2; 8x 2Q: 3/
Từ 3/;bằng quy nạp, ta dễ dàng chứng minh đượcg.n/ D n2C1với số nguyênn:Tiếp theo, ta chứng minh kết sau
g.xCn/Cg.x n/D2g.x/C2n2; 8n2 N; x2 Q:
Ta sử dụng phép quy nạp theo n: Với n D 0và n D 1; khẳng định hiển nhiên Giả sử khẳng định đếnn n>1/;lần lượt thayxbởixCnvàx nvào.3/;ta
g.x CnC1/Cg.xCn 1/D2g.xCn/C2
g.x n 1/Cg.x nC1/D2g.x n/C2: Từ suy
g.xCnC1/Cg.x n 1/Cg.xCn 1/Cg.x nC1/D2g.xCn/Cg.x n/C4: Mặt khác, theo giả thiết quy nạp, ta cóg.xCn 1/Cg.x nC1/ D2g.x/C.n 1/2và g.xCn/Cg.x n/D2g.x/C2n2:Do
g.xCnC1/Cg.x n 1/D2g.x/C4n2 2.n 1/2C4D2g.x/C2.nC1/2: Từ suy khẳng định vớinC1:Theo nguyên lý quy nạp, ta có khẳng định với mọin:Bây giờ, thayybởinvà n(nnguyên dương) vào.2/;ta
g.x nC1/Cg.x n/D.n2C1/g.x/C2
g x nC1/Cg.xCn/D.n2C1/g.x/C2: Cộng hai phương trình lại thayx D mn vớimnguyên, ta
2.n2C1/gm n
C4Dgm n Cn
Cgm
n n
Cg.mC1/Cg mC1/ D2gm
n
C2n2C.mC1/2C1C.m 1/2C1 D2gm
n
C2m2C2n2C4: Từ suy g mn
D mn
(122)Bài toán 10 (Iran, 2010) Tìm tất hàm sốf WR>0!R>0thỏa mãn f
yC xCf x/
D2x f x/Cf f y/ 1/
với số thực không âmx; y: Ở đâyR>0 được ký hiệu tập số thực không âm./
Lời giải Trước hết, ta chứng minh hàmf đơn ánh Thật vậy, giả sử tồn hai số không âm a; b vớia > b chof a/Df b/:Khi đó, từ.1/;ta có
f
yC aCf a/
D2a f a/Cf f y/
f
yC bCf b/
D2b f b/Cf f y/ : Suy
f
y CaCf a/
Df
yCbCf a/
C2.a b/; 8y >0: Từ đây, ta có
f yCc/Df y/C4c
với mọiy > bCf a/2 ;trong đóc D a b2 > 0:Và thế, quy nạp, ta chứng minh f yCnc/Df y/C4nc
với mọiy > bCf a/2 mọin2N:Bây giờ, thayxbởixC2cvào.1/và xétx; y > bCf a/2 ;ta
f
y CxCf x/
2 C5c
D2x f x/ 4cCf f y/; hay
f
yC xCf x/
C20c D2x f x/ 4cCf f y/ :
Đối chiếu với phương trình.1/;ta được20cD 4c;mâu thuẫn Vậy hàmf đơn ánh Đặtd Df 0/:Thayx Dd vày D0vào phương trình.1/;ta
f d
Cf d /
D2d: 1/
Thayx D0vào phương trình.1/;ta f
yC d
2
Df f y/ d; 8y >0: 2/
Do đó, phương trình.1/có thể viết lại thành f
yC xCf x/
D2x f x/Cd Cf
y Cd
(123)
Thayx D d2 vày D0vào phương trình này, ta f
d Cf
d
2 !
D2d: 3/
Từ.1/và.3/;với ýf hàm đơn ánh, ta suy d2 Cf d /D d2 Cf d
:Mặt khác, từ.2/; ta cóf d2
Df d / d:Do d2 Cf d /Df d / d;hayd D0: Vìd D0nên phương trình.2/có thể viết lại thànhf y/ Df f y/
với mọiy >0:Do f đơn ánh nên từ đây, ta suy raf y/ D y với mọiy > 0:Thử lại, ta thấy hàm sốf x/ D x thỏa mãn yêu cầu tốn
Bài tốn 11 (RMM Shortlist, 2019) Tìm tất hàm sốf WR!Rthỏa mãn
f xCyf x/Cf xy/ Df x/Cf 2019y/; 8x; y2 R: 1/
Lời giải Xét trường hợp sau
Trường hợp 1: f 0/D0:Thayx D 0vào phương trình 1/; ta đượcf 2019y/ D 0với y 2R:Từ đóf x/D 0với mọix 2R:Thử lại, ta thấy hàm thỏa mãn yêu cầu ca bi toỏn
Trng hp 2: f 2019/Ô0:Thayx D2019vo phương trình.1/;ta f yf 2019/C2019Df 2019/
với y R: Vì yf 2019/ C2019 nhận giá trị R nên từ ta suy f x/C vớiC số khác0:Thử lại, ta thấy hàm thỏa mãn yêu cầu toỏn Trng hp 3: f 0/Ô0; f 2019/D0:
Nuf x/D0vi mixÔ0thỡ ta cú f x/D
(
0 nux Ô0 c nux D0;
trong úcl hng số khác0:Hàm thỏa mãn yêu cầu toán
Nuf x/Ô0vi mixÔ0thỡ bng cỏch thayy D1vo phng trình.1/;ta f xCf x/
D0; 8x 2R:
(124)Xột trng hp tn tiaÔ0; b ¤2019sao chof a/¤0vàf b/D0:ThayxDbvào phương trình.1/;ta đượcf by/ Df 2019y/với mọiy 2R:Từ suy
f x/Df kx/
vi mi s thcx;trong úk D 2019b Ô1:Bõy gi, thayxbikav thayy y f a/ vào phương trình.1/;ta
f yCka/Cf
ay f a/
Df
2019y
f a/
Cf a/Df yCa/Cf
ay f a/
: Suy raf yCka/Df yCa/với số thựcy:Từ đó, ta có
f x/Df xCm/
vi mi s thcx;trong úm D.k 1/aÔ0:ThayxbixCmvo phng trỡnh.1/; ta
f xCyf x/
Cf xyCmy/Df x/Cf 2019y/Df xCyf x/
Cf xy/; hay
f xyCmy/Df xy/; 8x; y 2R: Bây giờ, thayx D a bmb y D
a b
m vào phương trình trên, ta đượcf a/ D f b/; mâu thuẫn Do đó, trường hợp khơng xảy
Tóm lại, hàm số thỏa mãn yêu cầu đề
f x/DC; f x/D2019 x; f x/D (
0 nuxÔ0 c nuxD0; úC; cl cỏc hng s thc vc Ô0:
Bi toỏn 12 (IMO, 2011) Cho hàm sốf WR!Rthỏa mãn
f xCy/6yf x/Cf f x/ 1/
với số thựcx; y:Chứng minh rằngf x/D0với mọix 60:
Lời giải ĐặtaDf 0/vàbDf f 0/:Từ giả thiết, cách thayx D0vào.1/;ta có f y/6ayCb; 8y R:
Từ suy
f xCy/6yf x/Cf f x/6yf x/Caf x/CbD.yCa/f x/Cb; 8x; y R: 2/ Ta chứng minhf x/60với mọix2 R:Thật vậy, giả sử tồn số thựccsao chof c/ > 0: Thayx Dcvà thayy bởiy cvào.2/;ta
(125)Vì limy!
.y cCa/f c/CbD 1nên
lim
y! 1f y/D 1: Bây giờ, thayy Dc xvào.2/;ta
f c/6.c xCa/f x/Cb; 8x 2R: Từ đó0 < f c/6limx!
.c xCa/f x/CbD 1;mâu thuẫn Tóm lại, ta có
f x/60; 8x2 R: 3/
Bây giờ, bất phương trình.2/;thayy Da x;ta 2a x/f x/>0; 8x 2R:
Từ đó, với mọix < 2a thìf x/>0:Kết hợp với.3/;ta đượcf x/D0với mọix < 2a: Cuối cùng, ta chứng minh a D 0:Thật vậy, giả sửa < 0:Trong.1/chox < 2avà thay y bởiy x;ta đượcf y/6avới mọiy 2R:Nhưng bất đẳng thức khơng khiy < 2a: Do đóaD0:Từ kết hợp với kết thu được, ta cóf x/D0với mọix 60:
Bài tốn 13 Tồn khơng hàm sốf WR!Rthỏa mãn
f x f y/6yf x/Cx; 8x; y2 R‹ 1/
Lời giải Giả sử tồn hàm sốf thỏa mãn yêu cầu ĐặtaDf 0/;khi từ.1/;dễ thấy
f x/6xCa; 8x 2R: 2/
Thayxbởif y/vào.1/và sử dụng bất đẳng thức trên, ta a6yf f y/
Cf y/6yf f y/
CyCa: Từ suy
y
f f y/ C1
>0; 8y 2R: Trong bất phương trình trên, xéty > 0và sử dụng kết quả.2/;ta
f y/> a 1; 8y > 0: 3/
Bây giờ, cố địnhx2 Rvà xéty < x a:Theo kết quả.2/;ta cóx f y/>x yCa/ > 0: Từ đó, theo kết quả.3/thìf x f y/ > a 1:Sử dụng đánh giá này, ta
a 16f x f y/
6yf x/Cx; 8y < x a: Choy ! 1;ta suy raf x/>0:Tóm lại, ta ln có
(126)Nói riêng, ta cóaDf 0/60:Bây giờ, thayxbởixCf y/vào.1/vớix < 06 acố định vày > 0;ta
f x/6yf xCf y/CxCf y/6y xCf y/Ca/CxCf y/6y.xCa/Cx: Từ suy
y x f x/
.xCa/; 8y > 0:
Bất đẳng thức thỏa mãn y nhận giá trị đủ lớn Vậy không tồn hàm sốf thỏa mãn yêu cầu
Bài toán 14 (Japan, 2020) Tìm tất hàm sốf WN !Nthỏa mãn x2C f y/2C
x f y/2 > f x/2Cy2; 8x; y 2N: 1/
Lời giải Để tiện cho việc trình bày, ta ký hiệu fn.x/ D f f : : : f x/ : : :
(n lầnf) Thay x Df y/vào.1/;ta
2 f y/2> f f y/2Cy2; 8y 2N: 2/ Nếu tồn số nguyên dươngasao > f a/thì từ bất phương trình.2/;ta dễ dàng suy f a/ > f f a/
:Từ đó, quy nạp, ta chứng minh a > f a/ > f f a/
> f3.a/ >
Suy hai số0vàacó vơ số số nguyên dương, mâu thuẫn Do f x/>x; 8x 2N:
Cố địnhx nguyên dương Ta chứng minh tồn số tự nhiênn0sao chofn0C1.x/ Dfn0.x/:
Thật vậy, giả sử ngược lại, với số tự nhiênnthìfnC1.x/ > fn.x/: Thayy bởifn.x/vào.2/;ta được2 fnC1.x/
2
> fnC2.x/
C fn.x/2;hay
fnC1.x/ fn.x/
fnC1.x/Cfn.x/
>
fnC2.x/ fnC1.x/
fnC2.x/CfnC1.x/ : Do
fnC1.x/ fn.x/
fnC1.x/Cfn.x/
< fnC1.x/ fn.x/
fnC2.x/CfnC1.x/
nên từ bất phương trình trên, ta suy
fnC1.x/ fn.x/ > fnC2.x/ fnC1.x/; 8n2 N: Từ kết này, ta có
f x/ x > f2.x/ f x/ > f3.x/ f2.x/ > >0:
Suy giữa0vàf x/ x có vơ số số ngun dương, mâu thuẫn Vậy phải tồn số tự nhiên n0 chofn0C1.x/ D fn0.x/:Bây giờ, trong.1/;thay xbởifn0 1.x/và thayy bởifn0.x/;
ta
fn0 1.x/
2
C fn0.x/
2 C
fn0 1.x/ fn0.x/
2
>2 fn0.x/
2 ; hay
2fn0 1.x/
fn0 1.x/ fn0.x/
>0:
Từ suy rafn0 1.x/ > fn0.x/:Màfn0.x/ > fn0 1.x/nên ta có fn0 1.x/ D fn0.x/:
(127)Bài toán 15 (Iranian IMO TST, 2017) ĐặtRClà tập số thực dương Tìm tất hàm sốf WRCRC!RCthỏa mãn đồng thời điều kiện sau với số thực dươngx; y; zW
f f x; y/; z
Dx2y2f x; z/: .1/
f x; 1Cf x; y/
>x2Cxyf x; x/: 2/
Lời giải Thayy Dz D1vào.1/;ta
f f x; 1/; 1/Dx2f x; 1/; 8x > 0: 3/ Từ đây, dễ dàng suy hàm số f x; 1/ đơn ánh RC: Từ đó, với ýf f 1; 1/; D f 1; 1/;ta suy raf 1; 1/D1:Thayx Dz D1vào.1/;ta
f f 1; y/; 1/Dy2; 8y > 0: 4/
Suy hàm sốf x; 1/có thể nhận giá trị trênRC:Tóm lại, hàm số f x; 1/là song ánh trênRC:
Từ 4/;ta suy hàm số f 1; x/đơn ánh trênRC:Mặt khác, hàm số f x; 1/là song ánh RC nên ứng với số dươngx đều tồn số dươngy sao choy2
D f x; 1/:Kết hợp với.4/;ta đượcf f 1; y/; 1/ D f x; 1/;tứcf 1; y/ Dx:Suy hàm sốf 1; x/cũng tồn ánh trênRC:Tóm lại, hàm sốf 1; x/là song ánh trênRC:
Bây giờ, thayxD1vào.1/;ta f f 1; y/; z
Dy2f 1; z/ Df f 1; y/;
f 1; z/: Vì hàm sốf 1; y/là song ánh trênRCnên từ đây, ta có
f x; y/Df x; 1/f 1; y/; 8x; y > 0: 5/ Đặt g.x/ D f x; 1/và h.y/ D f 1; y/thì từ.5/; ta cóf x; y/ D g.x/h.y/:Các giả thiết 1/và.2/có thể viết lại thành
g g.x/h.y/
Dx2y2g.x/; 8x; y > 10/
g.x/h 1Cg.x/h.y/
>x2Cxyg.x/h.x/; 8x; y > 0: 20/ Rõ ràng g.1/ D h.1/ D nên từ 10/; ta có g g.x/ D x2g.x/ g h.y/ D y2: Do đó, phương trình.10/có thể viết lại thànhg g.x/h.y/
Dg g.x/
g h.y/
:Vìg.x/; h.y/là hàm song ánh trênRCnên từ đây, ta suy raglà hàm nhân tính trênRC:
Với ýx2y2 Dg h.xy/
;từ.10/;ta có
(128)Choy D1;ta đượcg.x/Dxh.x/với mọix > 0:Từ đây, dễ thấyh.x/cũng hàm nhân tính trênRC:Ngồi ra, giả thiết.20/có thể viết lại thành
h.x/h 1Cxh.xy/
>xCxy h.x/2
; 8x; y > 0; hay
h.x/h 1Cxh.y/
>xCy h.x/2
; 8x; y > 0: 200/ Bây giờ, thay x D h.y/1 vào phương trình 200/ với ý h
h.y/
D
h h.y/ D h.y/
y2 (do
g h.y/
Dy2nênh.y/h h.y/
Dy2), ta h.y/h.2/
y2 > h.y/ C
h.y/2 y3 ; hay
h.2/ > y
h.y/2 C h.y/
y : Sử dụng bất đẳng thức AM-GM, ta có y2
h.y/
2 C
h.y/ y >2
q y
h.y/:Do đó, từ bất đẳng thức trên, ta suy tồn sốk > 0sao choh.y/>ky với mọiy > 0:
Vìh.x/là hàm nhân tính nên ta cóh.xn/D h.x/nvới mọinngun dương Doh.xn/>kxn nênh.x/ > pn kx với mọinnguyên dương Chon ! C1;ta đượch.x/ > xvới mọix > 0: Từ đây, kết hợp với h.x/h h.x/
D x2; ta dễ dàng suy ra h.x/ D x với mọi x > 0: Từ đó g.x/ Dxh.x/D x2với mọix > 0:Vậyf x; y/ Dg.x/h.y/ D x2y với mọix; y > 0:Thử lại, ta thấy hàm thỏa mãn yêu cầu toán
1 Bài tập tự luyện
Bài tập (GQMO, 2020) Tìm tất hàm sốf WQ!Qthỏa mãn f x/f y C1/Df xf y/Cf x/; 8x; y 2Q:
Bài tập Tìm tất hàm sốf WR!Rthỏa mãn f xCy/2
D f x/2
C f y/2
Cxf y/Cyf x/; 8x; y 2R:
Bài tập (Taiwan IMO TST, 2019) Tìm tất hàm sốf WR!Rthỏa mãn f xf y/ f x/ y
Dyf x/ f y/ x; 8x; y 2R:
Bài tập Tìm tất hàm sốf WR!Rthỏa mãn
(129)Bài tập (Tuymaada, 2003) Tìm tất hàm số liên tụcf WRC !Rthỏa mãn f
xC
x
Cf
yC y
Df
xC
y
Cf
yC x
; 8x; y > 0:
Bài tập (RMM Shortlist, 2016) Tìm tất hàm sốf WN!Nthỏa mãn f aCb/Df a/Cf b/Cf c/Cf d /
với số tự nhiêna; b; c; d sao cho2abDc2Cd2:
Bài tập (IMO Shortlist, 2013) Tìm tất hàm sốf WQ!Zthỏa mãn f f x/ Ca b Df x Ca b
; 8x 2Q; a2Z; b 2ZC:
Bài tập (USAMO, 2018) Tìm tất hàm sốf WRC !RCthỏa mãn f
xC
x
Cf
yC y
Cf
z C1
z
D1
với số thực dươngx; y; zmàxyz D1:(Ở đâyRCđược ký hiệu tập số thực dương.)
Bài tập Tìm tất hàm liên tụcf WR!Rthỏa mãn
f xCy/Cf xy/Df x/Cf y/Cf xyC1/; 8x; y 2R:
Bài tập 10 (IMO Shortlist, 2000) Tìm tất cặp hàm sốf; g WR!Rthỏa mãn f xCg.y/
Dxf y/ yf x/Cg.x/; 8x; y 2R:
Bài tập 11 Tìm tất hàm sốf WR!Rthỏa mãnf 0/>0và f xCy/>f x/Cyf f x/
; 8x; y 2R:
Bài tập 12 (IMAR, 2008) Ký hiệuRClà tập số thực dương Chứng minh với hàm sốf WRC!RC;luôn cặp số dương.x; y/sao cho
f xCy/ < yf f x/:
Bài tập 13 (Iran, 2019) Tìm tất hàm sốf WR!Rsao cho với số thựca; b; c;nếu aCf b/Cf f c/
D0; thì
f a/3
Cb f b/2
Cc2f c/D3abc:
Bài tập 14 (Turkish IMO TST, 2011) Tìm tất hàm sốf WQC!QCthỏa mãn f
x xC1
D f x/ xC1; f
x
D f x/
(130)KHÁM PHÁ TỪ MỘT BÀI TOÁN HAY
Nguyễn Duy Phước
(Trường Trung học phổ thơng Chun Quốc Học – Huế)
TĨM TẮT
Bài viết kết khám phá, mở rộng mà tác giả khai thác từ tốn có cấu hình đơn giản Qua đó, xin chia sẻ với bạn đọc cách học tốn, đặc biệt với phân mơn hình học thân tác giả theo hướng khai thác toán ban đầu để hiểu thêm tốn Cụ thể ta đến nội dung toán sau
1 Bài tốn mở đầu
BÀI TỐN1.Cho tam giácABC Đường thẳng quaB, C vng góc vớiAB, AC theo
thứ tự cắt đường thẳng quaAvng góc vớiBC tạiX, Y Đường trịn đường kính XY
cắtBX, CY lần thứ hai tạiZ, T Chứng minh rằngZT qua trực tâm tam giácABC
LỜIGIẢI
CÁCH 1.GọiS giao điểm củaBX vàCY, K hình chiếu củaSlênXY, Hlà trực tâm
tam giácABC
Ta có ∠AKS = ∠ABS =∠C = 90◦ nên năm điểmA, B, C, K, S nằm đường tròn
đường kính AS Từ đó, dễ thấy tứ giácBHCS hình bình hành NênSH chia đơiBC Mặt
khácSK ⊥XY, XY ⊥BC, suy raSK kBC
Do đóS(KH, BC) =−1, suy raS(KH, XY) =−1nên(KH, XY) =−1
Dễ thấySK, Y Z, XT đường cao tam giácSXY nênSK, Y Z, XT đồng quy Lại có
(131)A
C X
Y B
Z T
S H
K
CÁCH 2.Ta kí hiệu điểm tương tự cách
Ta có ∠SKY = ∠SZY = 90◦ nên tứ giác SZKY nội tiếp suy ra ∠BZK = ∠KY S Vì BH ⊥CA, CY ⊥CAnênBH kCY đó∠KY S =∠HY C =∠BHY =∠BHK
Dẫn đến∠BZK =∠BHK Suy tứ giácBHZK tứ giác nội tiếp Nên ta có biến đổi góc
∠KHZ =∠KBZ =∠KBS =∠KAS
(132)A
C X
Y B
Z T
S H
K
Nhận xét Từ hai cách chứng minh trên, ta rút hai hệ sau:(KH, XY) = −1và
ba điểm Z, T, H nằm đường thẳng song song với AS Tuy hai hệ đơn giản
nhưng đóng vai trị quan trọng cho toán Tiếp theo vấn đề khai thác đến toán
2 Một số tốn
BÀI TỐN2.Cho tam giácABC,Hlà trực tâm Đường thẳng quaB, Clần lượt vng góc với AB, AC theo thứ tự cắt AH tạiX, Y Chứng minh BC trục đẳng phương đường
trịn đường kínhXY đường trịn(H; 0)
LỜIGIẢI.GọiDlà giao điểm củaAH vàBC,K giao điểm thứ hai củaAH đường trịn
ngoại tiếp tam giácABC Khi đó, ta có kết quen thuộcHvàK đối xứng quaBChay Dlà trung điểm củaHK
(133)trịn đường kínhXY đường tròn(H; 0)đi qua điểmD
Ta nhận thấy tâm đường trịn đường kínhXY trung điểm củaXY, BC quaD
vàBC ⊥XY
VậyBC trục đẳng phương đường trịn đường kínhXY đường tròn(H; 0)
A
C X
Y B
H
K D
BÀI TOÁN 3.Cho tam giác ABC, trực tâm H Đường thẳng quaB, C vng góc với AB, AC theo thứ tự cắt AH X, Y Đường trịn đường kính XY cắt đường tròn
(BHX), (CHY)lần thứ hai tạiZ, T ZX, T Y cắtBC Q, P GọiU giao điểm
củaHP vàZX,V giao điểm củaT Y vàHQ
a) Chứng minh bốn điểm Z, T, U, V nằm đường tròn Gọi đường trịn
(ω)
b) Chứng minh đường tròn(ω)tiếp xúc vớiAH
LỜIGIẢI a) Theo tốn thìBC trục đẳng phương đường trịn đường kính XY
đường trịn (H; 0) Suy raP H2 = P T P Y Do đóP H là tiếp tuyến đường tròn(CHY). Nên∠U HT =∠P HT =∠HY T =∠XY T =∠U ZT suy bốn điểmH, U, Z, T nằm
(134)đường tròn
Vậy năm điểmH, Z, T, U, V nằm đường tròn(ω)
A
C X
Y B
H D
T Z
P Q
V
U
b) GọiDlà giao điểm củaAH vàBC Khi theo cách chứng minh tốn thìDH2 =
DX.DY Ta chứng minh ba điểmZ, T, D thẳng hàng
Ta cóP H, QH tiếp tuyến đường tròn ngoại tiếp tam giácCHY, BHX Suy
ra ∠QHD = ∠QHX = ∠HBX,∠P HD = ∠P HY = ∠HCY Mặt khác doBH k CY
(do vuông góc với AC), CH k BX (do vng góc với AB) ba điểm H, X, Y
thẳng hàng nên∠HBX = ∠HCY Do ∠QHD = ∠P HD nên HDlà tia phân giác
gócP HQ Lại cóHD⊥P Q≡BC
Vì HD đường trung trực P Q hay P điểm đối xứng với Q qua HD dẫn đến
∠DQX =∠DP X
Ta có∠P T X = 180◦−∠XT Y = 180◦−90◦ = 90◦ =∠P DX nên tứ giácP DT X nội tiếp,
∠Y ZQ =∠Y ZX = 90◦ =∠Y DQnên tứ giácY ZDQnội tiếp Suy ta có biến đổi góc
∠ZT X =∠ZY X =∠ZY D=∠ZQD=∠XQD =∠XP D =∠DT X
Vì thế, ba điểm Z, T, D thẳng hàng Do DZ.DT = DX.DY = DH2.Vậy đường trịn
(135)Nhận xét 2.Chúng ta chứng minhU V kDH cách sử dụng bổ đề Reim vào đường
trịn(ω)và đường trịn đường kínhXY (khi đóU V k XY ≡ DH) Sau đó, sử dụng yếu tốP
và Qlà hai điểm đối xứng với quaDH biến đổi góc đơn giản ta
được(ω)≡(HU V)tiếp xúc vớiDH≡AH
BÀI TỐN4.Cho tam giácABC Gọi Dlà hình chiếu củaA lênBC Đường thẳng quaB, C
lần lượt vng góc với AB, AC theo thứ tự cắt ADtại X, Y BX, CY cắt lại đường
trịn đường kínhXY tạiE, F.(BEC),(BF C)lần lượt cắt đường trịn đường kínhXY
điểm thứ hai làU, V
a) Chứng minh đường tròn ngoại tiếp tam giácU V Dđi qua trực tâm tam giácABC
b)(BEC)cắtACtạiQkhácC,(BF C)cắtABtạiP khácB Chứng minh rằngP Qtiếp xúc
với đường tròn(U V D)
LỜIGIẢI.a) GọiHlà trực tâm tam giácABC,Slà giao điểm củaBH vàCX Trước tiên,
ta chứng minh ba điểmU, X, C thẳng hàng; ba điểmB, V, Y thẳng hàng GọiU0là giao điểm
thứ hai củaCX đường trịn đường kínhXY Ta có
∠Y U0C =∠Y U0X = 90◦ =∠Y DC,∠Y EB =∠Y EX = 90◦ =∠Y DB
Suy tứ giácY U0DC,Y EDBnội tiếp Do đóXU0.XC =XD.XY =XE.XB Vì
tứ giácBU0ECnội tiếp suy raU ≡U0 hay ba điểmC, X, Uthẳng hàng Chứng minh tương tự,
ta ba điểmB, V, Y thẳng hàng
Ta có tứ giácY U DCnội tiếp nên∠U DY =∠U CY Mặt khácBH kCY (do vng
góc vớiAC) nên∠HSU =∠U CY Do đó∠U DY =∠HSU Khi tứ giácSHDU nội tiếp
Theo tốn thìBC trục đẳng phương đường trịn đường kính (XY)và đường trịn (H; 0) Suy raBH2 =BV.BY vì ta dễ dàng chứng minh hai tam giácBHV vàBY H đồng dạng theo trường hợp c - g - c Do đó∠SHV =∠BHV =∠BY H =∠V Y X =∠SU V
nên tứ giácHSU V nội tiếp
(136)A
C X
Y B
H D P Q
T
A’ E
F U
V S
b) GọiA0 là giao điểm củaBX vàCY Theo toán ba điểmH, E, F thẳng hàng ba
điểmH, E, F nằm đường thẳng song song vớiAA0 Dễ thấyAA0 đường kính
đường trịn ngoại tiếp tam giácABC
Ta có biến đổi góc: ∠BP F = ∠BCF = ∠BCA0 = ∠BAA0 suy raP F k AA0 Lại có cát
tuyếnHEF kAA0 Suy bốn điểmP, H, E, F thẳng hàng Chứng minh tương tự, ta bốn
điểmQ, H, E, F thẳng hàng
GọiT giao điểm củaU V vàEF Áp dụng định lý Pascal cho điểmU, E, Y, F, V, X với
ýU V ∩EF =T, EX ∩V Y =B, U X∩F Y =Cta có ba điểmB, T, C thẳng hàng
Theo toán BC trục đẳng phương đường trịn (XY)và đường trịn(H; 0) Khi
đó, ta đượcT H2 =T U.T V VậyT H là tiếp tuyến đường tròn(HU V)≡(U V D)hayP Q tiếp xúc với(U V D)tạiH
BÀI TỐN5.Cho tam giácABCnội tiếp đường trịn(O) Đường thẳng quaB, Clần lượt vng
góc vớiAB, AC theo thứ tự cắt đường thẳng quaAvng góc vớiBCtạiX, Y.M, N
(137)a) Chứng minh bốn điểmA, P, Q, Ocùng nằm đường tròn
b) Chứng minh đường tròn (O),(AP Q),(P M),(QN) (OM N)cùng qua
điểm
LỜI GIẢI a) GọiZ hình chiếu vng góc AlênBC, K giao điểm củaBX CY
Dễ thấyAK đường kính đường trịn(O),Z thuộc đường trịn đường kínhBXvà đường
trịn đường kínhCY GọiDlà giao điểm thứ hai đường trịn đường kínhBX đường trịn
đường kínhCY
Ta có ∠BDC = ∠BDZ +∠ZDC = ∠BXZ +∠ZY C = ∠KXY +∠XY K = 180◦ −
∠XKY =∠BKC Suy raDnằm đường tròn(KBC)≡(O)
Dễ thấy M B = M D, OB = OD suy OM đường trung trực BD hay B D đối
xứng quaOM Suy ra∠M OD = ∠BOD2 =∠BKD =∠M KDnên tứ giácOM DK nội
tiếp hayM thuộc đường tròn (OKD) Chứng minh tương tự, ta N nằm đường tròn
(OKD)
Từ suy ra∠P OQ = 180◦ −∠M ON = 180◦ −∠M KN = ∠BKC = 180◦−∠BAC =
(138)A
C X
Y
B Z
D P
Q
O
M
N K
b) Ta chứng minh đượcB D đối xứng quaOM Mặt khác ∠M BP = 90◦ suy
∠M DP = 90◦ nênD thuộc đường trịn đường kính P M Chứng minh tương tự, ta được D
thuộc đường trịn đường kínhQN Ta có biến đổi góc
∠AP D=∠BP D= 2∠M P D = 2∠M BD= 2∠KBD= 2∠KAD= 2(90◦−∠AKD) =
180◦−2∠AKD= 180◦−∠AOD
Do tứ giác AP DO nội tiếp hayDnằm đường tròn(AP O) ≡ (AP Q) Vậy đường
tròn(O),(AP Q),(P M),(QN)và(OM N)cùng qua điểmD
Nhận xét Từ tốn này, chứng minh trực tâm tam giác OM N
nằm trênBC Lúc này, đường thẳng quaB C đường thẳng Steiner tam giác OM N ứng với điểmD
BÀI TOÁN6.Cho tam giácABC,Dlà chân đường cao hạ từAxuốngBC,D0là điểm đối xứng
(139)tự cắtADtạiX, Y GọiM trung điểm củaXY Gọi N giao điểm củaBX vàCM,P
giao điểm củaBM vàCY Chứng minh ba điểmN, P, D0 thẳng hàng
LỜI GIẢI Gọi A0 giao điểm BX CY Dễ thấyAA0 đường kính đường trịn
ngoại tiếp tam giácABC
Vì D0 là điểm đối xứng với D qua trung điểm của BC, AA0 là đường kính của (ABC) nên A0D0 ⊥BC Mặt khácXY ⊥ BC Suy raXY kA0D0 Lại cóM trung điểm củaXY Do
đóA0(D0M, XY) = −1suy raA0(D0M, BC) =−1 Từ đó, dễ dàng đượcBA0, CM và P D0 đồng quy MàN là giao điểm củaBA0, CM Vậy ba điểmN, P, D0 thẳng hàng.
A
C X
Y
B D D’
A’ M
N
P
BÀI TOÁN7.Cho tam giácABC Đường thẳng quaB, C vng góc vớiAB, AC theo
(140)LỜIGIẢI.GọiHlà trực tâm tam giácABC,F điểm đối xứng vớiHquaAB,M trung
điểm củaBC, AM cắtCH tạiN, Slà giao điểm củaCH vàAB
VìF điểm đối xứng vớiH quaABnên ba điểmF, H, C thẳng hàng Lại cóP đối xứng với X quaABnên theo tính chất đối xứng trục ba điểm A, F, P thẳng hàng vàAB, P H, XF
đồng quy tạiT Ta cóBlà trung điểm củaP X
A
C X
Y B
P
Q F
H
T N
M S
Áp dụng định lý Menelaus vào tam giácABM với ba điểmS, N, Cthẳng hàng
SA SB
N M N A
CB CM =
Tương đương N M N A =
CM CB
SB SA =
1
HX HA
Áp dụng định lý Menelaus vào tam giácABX với ba điểmP, T, Hthẳng hàng
T B T A
HA HX
(141)Tương đương T B T A =
HX HA
P B P X =
1
HX HA
Suy N M N A =
T B T A(=
1
HX
HA) Do T N k BM Mặt khác AH ⊥ BM Nên AH ⊥ T N,
ta có N H ⊥ AT Vì H trực tâm tam giác AT N Như T H ⊥ AN hay P H ⊥AM Chứng minh tương tự, ta đượcQH ⊥AM
Vậy ba điểmP, H, Qcùng nằm đường thẳng vng gócAM
BÀI TỐN8.Cho tam giácABC,Hlà trực tâm Đường thẳng quaB, Clần lượt vng góc với AB, AC theo thứ tự cắtAH tạiP, Q Gọi(ω)là đường tròn quaH tiếp xúc với AH
(ω)lần lượt cắt đường tròn(BP H),(CQH)tại điểm thứ hai làX, Y Gọi Z giao điểm
củaBXvàCY Chứng minh đường tròn(XY Z)tiếp xúc với(BP H),(CQH)
LỜIGIẢI (Lê Viết Ân, GV trường PTNK, thành phố Hồ Chí Minh)GọiE giao điểm thứ hai
của đường tròn(CHQ)vàAC,F giao điểm thứ hai đường tròn(BHP)vàAB
A
C B
E
F H
P
Q X
Y N
M T
K Z
(142)GọiK giao điểm thứ hai đường trịn(ω)vàEF Vì(ω)tiếp xúc vớiAHnên tâm của(ω)
nằm trênEF nênHK đường kính của(ω) Đến đây, kẻKM ⊥ CH tạiM,KN ⊥ BH N đóM, N thuộc đường trịn(ω)
Ta cóKM kBF (do vng góc vớiHC) nên theo bổ đề Reim ba điểmB, X, M thẳng
hàng Chứng minh tương tự, ta ba điểmC, Y, N thẳng hàng
GọiT giao điểm thứ hai đường tròn ngoại tiếp tam giácXY Z vàHY Khi đó, ta có biến
đổi góc
∠Y T Z =∠Y XZ =∠Y XM =∠Y HM =∠Y HC
NênZT k CH Do đường trịn(XY Z) ≡(Y T Z)tiếp xúc với(Y HC)≡ (CHQ) Chứng
minh tương tự, ta đường tròn (XY Z)tiếp xúc với(BHP) Vậy đường tròn(XY Z)tiếp
xúc với(BHP),(CHQ)
Nhận xét 4.Tác giả Lê Viết Ân phát hai kết sau từ toán:
1) Tâm đường tròn ngoại tiếp tam giác XY Z thuộc elip cố định đường tròn (ω)
thay đổi
2) Nếu cho đường trịn(ω)tiếp xúc vớiBC đường tròn ngoại tiếp tam giácXY Z tiếp xúc
với đường tròn ngoại tiếp tam giác ABC Sau hồi biến đổi cấu hình, ta có tốn tổng
quát sau:
Cho tam giácABC nội tiếp đường tròn (O);E, F lần lượt nằm trênCA, AB. P là điểm bất
kì trên EF Một đường trịn quaP và tiếp xúc vớiBC lần lượt cắt(BP F),(CP E)tại các
điểm thứ hai làX, Y GọiZ là giao điểm củaBXvàCY Chứng minh đường tròn ngoại
tiếp tam giácXY Z tiếp xúc với đường tròn(O).
Lời giải cho toán xin dành cho bạn đọc Chúng ta tiếp tục với toán
BÀI TOÁN9.Cho tam giácABC Đường thẳng quaB, C vng góc vớiAB, AC theo
thứ tự cắt đường thẳng quaAvng góc vớiBCtạiX, Y Đường trịn đường kínhXY
cắtBX, CY lần thứ hai tạiZ, T.ZT cắtAX,(BHX),(CHY)tạiH, P, Q(P, Qkhác H).BP cắtCQtạiR Chứng minh đường tròn(P QR)tiếp xúc vớiXY
LỜIGIẢI.GọiE giao điểm thứ hai của(CHY)và AC,F giao điểm thứ hai của(BHX)
và AB,K giao điểm củaBX vàCY Khi theo tốn thìH trực tâm tam giác
ABC cát tuyếnHZT kAK
Dễ thấy ba điểmH, E, F thẳng hàng cát tuyếnEHF kBC Ta có
∠BF X =∠BHX =∠ACB =∠AKB=∠AKX
Suy tứ giácAF XK nội tiếp Chứng minh tương tự, ta tứ giácAEKY nội tiếp
GọiP0 là giao điểm thứ hai củaF K và đường trịn(BHX) Thì ta có∠XHP0 = ∠XF P0 =
∠XF K =∠XAKnênHP0 kAK Lại có cát tuyếnHZT kAK Suy bốn điểmH, Z, T, P0
thẳng hàng Do đóP ≡ P0hay nói cách khác ba điểmK, P, F thẳng hàng Chứng minh tương
(143)A
C B
E
F H
X
Y
S K
Z T P
Q
R
GọiS hình chiếu vng góc củaK lênXY Theo toán 2, BC trục đẳng phương
đường trịn đường kính XY đường trịn (H; 0) nên CH2 = CT.CY dẫn đến 4CHT v 4CY H (c - g - c) suy ra∠CHT =∠CY H
Vì ∠KQY = ∠EQY = ∠ECY = 90◦ = ∠KSY nên tứ giác KSY Q nội tiếp Lại có
∠KY Q = ∠CY Q = ∠CHQ = ∠CHT = ∠CY H = ∠KY S suy raKS = KQ Chứng
minh tương tự, ta đượcKS =KP.Vì vậyKS =KP =KQsuy raKlà tâm đường tròn ngoại
tiếp tam giácSP Q
Tiếp tục biến đổi góc
∠P RQ= 180◦−∠RP Q−∠RQP = 180◦−∠HP B−∠HQC =
180◦ −∠AF H−∠AEH =∠EAF =∠BAC
Do tam giác KP Qcân tạiK nên∠P KQ = 180◦−2∠KP Q = 180◦ −2∠F P H = 180◦ −
∠F BH = 2∠BAC = 2∠P RQ
Suy K tâm đường tròn ngoại tiếp tam giácP QR Khi đóK tâm đường trịn
ngoại tiếp tứ giácP RQS Mặt khác KS ⊥ XY VậyXY tiếp xúc với đường tròn (P QR)tại
S
BÀI TOÁN 10 Cho tam giác ABC Đường thẳng qua B, C vng góc với AB, AC
(144)lần lượt cắt BX, CY E, F (E khác X, F khác Y) Đường trịn đường kínhXY
cắt (BEC),(BF C) U, V (U khác E, V khác F) K giao điểm BC U V, KZ
tiếp xúc với đường tròn(XY)tạiZ (BEC)cắtAC tạiQkhácC, (BF C)cắtAB tạiP khác B, đường thẳng quaA song song với BC cắt P Qtại S M điểm Miquel tứ giác toàn
phần (AB, BC, CA, P Q) Chứng minh đường tròn (K, KZ)tiếp xúc với đường tròn (ABC)
và(ASM)
LỜIGIẢI.(Phan Như Hải, học sinh trường THPT chuyên Lê Quý Đôn, Đà Nẵng)
GọiO tâm đường tròn ngoại tiếp tam giácABC Rõ ràngBX, CY vàAOđồng quy I nằm đường tròn ngoại tiếp tam giácABC
Gọi Dlà trung điểm XY, H trực tâm tam giácABC, Glà giao điểm thứ hai AH đường tròn ngoại tiếp tam giácABC
Ta có(HG, XY) = −1nên theo hệ thức Newton DZ2 = DX2 = DG.DH dẫn đếnDZ tiếp xúc với đường trịn(HGZ)hay gọiK0 tâm đường trịn(HGZ)thìK0nằm BC trung trực củaGH K0Z vng góc vớiDZ MàKZ tiếp xúc với đường tròn
(XY)tạiZ nênKZ vng góc vớiDZ chứng tỏK ≡K0.
A
C B
H
X
Y Q
P S
M
O
I G
D V
U E
F Z K
(145)thẳng hàng Khi đó, áp dụng định lý Pascal cho điểmU, E, Y, F, V, X với ýEX∩V Y =
B, U X ∩F Y =C ta có giao điểm củaU V vàEF nằm trênBC DoK giao điểm củaBC
vàU V nênK thuộcEF
Theo tốn ba điểmH, E, F thẳng hàng cát tuyếnHEF kAI
Ngoài ra,∠QEB = ∠QCB = ∠AIB nênQE k AI, tương tự thìP F k AI Như vậy, ta có
năm điểm H, E, F, P, Q thẳng hàng nằm đường thẳng song song vớiAI Thành
thửHK k AOhay ba điểmO, K, Gthẳng hàng Vậy đường tròn(K, KG)tiếp xúc với(O)tại
G
Bằng biến đổi góc đơn giản, ta
4ASP v4HKB v4GKB
4AQP v4HCB v4GCB
Suy SQ SP =
KC
KB Xét phép vị tự quay tâmM biến tam giácM QP thành tam giácM BC nên
cũng biến S thànhK dẫn đến 4M SQ v 4M KC có được∠SM Q = ∠KM C = ∠KQC
do ∠M SH = ∠SM Q+∠SQM = ∠KQC +∠SQM = ∠M QA = ∠M AH nên tứ
giácASM H nội tiếp Lại có∠SAH = 90◦ do tứ giácASM H nội tiếp đường trịn đường
kínhSH Vì đường tròn (K, KZ)tiếp xúc với đường tròn (ASM)tạiH Vậy đường tròn
(K, KZ)tiếp xúc với đường tròn(O),(ASM)
Chúng ta hành trình khám phá toán ban đầu để đạt nhiều toán Thông qua viết này, tác giả muốn nhắn nhủ tới bạn đọc muốn học hình đạt kết tốt cách khai thác tốn theo nhiều khía cạnh khác nhau, ta thấy vẻ đẹp toán tăng thêm kinh nghiệm cho thân
Sau đây, xin đề nghị đến bạn đọc tập sau
3 Bài tập đề nghị
BÀI TẬP Cho tam giácABC, trực tâm H Đường thẳng qua B, C vng góc với AB, AC theo thứ tự cắt AH X, Y Lấy E, F đường tròn đường kính XY cho BX =BE, CY =CF.XF, Y E cắtBC tạiM, N
a) Chứng minh rằngHM kY E, HN kXF
b) GọiP, Q, R, Slần lượt trung điểm củaHM, HN, CA, AB Chứng minh rằngP S, QRvà AH đồng quy
BÀI TẬP2.Cho tam giácABC,Dlà chân đường cao kẻ từAcủa tam giácABC Đường thẳng
quaB, C vng góc vớiAB, AC theo thứ tự cắtAD tạiX, Y GọiM trung điểm
củaXY.N giao điểm củaBXvàCM,P giao điểm củaBM vàCY
a) Chứng minh đường thẳng nối trọng tâm tam giácABC tam giácM N P vuông
(146)b) Gọi U, V trực tâm tam giác XM N, tam giác Y M P Chứng minh ba
điểmD, U, V thẳng hàng
BÀI TẬP3 Cho tam giácABC, H trực tâm Đường thẳng quaB, C vuông góc với AB, AC theo thứ tự cắt AH X, Y Đường trịn đường kính XY cắt CX, BY E, F (E khác X, F khácY) Đường tròn (BEC)cắt AC tạiQ khác C, đường tròn (BF C)
cắtABtạiP khácB Đường tròn(HEF)lần lượt cắt(BEC),(BF C)tạiU, V (U khácE,V
khácF) GọiKlà giao điểm củaP U vàQV Chứng minh rằngHK chia đôiBC
BÀI TẬP Cho tam giácABC Đường thẳng qua B, C vuông góc vớiAB, AC theo
thứ tự cắt đường thẳng quaAvng góc vớiBCtạiX, Y Một đường trịn(K)thay đổi qua
B, C cắtCA,ABtạiE, F (E khácC,F khácB) GọiSlà giao điểm củaF X vàEY Chứng
minh rằngSdi động đường thẳng cố định khi(K)thay đổi
BÀI TẬP5.Cho tam giácABC Đường thẳng quaB, Clần lượt vng góc vớiAB, ACtheo thứ
tự cắt đường thẳng quaAvng góc vớiBC tạiX, Y Đường trịn đường kínhXY cắt BX, CY tạiZ, T (Z khácX,T khácY).ZT cắtAX, CA, AB tạiH, M, N GọiK
điểm choKB =KC KH ⊥ ZT Đường tròn(K, KH)lần lượt cắt(BHN),(CHM)
tạiP, QkhácH Chứng minh rằngBP, CQvàHK đồng quy
BÀI TẬP6 Cho tam giácABC có trực tâmH Đường thẳng quaB, C vng góc với AB, AC theo thứ tự cắtAH tạiX, Y GọiP điểm đối xứng vớiX quaBH, Qlà điểm đối
xứng với Y qua CH Chứng minh đường tròn ngoại tiếp tam giác HBC đường tròn
ngoại tiếp tam giácHP Qcắt điểm thứ hai nằm đường trung tuyến kẻ từAcủa tam
giácABC
BÀI TẬP Cho tam giácABC Đường thẳng qua B, C vuông góc vớiAB, AC theo
thứ tự cắt đường thẳng quaAvng góc vớiBCtạiX, Y Đường trịn đường kínhXY
cắtBX, CY tạiZ, T (Z khácX,T khácY).ZT cắtAXtạiH Gọiω1là đường tròn quaB, H tiếp xúc với BC;ω2 đường tròn qua C, H tiếp xúc vớiBC Đường trịn quaH, có tâm nằm ZT cắt ω1 ω2 tạiP, QkhácH BP cắtCQtạiR Chứng minh đường tròn(P QR)tiếp xúc vớiω1 vàω2
BÀI TẬP Cho hình bình hành ABCD Đường thẳng quaA vng góc vớiBD cắt CD, CB E, F Gọi P điểm đối xứng với E qua D, Q điểm đối xứng với F qua B
Các đường thẳng quaA, P, Qlần lượt vng góc vớiAE, CD, CBcắt tạo thành tam giác XY Z Chứng minh đường tròn ngoại tiếp tam giácXY Z tiếp xúc với đường tròn ngoại
(147)CÁC DẠNG TOÁN LIÊN QUAN ĐẾN
TRỰC TÂM CỦA TAM GIÁC
Lê Viết Ân (TPHCM)
GIỚI THIỆU
Trong viết này, xin đề cập đến số dạng toán liên quan đến trực tâm tam giác thường gặp chứng minh điểm trực tâm tam giác, trực tâm tam giác nằm đường thẳng hay đường tròn, dạng toán khác liên quan đến trực tâm tam giác ứng dụng tốn chứng minh quan hệ vng góc, song song, đồng viên điểm,
1 Phương pháp 1: Chứng minh trực tiếp từ định nghĩa
1.1 Nội dung phương pháp ví dụ
Để chứng minh điểm trực tâm tam giác, sử dụng định nghĩa trực tâm tam giác Trước hết có số nhận xét sau:
(1) Nếu H trực tâm tam giác khơng vng ABC AH ? BC, BH ? CA CH ?AB:
(2) Nếu tam giácABC vng tạiAthì xemAlà trực tâm tam giácABC:
(3) NếuH trực tâm tam giác khơng vngABC thìAlà trực tâm tam giácHBC, B trực tâm tam giácH CA, vàC trực tâm tam giácHAB
Chứng minh trực tiếp điểm H trực tâm tam giác ABC, cần chứng minh
được ba ý đơn giản sau: (a) AH ?BC vàBH ?CA
(b) BH ?CAvàCH ?AB
(148)Vì theo định lí đồng quy ba đường cao tam giác, ta suy có (a) (b) (c) thìH trực tâm tam giácABC
Để nắm rõ phương pháp này, xét số ví dụ sau:
Ví dụ Cho hình chữ nhậtX Y Z T và điểmR bất kì khácX; Y; Z; T nằm đường trịn
ngoại tiếp hình chữ nhật GọiA; B; C; Dtheo thứ tự đối xứng củaRquaX Y; Y Z; Z T; TX:Chứng minh rằngD là trực tâm tam giácABC:
Lời giải Khơng tính tổng qt, giả sửR thuộc đường trịn!ngoại tiếp hình chữ nhật
choR; X; Y; Z; T nằm trên!theo thứ tự
Chú ý rằngYX T[ D90ınên qua phép đối xứng trục, ba điểmA; X; Dthẳng hàng hayX 2AD
Tương tự,Y 2AB,Z 2BC vàT CD
Hơn nữa,RA?X Y ==T Z ?RC nênR2 AC GọiE DAD\CB Dễ thấy
\
EX T DDX T\ DTXR[ DT ZR[ D\T ZC DEZ T :\
Suy raE ! Do đóAD XE ? ZE BC Tương tự,CD ? AB Do đóD trực tâm
của tam giácABC:
Ví dụ (TTT2) Cho tam giác vngABC (vng tạiA/ GọiM là trung điểm cạngBC và H là chân đường vng góc hạ từ đỉnhAxuốngBC Trên tia đối tiaAM lấy điểmP
(không trùng vớiA/ Các đường thẳng quaH vng góc vớiABvàAC lần lượt cắt đường
thẳngPB vàP C tạiQvàRtương ứng Chứng minh rằngAlà trực tâm tam giácPQR.
Lời giải GọiE DBP \AC vàF DCP \AB Dễ thấyEF==BC để có EAAC D ABFA:
Gọi I D HQ \AC vàK D HR\AB Từ HQ==CE(? AB) vàHR==AB (? AC), theo
định lí Thales, ta có
QI IH D
EA AC D
FA AB D
RK
(149)Mặt khác,4IHK 4BHA(g.g) nên HQHR D HKHI D HB
HA, từ suy HB
HQ D
HA
HR Kết hợp
với BHQ\ D AHR[ (cùng 90ı AHI[) Ta nhận 4BHQ 4AHR (c.g.c) Do
đó gọi S D RA\BP thìBQH\ D ARH\ D SRH[ Suy tứ giácHQSR nội tiếp Mà
\
QHR D90ınênQSR[ D90ıhayRA?PQ Tương tự,QA?PR
Ví dụ (Grade level 9, All-Russian Olympiad, 2013) Cho tam giác nhọnABC nội tiếp đường
tròn Các tiếp tuyến củatạiB vàC cắt tạiP GọiDvàE theo thứ tự hình chiếu
vng góc củaP lênABvàAC Chứng minh trực tâm tam giácADE là trung điểm
cạnhBC.
(150)GọiK DEM \AD Ta có
\
KMB DEM C\ D\EP C D90ı \P CE D90ı KBC :\
Suy raBKM\ D 90ıhay EM ? AD Tương tự,DM ? AE đóM trực tâm tam
giácADE
Ví dụ (Lê Viết Ân) Cho tam giác không vuôngABC Các đường caoBB0vàC C0cắt nhau
tạiH GọiK là điểm trênB0C0(K khácB0; C0/ Đường thẳngAK cắtMB0; M C0
theo thứ tự tạiE; F GọiN là giao điểm củaBE vàCF Chứng minh rằngK là trực tâm của
tam giácNBC.
Lời giải Gọi.M / đường trịn đường kínhAH Kẻ BP ? CK P Đường thẳng qua A
vng góc vớiAC cắtBP tạiQvà cắt.M /tạiRkhácA
Dễ thấyB0Rlà đường kính của.M /và bốn điểmA; C; P; Qcùng thuộc đường trịn.I /đường
kínhCQ Dễ thấy năm điểmB; C; B0; C0vàP thuộc đường trịn đường kínhBC
Do đóKC :KP DKB0KC0 Suy ra
PK=.I / DKC :KP DKB0KC0DPK=.M /:
Điều chứng tỏAK trục đẳng phương của.M /và.I / Ta có
.PB0; PQ/.PB0; PB/ C0B0; C0B/.C0B0; C0A/.RB0; RA/.RB0; RQ/mod:
(151)Từ suy raPQlà trục đẳng phương của.I /và.J /, vàB0R B0M trục đẳng phương J /và.M / Chú ýE DAK\B0M vàAKlà trục đẳng phương của.M /và.I /, theo định
lý tâm đẳng phương, suy raE 2PQ Do đóCK ?PQ BE BN Chứng minh tương
tự, ta cóBK ?CN Do đóK trực tâm tam giácNBC 1.2 Các tập đề nghị
Bạn đọc giải số tập sau để rèn luyện phương pháp 1:
Bài tập (TTT2) Cho tam giácABC ĐiểmM nằm tam giác.AM; BM; CM theo thứ
tự cắt BC; CA; AB A0; B0; C0 Biết M tâm đường tròn nội tiếp tam giácA0B0C0
Chứng minh rằngM trực tâm tam giácABC
Bài tập (Grade level 9, All-Russian Olympiad, 2011) Cho tam giác nhọnABC Một đường
tròn qua B tâm đường tròn ngoại tiếpO tam giác, thứ tự cắtBC vàBA lần
các điểmP vàQ Chứng minh trực tâm tam giácOPQnằm đường thẳngAC
Bài tập (P2, Geometry Mathley, Round 1-2011) Cho tam giácABC nhọn,BE,CF
đường cao M trung điểm củaBC.N giao điểm củaAB EF X hình chiếu củaN
trênBC Y; Z theo thứ tự hình chiếu củaX trênAB; AC Chứng minh rằngN trực tâm
của tam giácAY Z
Bài tập (TTT2) Cho tam giácABC,.J /là đường tròn bàng tiếp đối diện đỉnhA J /tiếp
xúc với AB; AC tạiE; F: Giả sử JB; J C cắtEF K; L BL CK cắt
nhau tạiH Chứng minh rằngH trực tâm tam giácJBC
Bài tập (TTT2) Cho tam giácABC cân tạiA.M; Dtương ứng trung điểm củaBC,AM H hình chiếu củaM trênCD.AH cắtBC tạiN, BH cắtAM tạiE Chứng minh rằngE
là trực tâm tam giácABN
Bài tập (THTT) Cho tam giácABC vng tạiAngoại tiếp đường trịn.I; r/ Đường trịn I; r/tiếp xúc vớiAB; BC; CAlần lượt tạiP; Q; R GọiK trung điểmAC;đường thẳngIK
cắtAB tạiM Đoạn thẳngPQcắt đường caoAH tam giácABC tạiN Chứng minh N trực tâm tam giácMQR
2 Phương pháp 2: Chứng minh gián tiếp từ điều kiện đủ
2.1 Nội dung phương pháp ví dụ
Phương pháp gián tiếp chứng minh thông qua số kết khác mà phương pháp trực tiếp khơng khả thi khó mà thực Cụ thể sử dụng số điều kiện đủ để điểm trực tam tam giác sau:
Điều kiện đủ 1.Cho tam giácABC nội tiếp đường tròn.O/và đường cao hạ từ đỉnhAcắtBC
(152)Điều kiện đủ 2.Cho tam giácABC vàAA0là đường kính đường trịn ngoại tiếp tam giác ABC NếuH là điểm đối xứng vớiA0qua trung điểm củaBC thìH là trực tâm của4ABC:
Điều kiện đủ 3.Cho tam giácABC vàD là chân đường cao hạ từ đỉnhAxuốngBC NếuH
là điểm nằm trênAD thỏa mãnDADH D DBDC thìH là trực tâm của4ABC:
Điều kiện đủ 4.Cho tam giácABC và đường caoAD Dthuộc đoạnBC / Nếu điểmH thuộc
đoạnADthỏa mãnHBA[ DH CA\thìH là trực tâm của4ABC:
Điều kiện đủ 5.Cho tam giác nhọnABC và đường caoAD Nếu điểmH thuộc đoạnAD sao
choBH C\ D180ı BAC[ thìH là trực tâm của4ABC:
Điều kiện đủ 6.Cho tam giácABC nội tiếp đường tròn tâmO và M là trung điểm cạnhBC.
Nếu điểmH thỏa mãnAH!D2OM!thìH là trực tâm của4ABC:
Việc chứng minh điều kiện đủ1; 2; 3; 4; 5và 6đơn giản, xin phép bỏ qua dành lại cho
bạn đọc phần tập nhỏ Chúng ta đến số ví dụ minh họa sau:
Ví dụ (TTT2) Cho tam giác ABC Đường tròn.I / nội tiếp tam giác ABC, tiếp xúc với BC; CA; AB thứ tự tạiD; E; F Đường thẳng quaBsong song vớiDF cắt đường thẳng EF; DE thứ tự tạiM; N GọiH là giao điểm củaIB vàDF Chứng minh rằngH là trực tâm
của tam giácIMN.
Lời giải VìMN==FDvàDC tiếp xúc với.I /nên
\
(153)Tương tự,I NM <\ \DNB DMFB < 90\ ı:Do đó4BFM 4BND, suy BMBF D BNBD
VìBDI[ D90ı,DH ?BI,BC vàBF tiếp xúc với.I /nên BH:BI DBD2 DBD:BF D BD
BN BF
BM BN BM D BD BN
BN
BD BN BM DBM:BN:
Kết hợp với gócIMN\vàI NM\nhọn nên điểmH nằm đoạnBI Theo điều kiện 3, ta
cóH trực tâm tam giácIMN
Ví dụ (Turkey EGMO TST 2013) Cho tam giácABC vớiAB D AC Một đường tròn đi
quaAvàC cắt cạnhABtạiD Phân giác kẻ từAcủa tam giác cắt đường tròn tạiE khácA/.
Chứng minh trực tâm tam giácAEB nằm đường tròn.
Lời giải GọiK giao điểm đường tròn vớiBC Ta chứng minhK trực tâm tam
giácABE Thật vậy, gọiF DAE \BC vàLDKE \AB Ta có
[
LAF DBAF[ DCAF[ D\FKE D\LKF :
Suy tứ giác ALFK nội tiếp Chú ý AB D AC nên AF ? BC Suy KLA[ D [
KFA D90ınênKE ?AB Lại kết hợp vớiAE ?BK tạiF VậyK là trực tâm tam giác
ABE:
Ví dụ (Lê Viết Ân) Cho tam giác nhọnABC vớiO; H thứ tự tâm đường tròn ngoại tiếp
(154)Lời giải GọiAA0là đường kính của.O/ Ta có kết quen thuộc làBH CA0là hình bình
hành Do gọiK đối xứng củaA0quaBC thìHK==BC ?EF
\
CKD DCA\0D DCA\0ADCBA[ DCFD:[
Suy raK ˇ.CDF / Tương tự,K ˇ.BDE/ Do theo định lí Miquel cho tam giácHBC
vớiD BC; F CH vàE HB ta cóK ˇ.HEF /\ ˇ.CDF /\ ˇ.EBD/ Tức
ta cóK ˇ.HEF / Do gọiK0đối xứng củaK quaEF ý rằngHK ?EF nên
theo điều kiện đủ1;ta cóK0là trực tâm tam giácHEF:
Mặt khác, ý rằngBC ? EF nên theo phép đối xứng trục, dễ thấyK0 2 A0D AD Bài
toán chứng minh
Nhận xét Bài tốn tác giả khai thác từ G3IMOSL2017:Ta cịn có tính chất thú
vị là:NếuJ là tâm đường trịn ngoại tiếp tam giácHEF thìJB DJ C.Bạn đọc chứng
minh tập
2.2 Các tập đề nghị
Bạn đọc giải số tập sau để rèn luyện phương pháp 2:
Bài tập (Sharygin 2012 final) Cho! đường tròn ngoại tiếp tam giácABC Một điểmB1
trên tia đối tiaBAsao choAB1 D AC Phân giác củaBAC[ cắt!lần tạiW Chứng minh
rằng trực tâm tam giácAW B1nằm trên!
Bài tập (ELMO 2017) Cho tam giácABC với trực tâmH, vàM trung điểm cạnhBC
Giả sửP vàQlà hai điểm phân biệt nằm đường trịn đường kínhAH, khácA, choM
nằm đường thẳngPQ Chứng minh trực tâm tam giácAPQnằm đường tròn
(155)Bài tập (TTT2) ChoM điểm nằm hình bình hànhABCD thỏa mãnMAB\ D
40ı,MBC\ D25ı,M CD\ D65ı,MDA\ D50ı Tính góc hình bình hành
3 Phương pháp 3: Sử dụng định lí đường thẳng Steiner
3.1 Nội dung phương pháp ví dụ
Định lý (Đường thẳng Steiner) Cho tam giácABC có trực tâm H Với điểm P nằm
trong mặt phẳng tam giác, gọi A0; B0; C0 theo thứ tự đối xứng củaP qua đường
thẳngBC; CA; AB Khi đóA0; B0; C0thẳng khiP nằm đường tròn ngoại
tiếp tam giácABC Hơn nữa, trường hợpA0; B0; C0thẳng hàng đường thẳngA0B0C0
đi quaH:Đường thẳngA0B0C0được gọi đường thẳng Steiner củaP ứng với tam giácABC.
Việc chứng minh định lí đường thẳng Steiner có nhiều tài liệu, chẳng hạn xem [1] Tuy nhiên, xem xét biến thể sau có lợi việc chứng minh tốn u cầu chứng minh trực tâm tam giác nằm đường thẳng:
Phát biểu tương đương đường thẳng Steiner: Cho tam giác ABC và trực tâm H. P là
điểm thuộc mặt phẳng tam giác GọiA0; B0 thứ tự hình đối xứng củaP quaBC; CA.
Khi đóP nằm đường trịn ngoại tiếp tam giácABC khi khiA0B0 đi quaH.
Ta đến ví dụ minh họa sau:
Ví dụ (Sharygin 2013 final) Cho`là đường thẳng qua đỉnhBcủa tam giác đềuABC.
Một đường tròn!avới tâmIa tiếp xúc xúc vớiBC tạiA1, đồng thời tiếp xúc với`vàAC Một
đường tròn !c với tâmIc tiếp xúc vớiBA tạiC1, đồng thời tiếp xúc với`và AC Chứng minh
(156)Lời giải Từ giả thiết, dễ thấyBAI[c DBAC[ D60ıvàBCI\a DBCA[ D60ınên đối xứng
củaIc quaAB, củaIa quaBC nằm trếnAC
Nhưng lại vìABI\c C\CBIa D 60ı D \ABC, suy ảnh đối xứng phải trùng
Gọi ảnh đối xứng J Suy raJ D IcC \IaA Chú ý tứ giácBA1J C1 nội tiếp
Ia; Ic đối xứng J CB; BAnên theo định lí đường thẳng Steiner, ta cóIaIc
là đường thẳng Steiner củaJ ứng với tam giácA1BC1nên trực tâm tam giácA1BC1 nằm
trênIaIc:
Ví dụ (All-Russian 2011) Trên cạnhBC của hình bình hành ABCD (Alà góc nhọn), lấy
điểmT sao cho tam giácATD nhọn GọiO1; O2;và O3 theo thứ tự tâm đường tròn ngoại
tiếp tam giácABT; DAT vàCDT Chứng minh trực tâm tam giácO1O2O3nằm
trên đường thẳngAD:
Lời giải Dễ thấyO1O2; O2O3thứ tự trung trực củaAT; TDvà ý rằngADT\D \C TD,
(157)\
O1O2O3 DATD[ DO\1TO3:
Suy raT ˇ.O1O2O3/
Chú ý, theo tính chất trung trực, ta cóA; D đối xứng củaT quaO1O2; O2O3 nên
theo định lí đường thẳng Steiner, trực tâm tam giácO1O2O3nằm trênAD:
Ví dụ 10 (Lê Viết Ân) Cho tứ giácABCD nội tiếp đường tròn O/vớiAB D BC D CD.
Giả sử đường thẳng quaA song song vớiCO cắt đoạn BC tại E Gọi I và J theo thứ tự là
tâm đường tròn nội tiếp tam giácABE và CDO Chứng minh trực tâm tam
giácOIJ nằm trênAD.
Lời giải VìBC DCDnên
[
IAB D BAE[
2 D
[
BAC CAE[D
2
[
CAD ACO[D
2
\
BDC CAO[D \OAD
2 :
Chú ýBC ==ADvàBC DCDnênBDC\ D ADC\2
[
JDB DBDC\ CDJ[ D
2
\
ADC ODC\D ODA[
2 :
Từ OA D OD suy ra\OAD D ODA[ nên từ hai đẳng thức vừa thu được, ta cóIBAd D JDA[, tức làP DAI \JD nằm trên.O/ Từ đó, dễ thấy4IAC 4JDC suy raC ˇ.P IJ /:lại
vìOI ?CD vàOB ?CAnênIOJ[ D180ı ACD[ D180ı IPJd nênO 2.P IJ / Do
C ˇ.OIJ / Đến đây, ta có đượcADlà đường thẳng Steiner củaC ứng với tam giácOIJ
(158)Nhận xét Dễ dàng biến đổi góc để cóAI ?IJ
Ví dụ 11 (Lê Viết Ân) Cho tam giác nhọnABC nội tiếp đường tròn.O/ Các tiếp tuyến của O/tạiB vàC cắt tạiT. AT cắt.O/tạiD khácA, vàAO cắtBC tạiP Gọi K và L
theo thứ tự tâm đường tròn ngoại tiếp tam giác BDT và CDT Chứng minh trực
tâm tam giácOKLnằm trênP T.
Lời giải GọiH trực tâm tam giácOKL Dễ thấyOK ? DB OL ? DC Suy
[
KOL D180ı \BDC DBAC[
Dễ thấyBT C\D180ı 2BAC[,\K TB DBDT\ 90ıvàLT C[ DTDC\ 90ı Suy ra
\
K T LDK TB\C\BT C CLT C[
D\BDT 90ıC180ı 2BAC[ C\TDC 90ı
D.\BDT C\TDC / 2BAC[
D360 \BDC 2BAC[
D180ıCBAC[ 2BAC[
D180ı BAC[ D180ıCKOL:[
Suy tứ giácOK T Lnội tiếp
GọiX; Y theo thứ tự đối xứng T quaTB; T C Xét phép đối xứng trụcOK thìD; T; O/
biến thành B; X; O/ Qua phép đối xứng với ý BT tiếp xúc với.O/ suy raDX tiếp
xúc với O/ Tương tự, DY tiếp xúc với.O/ Do ba điểmX; D; Y thẳng hàng
(159)khác, gọi A0là đối xứng A quaO, ta cóOH ? KL ? DT nên OH==AD ? A0D nên OH trung trực củaA0H nên từHD tiếp xúc với.O/, suy raHA0tiếp xúc với O/ GọiE
là giao điểm thứ hai củaTAvà.O/ Theo kết quen thuộc (có thể chứng minh cực
và đối cực),DE quaP Áp dụng định lí Pascal cho sáu điểm
A0 E D
D A A0
, suy ba điểm
P DA0A\DE,H DA0A0\DD vàT DEA0\DAthẳng hàng hayH 2P T: 3.2 Các tập đề nghị
Bạn đọc giải số tập sau để rèn luyện phương pháp 3:
Bài tập 10 (Lê Viết Ân) Cho tứ giác nội tiếp ABCD Giả sử có điểmE nằm cạnhAD
sao cho tứ giácAECDvà BEDC ngoại tiếp GọiI; J vàK theo thứ tự tâm đường tròn
nội tiếp tứ giácAECD; BEDC tam giácCDE Chứng minh trực tâm
tam giácIJK nằm đoạn thẳngCD
Bài tập 11 (Lê Viết Ân) Cho hình chữ nhậtABCD Lấy điểmP thuộc cạnhAD Đường tròn
ngoại tiếp tam giácPBC cắtDA; AB; CDtheo thứ tự tạiE; F; G GọiK; Ltheo thứ tự
giao điểm cặp đường thẳng.EF; BP / EG; CP / GọiH hình chiếu vng góc
củaP lênBC Chứng minh đường thẳngKLđi qua trực tâm tam giácDAH
Bài tập 12 (Lê Viết Ân) Cho tam giác nhọnABC điểmDthuộc cạnhBC Các đường cao
của tam giác hạ từ đỉnhB vàC cắtADtheo thứ tự tạiE F Đường tròn ngoại tiếp
tam giácBDE vàCDF theo thứ tự cắt cạnhAB; AC tạiGvàH khácA:GọiDK vàDL
theo thứ tự đường phân giác tam giácBDG CDH Chứng minh
trực tâm tam giácAKLnằm trênGH
Bài tập 13 (THTT) Cho tam giác không vuôngABC, O; H thứ tự tâm đường tròn ngoại
tiếp trực tâm tam giácABC M điểm trên.O/ M khácA; B; C /: GọiN
điểm đối xứng củaM quaBC: P giao điểm thứ hai AM đường tròn ngoại tiếp tam
giácOMN:Chứng minh rằngHN qua trực tâm tam giácAOP
Bài tập 14 (G8, IMOSL 2009) ChoABCDlà tứ giác ngoại tiếp Gọiglà đường thẳng
quaAcắt đoạn thẳngBC tạiM đường thẳngCDtạiN GọiI1; I2, vàI3 theo thứ tự tâm
đường tròn nội tiếp tam giác ABM; MNC, NDA Chứng minh trực tâm
tam giácI1I2I3nằm trêng
Bài tập 15 (Lê Viết Ân) Cho tam giácABC không vuông tạiAvà nội tiếp đường tròn.O/ AP dây cung O/sao cho AP khơng đường kính khác AB; AC Giả sử AP cắt
trung trựcCAvàAB theo thứ tự tạiE vàF.OP cắtBC tạiD Chứng minh trực tâm
tam giácOEF thuộcAD
(160)4.1 Nội dung phương pháp ví dụ
Nội dung định lí Brocard sau:
Định lý (Định lí Brocard) Cho tứ giácABCD nội tiếp đường tròn tâmO NếuE D AC \
BD,F DAB \CDvàGDAD\BC thìO là trực tâm tam giácEF G:
Ví dụ 12 (Mathley 10/2015) Cho tam giác ABC và điểmP nằm tam giácABC sao
cho AP ? BC.E; F thứ tự hình chiếu củaP lênAC; AB Giả sử tiếp tuyến tạiE; F của
đường tròn ngoại tiếp tam giácAEF cắt trên BC Chứng minh rằngP là trực tâm của
tam giácABC.
Lời giải Gọi P0 D CF \BE Áp dụng định lí Pascal cho sau điểm
A F E P0 E F
với ý ba điểm C D AE \P0F, B D AF \P0E M D FF \EE thẳng hàng, suy P0 2.N /D ˇ.AEF /.
Gọi K D EF \AP0 áp dụng định lí Brocard, ta cóK trực tâm tam giác BCN,
suy NK ? BC Chú ý N trung điểmAP AP ? BC Do đóK AP Suy AP0 NK AP Điều chứng tỏP0 P Từ đóCP CP0 CF, tức làCP ? AB
kết hợp vớiAP ?BC ta suy raP trực tâm tam giácABC:
Ví dụ 13 Cho tam giácABC Đường tròn.I /nội tiếp tam giácABC, tiếp xúc vớiBC; CA; AB
thứ tự tạiD; E; F.H là điểm thuộc DF Đường thẳng quaB vng góc vớiIH cắt các
đường thẳngEF; DE thứ tự tạiM; N Chứng minh rằngH là trực tâm tam giácMI N:
(161)Áp dụng định lí Brocard cho tứ giácDEF Gthì trực tâm tam giácIMN phải nằm trênEF
và đường thẳng quaI vng góc vớiMN Do đóH D EF \IH trực tâm tam
giácIMN:
Nhận xét Bài toán tác giả mở rộng từ ví dụ
4.2 Các tập đề nghị
Bạn đọc giải số tập sau để rèn luyện phương pháp 4:
Bài tập 16 (TTT2) Cho tam giácABC không vuông Các đường coaBB0; C C0cắt H gọiK trung điểmAH:Giả sửI DAH \B0C0 Chứng minh rằngI là trực tâm tam giác
KBC
Bài tập 17 Cho tam giác nhọnABC, H trực tâm tam giác BH; CH theo thứ tự cắt AC; AB tạiE; F.EF cắtBC tạiK vàM trung điểmBC Chứng minh rằngH trực tâm
của tam giácAKM:
(162)5.1 Nội dung phương pháp ví dụ
Ta phối hợp nhiều phương pháp khác định lí lại với để chứng minh tốn Chúng ta điểm qua số ví dụ, hy vọng cung cấp cho bạn đọc số ý tưởng để xử lí tốn liên quan đến trực tâm
Ví dụ 14 Cho tam giácABC với trực tâmH ĐiểmDnằm trênBC Đường thẳng vuông góc
với BC tạiD cắtAB,AC thứ tự tạiF vàE Chứng minh trực tâm tam giácAEF nằm
trên đường thẳngHD:
Lời giải GọiK D AH \BC; M trực tâm tam giácAEF L D MA\EF Ta có AKDLlà hình chữ nhật nênAK DLD,
Dễ thấy 4MEF 4ABC (g.g), kết hợp vớiA; H theo thứ tự trực tâm hai tam giác
này, suy MA
ML D
AH
AK D
AH
LD Theo định lí Thales, suy ba điểmM; H; Dthẳng hàng Ta có
điều phải chứng minh
6 Ứng dụng chứng minh tính chất hình học
6.1 Ứng dụng
(163)Ví dụ 15 (AMPO 2010) Cho tam giỏcABC viBAC[ Ô90 GiOl tõm ng trũn ngoi
tip tam giác và là đường tròn ngoại tiếp tam giácBOC Giả sử cắt đoạn thẳngABtạiP
khácB, đoạn thẳngAC tạiQkhácC GọiON là đường kính của Chứng minh tứ
giácAPNQlà hình bình hành.
Lời giải GọiK DOQ\AB, Dễ thấyAQO\DOBC\ D90ı BAC[ D90ı KAQ\
Suy tam giác AKQ vng K Do \QKA D 90ı D OQN\ nênAP ==QN Tương tự AQ==PN VậyAPNQlà hình bình hành
Ví dụ 16 (TTT2) Cho tam giácABC vng tạiA, đường caoAH, điểm D nằm giữaH và C Kẻ DE; DK lần lượt vng góc vớiBC,AB (E AC; K AB) Chứng minh rằngBE
vng góc vớiHK.
(164)ĐặtF DHL\BE Áp dụng định lí Thales, ta có FH
FL D EC AC D
DC DH D
KA KL:
Từ suy raKF==AH ? BC BH Điều chứng tỏF trực tâm tam giácBHK
Do đóBF ?HK:
Ví dụ 17 (TTT2) Cho tam giácABC nhọn, đường caoAH ĐiểmM thuộc đoạnBC Đường
thẳng quaAvng góc vớiAM theo thứ tự cắt đường thẳng quaM vng góc vớiAB và AC tạiE vàF Chứng minh rằng:AH; BF; CEđồng quy.
Lời giải GọiK D AB\ME LDAC \MF đường thẳng quaB vng góc vớiCE, cắt AH tạiI Ta cóAIB[ DM CE\ vàABI[ DMEC\ nên4AIB 4M CE Do ABAI D M CME
Mặt khác
BM BC D
ŒABM ŒACM D
MKAB MLAC ;
EM FM D
AM2
KM AM2
LM
D LM
KM:
Do
BM FM D
BM CM
CM EM
EM FM D
KM AB LM AC
AI AB
LM KM D
AI AC:
Kết hợp vớiIAC[ DBMF\ suy ra4AI C 4MBF Từ dễ dàng có đượcCI ? BF Vậy AH; BF; CE đồng quy trực tâm tam giácIBC
6.2 Các tập đề nghị
(165)Bài tập 18 (TTT2) Về phía ngồi tam giácABC ta dựng tam giác vuông đồng dạngABE
và ACF ABE[ D ACF[ D 90ı/ Chứng minh BF; CE đường cao AH tam giác
đồng quy
Bài tập 19 (Lê Viết Ân) Cho tam giácABC nhọn, nội tiếp đường tròn.O/và H trực tâm
của tam giác.AH cắtBC tạiD Đường tròn ngoại tiếp tam giácAOH cắt.O/tạiGkhácA
QuaH kẻ đường thẳng vng góc vớiOH, cắtBC tạiE GọiM trung điểm củaGH; DM
cắtEGtạiN vàK trực tâm tam giácGDH Chứng minh rằngKN==OH
Bài tập 20 (TTT2) Cho tam giácABC vuông A, ngoại tiếp đường tròn I; r/ Kẻ đường
cao AH, gọi M trung điểm củaBC, Q giao điểm AH MI, E F
hình chiếu củaAtrênIB vàI C Chứng minh rằngAQ DEF
Bài tập 21 Cho tam giácABC với trung tuyếnAM Đường trịn đường kínhAMcắtBC; CA; AB
theo thứ tự lần thứ hai tạiD; E; F Chứng minh rằngAD; BE; CF đồng quy
Bài tập 22 (IMO 2012) ChoABC tam giác nhọn với trực tâmH, vàW điểm nằm
trên cạnh BC GọiM N theo thứ tự chân đường cao hạ từ đỉnhB C Gọi!1
đường tròn ngoại tiếp tam giácBW N, vàXlà điểm trên!1sao choW X đường kính Tương
tự, gọi!2là đường tròn ngoại tiếp tam giácC W M, vàY điểm trên!2sao choW Y đường
kính Chứng minh rằngX; Y vàH thẳng hàng
Tài liệu tham khảo
[1] Tài liệu chun tốn hình học10, Đoàn Quỳnh, Văn Như Cương, Trần Nam Dũng, Nguyễn
Minh Hà, Đỗ Thanh Sơn, Lê Bá Khánh Trình
[2] https://artofproblemsolving.com/community/c6h1497688p8819767
[3] https://artofproblemsolving.com/community/c6h1391916p7767007
[4] https://artofproblemsolving.com/community/ c6h1708869p11011491
[5] https://artofproblemsolving.com/community/ c6h1694248p10849645
[6] https://artofproblemsolving.com/community/ c6h1685247p10754378
[7] https://artofproblemsolving.com/community/c6h406967p2272760
[8] https://artofproblemsolving.com/community/c6h1468151p8509518
[9] https://artofproblemsolving.com/community/c6h1449744p8307127
(166)MỘT TỔNG QUÁT CHO ĐỊNH LÝ MUSSELMAN
Trần Quang Hùng, Hà Nội
TÓM TẮT
Bài báo đưa tổng quát cho định lý Musselman, với chứng minh hình học
Trong [1,2] đề cập tới định lý Musselman
Định lý 1. Cho tam giácABCcó tâm đường trịn ngoại tiếpO Ký hiệuA∗,B∗,C∗là đối xứng
củaA,B,Cqua cạnhBC,CA,AB Thì đường tròn ngoại tiếp tam giácAOA∗,BOB∗, COC∗ cắt điểm nghịch đảo qua đường tròn ngoại tiếp tam giácABC điểm đẳng giác
của tâm đường trịn chín điểm
Trong đợt tập huấn cho đội tuyển toán Việt Nam tham dự kỳ thi Olympic toán quốc tế năm 2006, tác giả viết sử dụng toán lời giải [4] Trong trình sử dụng tốn này, tác giả có phát triển thêm tốn thành hai ý 1) 2) (ở định lý đây) Điều thú vị việc phát triển lại vơ tình tạo mở rộng thú vị cho Định lý Hơn nữa, tác giả đề cho em đội tuyển năm ấy, tác giả lại nhận đóng góp nhiệt tình nhiều thành viên đội tuyển với lời giải thú vị khác Cuối nhờ có cần mẫn nhiệt tình biên tập bạnNgơ Quang Dươnghọc trị tác giả chuyên toán K48 THPT chuyên KHTN, tác giả hoàn thành viết Xin giới thiệu lại với bạn đọc tạp chí Epsilon, tốn dạng định lý sau
Định lý 2. Cho tam giácABCcóP,Qlà hai điểm đẳng giác.P A,P B,P C cắt đường tròn
(P BC),(P CA),(P AB)lần lượt X,Y,Z khácP XQ, Y Q,ZQ cắtY Z, ZX, XY theo
thứ tự tạiU,V,W Khi
i) đường thẳngAU,BV,CW điểm ký hiệu làT
ii) sáu đường trịn(AQX),(BQY),(CQZ),(AY Z),(BXZ),(CXY)có điểm chung
(167)A
B P C
X
Y
Z
Q U
V
W S T
Hình
Chứng minh. i) Ta có∠BAZ =∠BP Z =∠CP Y =∠CAY và∠AZB=∠AP Y =∠ACY
nên 4ABZ ∼ 4AY C g.g Lại có ∠QAC = ∠XAB ∠ACQ = ∠P CB = ∠P XB =
∠AXB nên4AQC ∼ 4ABX g.g Từ hai cặp tam giác đồng dạng dẫn đếnAB·AC =
AY ·AZ =AQ·AX Suy ra4AZQ ∼ 4AXY(cạnh-góc-cạnh) nên∠AQZ =∠AY W, dẫn
tới (QY W)đi qua A Tương tự, (QZV)đi qua A, theo định lý Miquel,(XZW) (XY V)
cũng quaA
(Phần chứng minh dựa ý tưởng emPhạm Nguyễn Mạnh) Ta lại có
(AV, AW) = (AV, AQ)+(AQ, AW) = (ZV, ZQ)+(Y Q, Y W) = (XZ, XY)+(QY, QZ) (mod π)
Tương tự ta có liên hệ
(BW, BU) = (Y X, Y Z)+(QZ, QX) (mod π) (CU, CV) = (ZY, ZX)+(QX, QY) (mod π)
Suy
0 = (AV, AW) + (BW, BU) + (CU, CV) = (U C, U B) + (V A, V C) + (W B, W A) (mod π)
nên(U BC),(V CA),(W AB)đồng quy điểmT
Theo trên, ta ra(XZW), (XY V), (QY W),(QZV)đi quaAnên cộng góc ta có
(168)cặp tam giác đồng dạng hướng nên ta có4CAV ∼ 4CU B
(T B, T C) = (U B, U C) (mod π) = (AV, AC) (mod π) = (T V, T C) (mod π)
Điều cho thấyT,B,V thẳng hàng Tương tự,AU,CW quaT
A
B P C
X
Y
Z Q
U
V
W S
T
Hình
ii)(BXZ),(CXY)cắt tạiSkhácXthì ta có biến đổi góc
(SY, SZ) = (SY, SX) + (SX, SZ) (mod π) = (CY, CX) + (BX, BZ) (mod π)
= (CY, CP) + (CP, CX) + (BX, BP) + (BP, BZ) (mod π) = (AY, AP) + (AP, AZ) = (AY, AZ) (mod π)
VậyS nằm trên(AY Z)
(Phần chứng minh dựa ý tưởng emĐào Vũ Quang) Chú ý tứ giácAQV Z,CQU Y,CQBXnội tiếp, ta có biến đổi góc
(SA, SX) = (SA, SY) + (SY, SX) (mod π) = (ZA, ZY) + (CY, CX) (mod π)
(169)Từ đóSthuộc(AQX) Tương tựS thuộc đường tròn(BQY),(CQZ)
(Phần chứng minh dựa ý tưởng emVũ Đức Tài)
Kí hiệu phép biến hình hợp phép nghịch đảo cực A phương tích AB ·AC với phép
đối xứng qua phân giác ∠BAC Φ Φ : B 7→ C, X 7→ Q, E 7→ F, (AEF) → EF,
(AQX)→QX đóT biến thànhXnên tiaASvàAXđẳng giác trong∠BAChayAU,AS
đẳng giác, tương tự đỉnhB,C Vậy ta kết luậnS,T đẳng giác tam giácABC
(Phần chứng minh dựa ý tưởng tác giả viết)
Chú ý.Thực việc lấy hợp thành phép nghịch đảo phép đối xứng qua đường thẳng chứa cực nghịch đảo sử dụng từ lâu, việc làm lần đề xuất nhà hình học John Wentworth Clawson [3] Việc trích dẫn [3] tránh cho nhầm tưởng gọi, đặt tên dịch sang tiếng Việt theo số tài liệu trôi mạng
Bài báo đăng vào thập niên đầu kỷ 20 (tức trước chiến thứ gần 20 năm) tờ báo tiếng ngày làAnnals of Mathematics Cần lưu ý thời điểm đóAnnals of Mathematicsđăng nhiều nghiên cứu sâu hình học sơ cấp mà bạn truy cập viết miễn phí sở liệu www.jstor.org Ngày Annals of Mathematicsvẫn tờ báo xếp vào hạng danh giá toán học mà người làm toán chuyên nghiệp muốn có đăng
Trong báo [3] có kết quan trọng, dẫn ta đến phép biến hình Nếu ta xét tứ giác tồn phần tạo bởi4đường thẳng`1,`2,`3,`4có điểm MiquelM, giao điểm của`ivới`j
Pij thìM P12·M P34=M P23·M P41 =M P31·M P24=pvà góc∠P12M P34,∠P23M P41,
∠P13M P24có chung phân giác` Do nếuΦlà hợp phép nghịch đảo cựcM, phương tíchpvới phép đối xứng qua`thì
Φ :P12, P23, P317→P34, P14, P24
cũng từ mà Φbiến`4 thành đường tròn ngoại tiếp tam giác tạo bởi`1,`2,`3 Bài báo rằngΦbiến đường tròn Miquel thành đường thẳng Steiner
Trong Định lý 2,Achính điểm Miquel tứ giác toàn phần(BZ, CY, P B, P C) Tài liệu
[1] J R Musselman and R Goormaghtigh, Advanced Problem 3928, Amer Math Monthly, 46 (1939) 601; solution, 48 (1941) 281–283
[2] K L Nguyen, A Synthetic Proof of Goormaghtigh’s Generalization of Musselman’s The-orem,Forum Geom., (2005) 17–20.
[3] J W Clawson, The Complete Quadrilateral, Ann Math, Second Series, Vol 20, No 4 (Jul., 1919), pp 232–261,http://www.jstor.org/stable/1967118